Главная Mobile Контакты NSFW Каталог Пожертвования Купить пасскод Pics Adult Pics API Архив Реквест доски Каталог стикеров Реклама
Доски

[Ответить в тред] Ответить в тред

Check this out!


<<
Назад | Вниз | Обновить тред | Автообновление
550 | 45 | 145

Тупых вопросов тред Аноним 28/03/18 Срд 15:06:43  435409  
37f733ccf06b57a[...].jpg (144Кб, 1024x628)
Предыдущий:
https://2ch.hk/sci/res/432585.html
Аноним 28/03/18 Срд 15:31:21  435413
Внимание, вопрос!
Мы залезли на суперкорабле за горизонт событий сверхмассивной чёрной дыры (ну чтобы нас на куски не порвало при пересечении горизонта). И вот вспомнили что у нас установлен двигатель алькубьерре. Вопрос: обладая неограниченной мощностью алькубьеровского движка - сможем ли мы вылезти из-под горизонта?
Аноним 28/03/18 Срд 15:41:57  435414
>>435413
Не уверен, но вроде как да. С таким движком получится, что мы как бы и не были в ЧД, а горизонт событий расступался перед нами.
Но есть нюанс - его нельзя просто так включить или выключить.
Аноним 28/03/18 Срд 16:52:18  435418
Как возникли самые крепкие яды типа ботулотоксина, тетрадотоксина и тд? Усиливались по нарастающей плавно/резко или чисто случайно возникли? Наука может ответить, как давно возникли, какой яд был предшественником данному? И можно ли, зная весь его эволюционный генезис, повторить цикл производства то же в отношении всех сложных природных веществ, которые не способно синтезировать химическое человечество?
Аноним 28/03/18 Срд 17:04:23  435420
>>435418
По разному же. Хотя резко и чисто случайно самые сильные яды не появятся, скорее просто всякие мерзкие на вкус бяки. Бывает ещё горизонтальный перенос, когда один вид копипастит у другого сырцы для производства ядов. Бывает, что вид сам по себе ядовитый, но яд не производит, а потребляет с пищей и затем выделяет.
Повторить цикл производства гипотетически возможно для чего угодно, но намного проще взять стандартные E.Coli, впихнуть в них соответствующие гены и дать и похавать.
Аноним 28/03/18 Срд 17:15:31  435421
Что будет, если, гипотетически, больному наследственным заболеванием исправить дефективный ген во всех клетках? Выздоровеет или помре?
Аноним 28/03/18 Срд 17:33:54  435422
>>435421
Почти всегда - выздоровеет, т.к. большая часть генетических заболеваний - это нокаутированный ген, т.е. белков, за которые этот ген отвечает, просто нет. Ещё здоровенный кусок таких болезней - нарушения регуляторных участков, из-за которых целевой белок или недорабатывает, или перерабатывает, или неправильно собирается. Но в любом случае, если сделать как надо - конечно всё начнёт работать как надо.
Гипотетически возможно, что из-за каскада взаимосвязанных регуляций ошибки какого-то гена или даже множества генов могут приводить к компенсаторным реакциям и внезапное появление этого гена приведёт к какому-нибудь дизбалансу, но это вообще дико редкое событие должно быть.
Аноним 28/03/18 Срд 18:43:46  435423
А я люблю обмазываться не свежим эфиром и мечтать. Каждый день я хожу по земле с синим соленоидом для эфира и собераю в него весь эфир, который вижу. На два полных соленоида целый день уходит. Зато, когда после тяжёлого дня я прихожу домой, иду в лабораторию, включаю осциллограф …ммм и соединию катушки с тороидами. И мечтаю, представляя, что меня поглотил единый организм эфир. Мне вообще кажется, что катушки, умеют думать, у них есть свои семьи, города, чувства, не игнорируйте их, лучше приютите у себя, говорите с ними, ласкайте их…. А вчера в лаборатории, мне преснился чудный сон, как будто я нырнул в море, и оно прератилось в эфир, рыбы, водоросли, медузы, все из эфира, даже небо, даже Аллах!.
Аноним 28/03/18 Срд 20:26:51  435426
Интересует пластичность металлов и сплавов для нубов. Что такое вязкость и хрупкость?
В каких единицах измеряется? Относится ли к этому же удлинение без разрушения металлов и сплавов? Есть ли справочники с коэффициентами для разных температур?
Аноним 28/03/18 Срд 21:03:07  435428
>>435426
иногда задумываюсь, как что-то, с не с сплошной, однородной структурой, а состоящее из отдельных зёрен, может иметь просто хоть сколь-нибудь значимую пластичность, не говоря уже о такой большой, как у металлов, а не рассыпаться уже при небольшой дефомации
Аноним 29/03/18 Чтв 11:01:43  435442
>>435426
>Что такое вязкость и хрупкость?
Если для нубов - сравни пластилин и стекло.
>Относится ли к этому же удлинение без разрушения металлов и сплавов?
Да, но там несколько параметров, конкретно это - прочность на растяжение.
>Есть ли справочники с коэффициентами для разных температур?
Есть.

>>435428
>как что-то, с не с сплошной, однородной структурой, а состоящее из отдельных зёрен, может иметь просто хоть сколь-нибудь значимую пластичность
Найди металлический(стальной, ферритовый) песок(на любом заводе где что-то вытачивают такой есть), возьми магнит и положи его в песок, а затем попробуй его помять - вот тебе модель того что происходит внутри металлов при деформации.
Аноним 29/03/18 Чтв 15:52:02  435453
Чем обмазаться для построения мммаксимум реалистичных солнечных систем? Шоб прямо математика из всех щелей. Из доступного в инете, да.
Для говнокодинга.
Аноним 29/03/18 Чтв 16:25:57  435455
>>435413
По моему этот вопрос можно прочитать как: превысит ли скорость нашего движла, силу гравитации чёрной дыры. Очевидно же что нет, она стремится к бесконечности. Да и откуда ты там вылезать собрался. Пространство за пределами горизонта свернуто в сингулярность. Имхо
Аноним 29/03/18 Чтв 17:23:26  435457
>>435453
Кеплеровской механикой и Patched Conics для орбит, ньютоновской физикой для остального. Большего ты на домашнем компе в любом случае не добьёшься.
Аноним 29/03/18 Чтв 17:28:20  435458
А кто-нибудь задумывался - почему всё существует? Я имею в виду вообще что угодно - люди, планеты, звёзды, галактики - тут всё просто, законы природы и щепотка энергии, и хуй бы с ним. Но почему есть хоть какие-то законы вообще, почему есть хоть что-то? Ведь с точки зрения современной научной методологии наиболее рациональным было бы абсолютное несуществование чего бы то ни было - как простейшее и наиболее вероятное состояние.
Но почему-то это, очевидно, не так. Какие есть соображения на этот счёт?
Аноним 29/03/18 Чтв 17:38:45  435461
>>435458
Как-то раз собрались признанные учёные и решили не ебать мозги этим вопросом. И придумали отмазку "антропный принцип". А дальше случился консенсус. Суть консенсуса - одни учёные решили, а все остальные молча согласились. Кто не согласился с консенсусом - тех записали в маргиналы, т.е. в петухи.
Таким образом по нынешним понятиям рассмотрение данного вопроса считается зашкваром.
Аноним 29/03/18 Чтв 17:51:04  435463
>>435461
Антропный принцип не канает - для него нужна некая над-вселенная, в которой будут перебираться варианты, среди которых будет такой, в котором возможно существование нас. Т.е. всё равно что-то да существует, что объясняет нас. И возникает вопрос - почему существует это что-то надвселенское.
Аноним 29/03/18 Чтв 17:59:44  435464
>>435413
Есть мнение, что суперкорабль объект, подходя снаружи к горизонту дыры, сгорит нахуй из-за резкого возрастания космической радиации. Почему? Рассмотрим упрощённый пример. Будем считать силу космической радиации в окрестностях чёрной дыры константой Ie. Тогда удельная плотность энергии этой космической радиации, принимаемой суперкораблём за единицу времени E`=Ie⋅t` Как известно, при приближении к горизонту собственное время суперкорабля замедляется. В следствие этого возрастает количество радиации, принятой суперкораблём за единицу собственного времени t``. В какой-то момент, предшествующий пересечению горизонта, поглощаемая суперкораблём энергия космической радиации E``= Ie⋅t`` достигнет такой величины, что суперкорабль испарится. В следствие испарения нарушится структурная целостность суперкорабля, что приведёт к невозможности функционирования этого суперкорабля. В таком случае, к тому моменту, когда суперкорабль пересечёт горизонт, он будет не в состоянии выбраться обратно ни при каких обстоятельствах.
Аноним 29/03/18 Чтв 18:01:12  435465
>>435463
Ты зашкварился.
Аноним 29/03/18 Чтв 20:16:34  435472
>>435409 (OP)
Что такое вид? как понять какого я вида? не надо мне говорить, что и дауну очевидно что человека я вид, у нас наука и требуется чёткое формальное определение, все с которыми я сталкивался до этого были расплывчатым говном.
Аноним 29/03/18 Чтв 20:33:48  435475
>>435472
> Что такое вид?
Это можно узнать в Википедии
> как понять какого я вида? не надо мне говорить, что и дауну очевидно что человека я вид, у нас наука и требуется чёткое формальное определение, все с которыми я сталкивался до этого были расплывчатым говном.
Homo sapiens. Это тоже есть в вики.
Аноним 29/03/18 Чтв 21:09:51  435480
>>435475
То есть ты не знаешь? Зачем тогда пишешь? И в википедии этого не узнать. Считаешь иначе? давай читани википедию и предложи определение вида как ты его понял я его обосру или боишься? вот и бойся, молча.
Аноним 29/03/18 Чтв 21:20:28  435482
>>435472
А нет чёткого формального определения. В таксономии постоянно по этому поводу споры и баталии разных масштабов, потому что с одной стороны каждый хочет назвать любую букашку с чуть другим расположением пятен новым видом в свою честь, с другой стороны именно отсутствие чёткого определения вида и стретьей - здравый смысл и непомерное разнообразие названий. Побеждает, обычно, случай.
А чёткого определения нет, потому что природа регулярно проявляет наплевательское отношение ко всем нашим определениям и критериям и каждый раз приводит нечто такое, что по определению должно бы считаться, к примеру, одним видом, но фактически имеет существенные различия и в один вид никак не вписывается.
Аноним 29/03/18 Чтв 21:28:00  435483
1024px-petersin[...].jpg (326Кб, 1024x1024)
>>435475
Открываешь википедию
@
Вид - группа особей с общими морфофизиологическими, биохимическими и поведенческими признаками, способная к взаимному скрещиванию, дающему в ряду поколений плодовитое потомство, закономерно распространённая в пределах определённого ареала и сходно изменяющаяся под влиянием факторов внешней среды.
@
так так так
@
Гомо сапиенсы и Неандертальнцы были разными видами
@
Но лже учнёые твердят что гомо сапиенсы имеют неандертальские гены и регулярно с ними скрещивались
@
понимаешь что эволюция человека полная лож согласно определению вида принятого в биологии

Аноним 29/03/18 Чтв 21:29:52  435484
>>435472
Вот тебе пример: есть лягушки, которые образовались в результате гибридизации двух разных видов. При этом они сами не способны оставлять жизнеспособное потомство, а размножаются путём скрещивания с одним из двух исходных видов (при этом хромосомный набор другого вида в их клетках кроссинговеру не подвергается, т.о. скрещивание происходит только с половиной хромосомного набора). И они существенно отличаются от обоих видов и имеют относительно стабильную популяцию. Как под это подогнать определение, чтобы не разрушить остальную классификацию и не пересматривать заново каждый вид - никто толком не знает. А это лишь один из примеров.
Аноним 29/03/18 Чтв 21:42:47  435485
>>435484
Это гипотетичекий пример или реальный? Что лягушки?
Аноним 29/03/18 Чтв 21:43:07  435486
>>435485
что за лягушки?*
Аноним 29/03/18 Чтв 22:02:21  435487
>>435485
Реальный. Не помню название вида, увы. Но с ними отдельный прикол был ещё и в том, что второй предковый вид вовсе вымер, но сохранился в геноме этого гибридного вида. Можешь попытаться загуглить, или я попозже поищу, сейчас с телефона неудобно.
Аноним 29/03/18 Чтв 22:10:41  435488
>>435485
Впрочем, первая же ссылка: http://elementy.ru/novosti_nauki/433147/Italyanskaya_ozernaya_lyagushka_vymerla_no_genom_ee_zhivet
Там же есть и ссылка на другую статью в частности о проблеме классификации видов. Но на английском.
Аноним 29/03/18 Чтв 22:27:35  435490
>>435482
Одна из проблем - систематика разработана еще до формулирования теории эволюции. Попытки привести ее в соответствие эволюционной картине ситуацию только усугубили. Хотя удалось избавится от бессмысленной произвольности принципов группировки (например, пришлось избавится от такого понятия как "рыбы вообще") но теперь как только мы начинаем рассматривать видообразование и эволюционную историю, систематика раскрячивается в залипает. Хотя бы потому, что в ней вообще не подразумевается возможность образования групп высоких рангов.
Т.Е. любой вид входит во всю иерархию, и потому не может "подняться" в ней. Но и бесконечное дробление на под-подвиды тоже не работает, особенно при взгляде в прошлое.
Поясню.
Любая группа обязана включать в себя всех потомков общего предка. Например, если общим предком всех птиц был динозавр, то мы обязаны включить птиц в группу динозавров.
Человек разумный относится к роду Люди (Homo Sapiens, род Homo).
Но предком всего рода Homo был один из видов австролапитеков.
Соответственно, мы должны либо упразднить род Homo, включив людей в род Австролапитеков, либо поднять австролапитеков до "надрода" (нет, триба уже занята) и включить предковый вид в род Homo. И это нужно будет проделать, по сути, для каждого периода истории, в котором произошло достаточно эволюционных изменений, что бы останки классифицировались как относящиеся к разным родам.
Аноним 29/03/18 Чтв 22:28:48  435491
>>435442
>возьми магнит и положи его в песок
>вот тебе модель того что происходит внутри металлов при деформации
т.е. по-твоему зёрна металлов держаться вместе за счёт магнетизма?
Аноним 29/03/18 Чтв 22:31:55  435492
В старостиа может, и раньше, когда мои члены будут ныть и болеть, смогу ли я на постоянной основе употреблять обезбаливающие, чтобы не чувствовать дискомфорта, а продолжать функционировать?

-они должны быть дёшевы
-они должны быть доступны
-побочные действия от них не должны влиять на мыслительную деятельность и не должны сильно ломать организм и сокращать оставшийся запас прочности
Аноним 29/03/18 Чтв 22:46:54  435493
>>435490
Что-то ты или сильно устарел, или просто выдумываешь. В таксономии нет никакого ограничения на название клад и видов, кроме того, что внутри одной клады нельзя использовать одинаковые названия.
Аноним 29/03/18 Чтв 22:54:15  435495
>>435490
Да и какое вообще отношение всё тобою описанное имеет к проблеме однозначного определения вида?
Аноним 29/03/18 Чтв 22:59:30  435496
>>435491
Лично я точного и однозначного ответа не знаю, но магнетизм как существенный фактор в прочности для меня выглядит довольно сомнительно, т.к. магнетизм по идее требует одинаковой ориентации кристаллов в зёрнах, а в таком случае это уже будут не зёрна, а один кристалл.
Могу предположить, что зёрна удерживаются вместе металлическими связями, так же как и в сплошном кристалле, только эти связи иррегулярны, вероятно, даже хаотичны (т.е. перестраиваются) и менее плотные. Ведь есть же эффект холодной сварки очищенных от оклидной плёнки металлов - в данном эффекте, насколько я понимаю, механизм тот же, который удерживает зёрна внутри одного куска металла.
Аноним 29/03/18 Чтв 23:11:09  435497
>>435496
лол, так ты просто слабенький теоретик, ну хоть сам написал про маловероятность влияния магнетизма в этом деле, и мне не пришлось писать про немагнитные металлы, лол
вот только б ещё сам написал, что предложенная тобой модель со стружкой и магнитом-крайне плоха ввиду полного неотрожения сути

всё бы хорошо, с этим прилипанием металла к металлу, вот только на шлифованном срезе(на котором соответственно разрезаны и сами зёрна) двух скрепляемых так кусков соприкасается именно металл с металлом, а между зёрнами-в металлах-всякая хня, и не о каком соприкосновении именно металла к металлу в этом случае речи вроде идти не может
Аноним 30/03/18 Птн 00:25:15  435500
>>435497
Мьсе, вам не кажется, что в данном треде может быть больше двух человек?
Автором ошибся. Я про стружку не писал.
Аноним 30/03/18 Птн 00:32:00  435501
>>435497
Ну, во-первых, шлифованный стрез в атмосфере Земли покрывается оксидной плёнкой в первые же микросекунды. Не полностью - полностью он покрывается немногим дольше - однако, этого достаточно, чтобы предотвратить образование или разрушить существующие связи.
Во-вторых, совсем не обязательно между зёрнами что-то находится. В сплавах - да, зачастую, примесные металлы находятся не в узлах кристаллической решётки, а именно на стыках между разнонаправленными зёрнами основного металла. Что, тем не менее, никак не мешает образованию связи (а, скорее всего, даже наоборот способствует образованию дополнительных связей и повышают плотность таких связей между соседними зёрнами; можно, кстати, проверить моё предположение, если загуглить какие металлы в малых концентрациях используются в сплавах и какова их химическая реактивность).
Аноним 30/03/18 Птн 01:40:25  435502
>>435483
Если ты чего-то не понимаешь то это не значит что учёные врут. Это значит что ты туповат маленько.
Аноним 30/03/18 Птн 01:45:10  435504
>>435502
Не уверен, но возможно это был сарказм на тему того, как всё печально в таксономии (а оно действительно печально, а точнее - находится в процессе долгостроя, когда старые морфологические устои не успели смениться эволюционными, как ворвалась геномика и тоже стала тоже всё по-своему перестраивать).
Аноним 30/03/18 Птн 06:46:23  435508
Можно ли сделать даигатель на фотонах? Сделать емкость из обычных зеркал и зеркала, где одна сторона зеркальная, а другая как окно
Фотоны же тоже обладают массой, значит должно полететь
Алсо, объект в космосе без рабочих двигателей будет лететь с неизменной скоростью, или он будет ускоряться/замедляться
Аноним 30/03/18 Птн 07:06:21  435510
>>435508
>Можно ли сделать даигатель на фотонах?
Можно, см. "Солнечный парус", "EmDrive"
>Алсо, объект в космосе без рабочих двигателей будет лететь с неизменной скоростью
Да. См. "1 закон Ньютона".
Аноним 30/03/18 Птн 08:43:02  435513
DZCFZ9eVMAAo-aI.jpeg (53Кб, 599x422)
Хуй знает в какой мне раздел, пусть тут тоже будет.

Почему человек имеет возможность довериться случаю, интуиции? Это же эволюционно невыгодно, ни одно животное так не поступает: собака, кошка, корова, ворона, все всегда подчиняется инстинкту, используют возможности мозга максимально. Тогда почему человек умеет лениться, умеет полагаться на случай? Это же нарушает логику, нарушает законы природы.

Ведь очевидно, что небольшой выброс гормона радости в кровь не сравним с результатом трудоёмкой работы. Почему же человек ИМЕЕТ ВОЗМОЖНОСТЬ выбрать первый вариант? Птица обязательно будет вить гнездо для своих яиц, а не надеяться, что случайно найдёт удачное сплетение веток посреди леса. Люди же часто рожают детей без достаточных условий для выживания, обрекая организм на смерть, что совершенно не вяжется с концепцией эволюции.

В чём заключается это отличие человека от животного?
От чего люди более развиты чем животные, но при этом менее логичны? Выходит, алогичность - венец биологической эволюции?
Аноним 30/03/18 Птн 09:42:27  435515
>>435513
Ну смотри: у тебя есть гипотеза эволюционной невыгодности такого-то типа поведения людей, но людей на планете уже под восемь миллиардов, а человеку как виду около полутора миллионов лет. Значит твоя гипотеза неверна?
>Ведь очевидно, что небольшой выброс [...]
Значит, не очевидно?
>[...] что совершенно не вяжется с концепцией эволюции.
То есть, ты утверждаешь, что знаешь некоторую истинную концепцию эволюции? И у тебя там с ней что-то не вяжется? С истинной-то моделью?
Аноним 30/03/18 Птн 10:16:08  435516
>>435513
Огромное количество факторов в жизни разумного существа трудновато подчинить четким программам. Значит, нужен управляющий компьютер, который может править собственный код без забитых заранее алгоритмов. На выходе получается то, что получается
Аноним 30/03/18 Птн 10:33:29  435517
>>435515
Спасибо.
> То есть, ты утверждаешь, что знаешь некоторую истинную концепцию эволюции?
Выживание вида, перенос генов?

Ещё один вопрос, который не даёт покоя: в чём фундаментальное отличие разумного существа от неразумного? В чём проявляет разум? Поведение, психические процессы, особые способности?

Пока создаётся впечатление, что никакого разума вообще нет, есть только несколько усложнённые инстинкты и рефлексы.
Аноним 30/03/18 Птн 11:22:52  435518
>>435517
Разум способен осознать сам себя. Осознать себя - значит мысленно выделить самого себя как отдельный объект.
Аноним 30/03/18 Птн 11:33:36  435519
>>435517
> Выживание вида, перенос генов?
Сапиенсы с этим успешно справляется.
> Ещё один вопрос, который не даёт покоя: в чём фундаментальное отличие разумного существа от неразумного? В чём проявляет разум? Поведение, психические процессы, особые способности?
В производстве орудий труда другими орудиями труда.
> Пока создаётся впечатление, что никакого разума вообще нет, есть только несколько усложнённые инстинкты и рефлексы.
Создаётся впечатление что ты дурак.
Аноним 30/03/18 Птн 11:37:26  435520
>>435519
> Создаётся впечатление что ты дурак.
Название треда видел?
Аноним 30/03/18 Птн 11:38:33  435521
>>435520
Тебе уже все объяснили а ты не понимаешь.
Аноним 30/03/18 Птн 11:39:34  435522
>>435518
Далее. Из способности разума выделять себя как отдельный объект исходит способность планировать свои действия. Из способности планировать действия исходит способность прогнозировать последствия своих действий. Из способности прогнозировать последствия своих действий исходит способность принимать разумные решения.
Возьмём к примеру воробушка как неразумное существо. Т.к. воробушек не осознаёт себя, то все его действия являются неразумными. Он просто берёт и действует, не беспокоясь о последствиях. Хочет кушать - кушает. Хочет вить гнездо - вьёт гнездо. Если ему стращно, он улетает или прячется. Когда он хочет ебаться, он не осознаёт этого. Он просто подлетает к тяночке и производит машинальные действия.
Аналогично глупый двачер может машинально вздрочнуть на видео с трапом.
Разумный человек, прежде чем дрочить, подумает и решит, дрочить ли на трапа или есть шанс выебать мамку ОПа. И примет соответствующее решение, основываясь на прогнозе последствий своих возможных действий. И всё это благодаря осознанию разумным человеком самого себя как личности.
Аноним 30/03/18 Птн 11:40:20  435523
>>435518
Осьминоги, дельфины, сороки осознают себя. Но они не разумны.
Аноним 30/03/18 Птн 11:41:48  435524
>>435519
>В производстве орудий труда другими орудиями труда.
Позвольте возразить. Знаю примеры человеков, которые не производят никаких орудий. И при этом считаются разумными.
Производство орудий - один из необязательных признаков разумности.
Аноним 30/03/18 Птн 11:48:21  435525
>>435523
>они не разумны.
Дельфины передают свой жизненный опыт новым поколениям. Можно считать их разумными.
Осьминоги и вороны не разумны. Особенно осьминоги, т.к. их жизненный цикл не позволяет им передавать свой опыт. Они тупо умирают во время нереста. Т.е. у них нет возможности развить мыслительную деятельность из-за биологических ограничений.
Сороки не осознают себя. Они узнают себя как отдельную единицу в составе группы. При этом не разделяют единицы на себя и всех остальных. Узнать и сосознать - две большие разницы.
Аноним 30/03/18 Птн 11:49:14  435526
>>435524
Это не значит что они этого делать не могут.
Аноним 30/03/18 Птн 11:51:00  435527
>>435525
Мы с антропной точки зрения рассматриваем или вообще? С человеком все давно понятно и разложено по полочкам. А с другой точки зрения мы и рассматривать не можем не рискуя пуститься в пустое словоблудие.
Аноним 30/03/18 Птн 12:11:01  435529
1454023122.png (420Кб, 640x472)
>>435523 Всё дело в том, что разумность, это не какая-то конечная инстанция, и граница между умом и разумом достаточно условна, хотя и имеет качественное отличие, соответственно здесь у нас видимо какие-то градации имеют место. Ну а homo sapiens это обычное наименование, как например морской конёк, понятно что к коням имеет далёкое отношение.
Аноним 30/03/18 Птн 12:12:47  435530
>>435518
>Осознать себя - значит мысленно выделить самого себя как отдельный объект
Тогда что делать со следующей ступенью? Когда разум может осознать что он сам себя осознает. И даже то что он может осознать это.
Когда появляется иттеративная бесконечность самоосознания?
Аноним 30/03/18 Птн 12:14:26  435531
>>435530
Это уже не наука а философия.
Аноним 30/03/18 Птн 12:16:10  435532
>>435531
В смысле? Это вполне себе конкретная граница. Животные способны к рефлексии? Могут ли они осознать своё сознание?
Аноним 30/03/18 Птн 12:26:09  435533
>>435532
Шимпанзе и гориллы могут. Например гугли горилла Коко. Она вполне осознавала прошлое, настоящее, будущее, рассказывала охуительные истории о себе. Другое дело что для того чтобы назвать ее разумным существом у нее не хватает гоминидных запчастей.
Аноним 30/03/18 Птн 12:31:27  435534
>>435533
Осознать себя может любое животное, уровни осознания разные.
Аноним 30/03/18 Птн 12:41:08  435535
>>435534
Это философия или эзотерика?
Аноним 30/03/18 Птн 13:15:14  435537
>>435535
Это то что означает осознавать. Здесь слишком многое на это понятие нагружается, какое-то особое вычленение себя, помнить прошлое, знать будущее, не слишком много? По себе могу сказать, помню себя в 1-2 года, и несколько моментов до 1, хотя казалось бы полуавтомат, ан нет.
Аноним 30/03/18 Птн 13:43:50  435538
>>435537
Это не имеет никакого отношения к разумности.
Аноним 30/03/18 Птн 13:53:46  435539
what-if-red-dot[...].jpg (81Кб, 600x788)
>>435538
Осознанность имеет непосредственное отношение к разумности, можно даже говорить что осознанность в себя её включает, возможно разумность и есть верхняя ступень осознанности, понятно что не отдельная ступень, а дорога уходящая в бесконечность...
Аноним 30/03/18 Птн 13:59:32  435540
>>435508
Откуда вы блять лезете? Фотоны не имеют массу, они имеют энергию и импульс.
Аноним 30/03/18 Птн 14:01:36  435541
>>435523
Это ты так считаешь, а они нет.
Аноним 30/03/18 Птн 14:12:08  435543
Чуваки, ваша осознанность, разумность, рефлексия - это просто долбанные слова, которые ваши предки выдумали задолго до вас. За слова отвечает кучка из нескольких миллионов нейронов, мозг кошки мог бы таких кучек вместить сотнями. И сами эти осознанность, разумность и рефлексия сводятся к словоблудию, которым человечество потешает своё ЧСВ и превозносит себя над всей планетой. А в реальности мозг наш, хотя и один из самых развитых, всё же едва лучше других млекопитающих и по большей части выигрывает за счёт количества, а не качества.
Современная наука только начинает осознавать, что граница между тем, что считалось разумом, и животным поведением - не то, что не чёткая, её и нет вовсе. Что весь наш вид оказался настолько успешным только из-за небольшого нововведения в виде речи.
Аноним 30/03/18 Птн 14:42:15  435544
>>435543
> А в реальности мозг наш, хотя и один из самых развитых, всё же едва лучше других млекопитающих и по большей части выигрывает за счёт количества, а не качества.
По такой логике выходит что самые интеллектуальные это киты.
А на деле человека делает человеком гоминидных триада.
Но в целом ты прав, а мамкины философы нет.
Аноним 30/03/18 Птн 14:42:34  435545
>>435544
> Гоминидная триада
Аноним 30/03/18 Птн 15:21:18  435546
>>435544
Я не имел в виду количество нейронов, само собой.
Аноним 30/03/18 Птн 17:34:02  435551
Верно ли, что если направить телескоп на солнце, чтобы его диск целиком покрывал поле зрения, то на глаз посмотревшего ниспадут живительные лучи температурой 5000 примерно (ну там какая-то часть поглотится воздухом и стеклом)?
Аноним 30/03/18 Птн 17:40:48  435552
>>435409 (OP)
Есть ли частицы с нулевой массой но при этом обладающие электрическим зарядом?
Аноним 30/03/18 Птн 17:50:09  435553
>>435552
В школьном учебнике говорят, что электрон и позитрон.
Аноним 30/03/18 Птн 18:13:34  435556
>>435553
>электрон и позитрон
>нулевой массой
)
Аноним 30/03/18 Птн 18:15:41  435557
>>435551
Это температура цвета, она и без телескопа такая:
https://ru.wikipedia.org/wiki/Цветовая_температура
Но от телескопа я бы всё-таки глаза поберёг на твоём месте анон, там фокусируется поток низкорассеяной плазмы и прямо в зрачёк - за пару секунд говорят выжигает, хотя смотря какая линза/восход/закат/ясность неба/ и тд., вообще для Солнца на телескопы специальные фильтры идут.
Аноним 30/03/18 Птн 18:19:41  435558
>>435557
Пару секунд? Даже на 50см тебе хватит доли секунды, чтобы прокипятить себе сетчатку. За пару секунд глаз успеет вздуться от давления образовавшихся газов.
Аноним 30/03/18 Птн 18:21:16  435559
>>435556
эх... Ну, здесь уже было три жертвы системы образования, которые считают, что фотоны имеют массу. Так что в принципе безмассовым электронам я теперь не удивляюсь.
Аноним 30/03/18 Птн 18:25:06  435560
>>435559
А на вопрос-то >>435553 ответ какой? Существуют? Могут существовать?
Аноним 30/03/18 Птн 18:27:30  435561
>>435560
Вроде бы нету. Но это не точно.
Аноним 30/03/18 Птн 18:29:07  435562
>>435553
фиговый у тебя учебник
Аноним 30/03/18 Птн 18:33:34  435565
>>435558
>Пару секунд?
>смотря какая линза/восход/закат/ясность неба/ и тд., вообще для Солнца на телескопы специальные фильтры идут.
Но в целом солидарен - с глазами рисковать не стоит, мало ли чего там ещё в этом солнечном свете может быть.
Аноним 30/03/18 Птн 18:36:33  435566
>>435565
Не, ну окей, через рассветную дымку, когда солнце визуально только взошло (а фактически всё ещё на пару градусов ниже горизонта) и через бинокль - возможно, пронесёт, ну или хотя бы пару секунд можно продержаться. Почти во всех остальных случаях - инвалидность.
Аноним 30/03/18 Птн 18:53:01  435568
3.gif (2Кб, 298x149)
>>435562
Вспомнил схему бета-распада и пизданул, приношу извинения.
Аноним 30/03/18 Птн 18:57:52  435569
>>435566
Не, я бы на твоём месте не стал, мало ли там какая вспышка радиации входа на рецепторах перемагнитит и все цвета съедут, или вообще изображение на 90 или 180 градусов повернёт, мало ли что ещё.
Аноним 30/03/18 Птн 19:01:23  435570
image.png (164Кб, 640x400)
>>435569
Ты как капчу взломал, бот?
Аноним 30/03/18 Птн 19:18:29  435572
>>435570
Ладно, ладно, но всё-таки, по-осторожней там, с глазами шутки плохи, за монитором тоже много сидеть вредно.
Аноним 30/03/18 Птн 20:40:54  435573
https://www.youtube.com/watch?v=ewVHImqQ_zs
Аноним 31/03/18 Суб 00:00:39  435578
Прочитал про квантовую запутанность. Типа если запутать два кванта, то распутывание одного приведёт к моментальному распутыванию другого. Короче. Объясните мне, почему нельзя с помощью этого эффекта передавать информацию. Запутываем два кванта, один даём Васе, второй Пете. Петя улетает на Луну и там распутывает свой квант. Вася об этом узнаёт сразу, без секундного лага полёта света до Луны.

Где подвох?
Аноним 31/03/18 Суб 00:09:46  435579
>>435578
Их не "распутывают". Просто их запутанность разрушается при взаимодействии с другими частицами, но разрушается довольно странным образом, похожим на эдакое моментальное взаимодействие между запутанными частицами.
Если говорить совсем простым языком, то квантовая запутанность - это когда у тебя есть два 1носка. Как только ты надеваешь один на правую ногу, моментально и на любом расстоянии второй носок автоматически становится левым. Если сможешь с помощью этого передать информацию - беги за нобелевкой.
Аноним 31/03/18 Суб 00:22:44  435580
>>435578
Ах, ну и да, Вася не узнаёт об этом никак, абсолютно, совершенно, он ни сном ни духом не знает о том, что происходит с частицей Пети. Однако, при этом, Петя знает какая частица у Васи - знает как только измерит свою частицу. И ещё если Петя скажет Васе - я свою частицу измерил, она такая. И Вася теперь знает, что в тот момент (и какое-то время после него - до тех пор, пока частица с чем-нибудь не провзаимодействует) его частица была ровно противоположной частице Пети. Вот это и есть квантовая запутанность.
Здесь, мой дорогой читатель, ты, как и каждый из самых образованных физиков когда-то, скорее всего будешь думать "хм, ну наверное запутанные частицы всегда были такими, какими их и измерили" и будешь не прав - до момента измерения они не были никакими определёнными. Допустим, есть правая и левая частица, как с носками. Самым логичным кажется, что они сразу с момента своего появления правые или левые - но нет, они одновременно и правые и левые обе. В абсолютно одинаковых условиях при абсолютно одинаковом измерении мы можем получить разные значения без малейшего намёка на то, почему они могут различаться. Однако, как только мы один раз измерим частицу - каждое следующее измерение будет иметь в точности то же значение. Это и называется суперпозиция. Ещё можно было бы предположить, что частицы осцилируют - т.е. каждая из связанных частиц просто очень-очень быстро, но одновременно меняют значение. И снова нет - как только мы измерим одну из частиц, значение второй при каждом измерении (даже при первом) будет всегда противоположно значению первой, хотя вроде бы до первого измерения на неё никак не воздействовали и если бы были какие-то осциляции, мы бы их увидели.
Вот как-то так.
Тут ещё есть неравенство Белла, но как его на бытовом уровне объяснить я не знаю, поэтому не возьмусь, но если захочешь, анон, я объясню хотя бы как могу.
Аноним 31/03/18 Суб 00:31:26  435581
>>435578
Ловишь ты фотон, и такой последовательность его спина видишь как абсолютно рандомную хуйню. И как ты собрался с помощью этого что-то передавать?
Аноним 31/03/18 Суб 00:47:24  435582
>>435580
Лел, это напоминает какую-то прогрузку игровых текстур по требованию. А пока на них никто не смотрит, они и не существуют.
Аноним 31/03/18 Суб 00:51:29  435583
>>435582
Частицы - точно существуют. Их измеряемые параметры... ну, по идее, существуют все сразу и одновременно, включая взаимоисключающие.
В принципе, гипотетически, может быть и существует какое-то одно значение. Но нет ни одной толковой теории, которая могла бы это значение предсказать до измерения.
Аноним 31/03/18 Суб 05:49:43  435591
Объясните зачем нужен закон Ома если в деятельности все наоборот: чем выше напруга, тем меньше ток? Я по отведя что это зависит от мощности электроприемника, но вещи электроприемник это конечный пункт распределительной сети. Где тогда закон Ома актуален?
Аноним 31/03/18 Суб 05:51:32  435592
>>435591
>Я по отведя
Я понимаю
Аноним 31/03/18 Суб 06:49:43  435595
>>435591
Чой-та у тебя ток меньше должен стать? И откуда ты электроприёмник выдумал, наркоман? Ток течёт по цепи. В ней нет приёмника, в ней есть сопротивление, сиречь, нагрузка. Либо совершающая полезную работу, либо просто нагревающаяся, не суть важно. Пустишь больше напряжение - увеличится ток, увеличится совершаемая полезная работа или нагрев.
Аноним 31/03/18 Суб 08:17:54  435596
>>435595
Твой обогреватель в соседней комнате на 1000 Вт. Соответственно сила тока протекающая по проводу от сети 220В — 4,5 А. Уменьшишь напряжение в два раза получишь 9,1 А.

То же самое и в трансформаторных подстанциях, расположенных во дворах. Если изменить напругу и сила тока на отходящих линиях низкого напряжения, то получишь ту же самую закономерность. Там где ток выше, напряжение ниже и наоборот.

Аноним 31/03/18 Суб 08:18:42  435597
*Если изменить
Аноним 31/03/18 Суб 12:16:16  435603
>>435409 (OP)
Анон, подскажи хорошие книги на тему методологии научного познания для людей не близких к науке. Не конкретной науки а науки в целом.
Не первые попавшиеся в интернете, а хорошие. Хорошие в том плане, что вы ими восхищаетесь и считаете полезными для чтения другими людьми.
Аноним 31/03/18 Суб 12:43:19  435605
>>435596
Ты либо вопрос задавай, либо отвечай правильно, либо вали отсюда, неуч.
Он 1000Вт при условии, что в сети у тебя 220В. Поезжай в ПГТ, особенно где-нибудь в районе краснодарского края или ростовской области, и посмотри летним вечером вольтаж в сети, яркость лампочек и скорость вентилляторов.
В трансформаторах у тебя с падением напряжения увеличивается ток только потому, что на первичной обмотке напряжение не меняется. Будешь менять напряжение на первичной обмотке - у тебя пропорционально будет менятся напряжение и ток на вторичной.
Здесь я покрываю тебя хуями за то, что твой вопрос не вопрос, а утверждение, что весь мир двести лет электричество неправильно использует, закон ома фигня, максвелл даун, а ты у мамки гений (нет).
Аноним 31/03/18 Суб 12:45:06  435606
>>435603
Вроде бы "Гарри поттер и методы рационального мышления", но я разные, в т.ч. и совсем не хорошие отзывы на её тему видел. Сам не читал.
Аноним 31/03/18 Суб 12:56:21  435608
>>435583
>Но нет ни одной толковой теории, которая могла бы это значение предсказать до измерения.
Вот этого я никогда не понимал. Каким образом акт "измерения" вдруг перенёсся будто бы в ряд каких-то физических терминов. Что тут понимается под "измерением"? Физическое воздействие, посредством которого производится измерение? Оно не может быть разным? Или при измерении важен сам факт "узнавания данных"? Останется ли измерение измерением, если человек не посмотрит на результат измерения? А если человек идиот? А если замер сделает машина?
Аноним 31/03/18 Суб 13:23:47  435610
>>435606
>"Гарри поттер и методы рационального мышления"
Это шутка такая? Если нет, то под
"Не близкий к науке" подразумевается, что у меня есть гуманитарная вышка (российская), но нет ученой степени, а не то, что я в 8 классе учусь.
Аноним 31/03/18 Суб 13:25:26  435612
>>435605
В теории всё так, только почему-то на практике всё наоборот.
В ТП две отходящие линии НН: от одной 15 домов запитано, от другой 3. Там где 15 домов нагрузка 30А, напряжение 230В (В ТП), а там где 1 дом нагрузка 6А, напряжение 240В.
Аноним 31/03/18 Суб 15:48:58  435624
>>435612
Ну так трансформаторная подстанция во многом для этого и используется - чтобы выравнивать напряжение в зависимости от потребления. То, что с малой нагрузкой у тебя 240 - так это она хреново отработала, должно быть 230. А в остальном - чем больше потребление (сопротивление), тем ниже напряжение, ток остаётся неизменным, подстанция видит снижение напряжения и повышает его - ток по закону ома увеличивается, чтобы при данном сопротивлении выдавать стандартное напряжение. Во внешнем для подстанции контуре при этом так же снижается напряжение. Электростанция или другие сегменты электросети тоже детектят это и стремятся увеличить ток. Так всё и работает. Т.е. в конечном счёте напряжение в сети зависит от тока и напряжения источников питания - электростанций, и зависят именно по закону Ома. Но из-за сложности электросетей и механизмов их автоматической регуляции всё выглядит так, как ты видишь.
Аноним 31/03/18 Суб 16:42:56  435627
>>435608
Самым что ни на есть непосредственным образом.
Как ты хочешь что-либо измерить? Приложить линейку? Окей - ты приложил убермикроскопическую линейку и... не видишь ничерта, потому что чтобы считать с неё показания, нужны фотоны. Ты светишь фотонами - всё, ты провзаимодействовал с частицей. Хочешь измерить скорость частицы - придётся поставить на её пути препятствие в виде детектора-калориметра. Хочешь измерить её положение? Придётся остановить её и долго и упорно бомбить какими-нибудь другими частицами, отлавливая уже их и по их отклонению расшифровывая где произошло столкновение. И так везде. В физике невозможно провести измерение, не взаимодействуя с объектом и не влияя на все его остальные свойства.
Да, измерение остаётся, смотрит человек или нет, следовательно и последствия измерения тоже остаются. Под наблюдателем всегда понимают именно детектор.
Хотя есть какая-то наркомания про откложенный выбор в эксперименте с двойной щелью, которую я не очень раскурил, но может быть здесь появятся люди, которые поняли это лучше и которые смогут объяснить его на пальцах.
Аноним 31/03/18 Суб 19:04:51  435631
>>435608
>Останется ли измерение измерением, если человек не посмотрит на результат измерения?
Да. Останется. Самый простой способ объяснить, это через описание измерения через уравнение Шредингера.
Аноним 31/03/18 Суб 19:52:47  435633
Чет кекнул с вашей квантовой физики. "Мы не можем измерить положение/скорость электрона, не подвинув его - значит у электрона на самом деле нет положения и скорость! Нет! Врёти! Ррряяя!"
Аноним 31/03/18 Суб 20:06:16  435634
>>435633
Мы не можем измерить положение и скорость спермы, летящей из хуя тебе на лицо, не надрочив хуй - значит, спермы на самом деле нет, если не надрочить хуй. Поняла, машка?
Аноним 31/03/18 Суб 20:06:58  435635
>>435633
>"Мы не можем измерить положение/скорость электрона, не подвинув его - значит у электрона на самом деле нет положения и скорость! Нет! Врёти! Ррряяя!"
Это чушь конечно. Кто так объясняет, тот профан.
Есть эксперименты тонкие, когда "трогают" уже после. А интерференция исчезает до измерения.
Ну и ЭПР парадокс показывает что скрытых параметров нет.
Аноним 31/03/18 Суб 20:32:35  435639
>>435633
>Мы не можем измерить положение/скорость электрона,
А зачем?
Аноним 31/03/18 Суб 21:10:28  435643
>>435579
чёт не вижу сходу так никаких причин невозможности передавать инфу благодаря этому
или ты про невозможность технической реализации этого сейчас, а не про принципиальную невозможность?
Аноним 31/03/18 Суб 21:23:22  435645
>>435643
Да мне тоже нифига не ясно, но я тут книжку по квантовой механике читаю, чтобы в пух разнести местных профанов.
Аноним 31/03/18 Суб 21:49:57  435647
>>435643
Расскажи как ты видишь себе передачу информации носками)
Аноним 01/04/18 Вск 08:29:18  435657
>>435643
https://www.youtube.com/watch?v=Y-KOPGuLY9c
Посмотри первую половину, там доходчиво показано, в чём суть спора Эйншнейна и Бора о квантовой запутанности.

p.s. А вот эксперимент Белла я так и не понял.
Аноним 01/04/18 Вск 10:23:28  435664
Он прав?
https://twitter.com/US__progress/status/980146693600632835
Аноним 01/04/18 Вск 11:15:32  435665
>>435647
Есть два состояния носка-тоже самое как ноль единица в классических современных способах передачи информации;
Один носок из пары может менять своё состояние из одного в другое, в зависимости от того, как где-то далеко поменял своё состояние второй носок-прям как подача нуля или единицы от источника к датчику в современных системах передачи информации электричеством или светом. Только тут убирается промежуточная цепочка-путь/перемещение сигнала.

>>435657
не, может всё это не так, я про то что как тут это описывают аноны-кажется что вполне можно передавать инфу благодаря этому
Аноним 01/04/18 Вск 11:22:16  435666
>>435664
>Он прав?
Узнай оценки мировых запасов кобальта. Узнай его необходимое количество для производство 1-го аккумулятора. Узнай сколько их в среднем нужно для 1-й машины. Посчитай.
В чём проблемс?
Или хочешь ничего не делать и положиться на ответы анонов? Не надо так. они тебя обманут, а ты будешь плакать
Аноним 01/04/18 Вск 11:30:43  435668
>>435665
Фокус в том, что Вася из этого эксперимента >>435578 просто не знает, какого результата ему надо ждать. Даже если он померяет свой электрон и получит, скажем, спин +1, то он всё равно не узнает, получился такой результат случайно или ввиду того, что Петя на Луне уже померял свой электрон и получил там -1.
Аноним 01/04/18 Вск 11:37:54  435669
>>435668
Вообще-то нет, я сказал чушь. Вася ведь может подождать оговоренного момента времени, когда Петя уже точно будет должен измерить свой электрон.
А если договориться также, что Петя, получив после измерения -1, начинает ебсти себя дилдаком в жопу, то Вася, проведя измерение позднее, будет точно знать, пидор Петя или нет. Вуаля - информация передана.
Аноним 01/04/18 Вск 12:31:28  435674
>>435669
>информация передана.
cо скоростью полёта на луну
Аноним 01/04/18 Вск 15:08:26  435683
>>435674
Ты сказал лишь бы что ляпнуть? Укажи на противоречие, маня. Если Пётр и Василий договорятся, что Пётр измеряет свой электрон в 9:00, а Василий в 10:00, то, измерив свой электрон в 10, Василий будет знать, что получилось у Петра и что Пётр уже час долбит себя в сраку.
Аноним 01/04/18 Вск 15:13:21  435685
image.png (4971Кб, 2272x1704)
Вопрос про черныя дупки.
Всем известно, что чем ближе к горизонту, тем медленнее время. Собственно горизонт это такая умозрительная поверхность, где скорость времени становится нулевой, т.е. время останавливается и останавливаются физические процессы.
А дальше что? Что происходит со временем под горизонтом? Имеет ли вообще смысл рассуждать о том, что там что-то есть и что-то происходит?
Аноним 01/04/18 Вск 15:24:37  435687
А что если Петя измерил, получил -1 и передумал ебсти себя в жопу.
А Вася измерил, получил +1 и думает, что Петя пидор.
Получаем парадокс: Вася думает, что Петя пидор, но Петя не пидор.
Хуйня эта ваша квантовая телепортация.
Аноним 01/04/18 Вск 15:28:21  435688
>>435669
Это не передача информации, это протокол, отпределяющий действия в зависимости от внешнего события. С тем же успехом между ними можно поставить маячок, который будет в обе стороны посылать случайный сигнал, а на базе этого сигнала принимать решение. Но в таком случае это почти то же самое, что держать у Васи и Пети два таких генератора, когда-то давно синхронизированных между собой. Тут нет передачи информации совсем.
Аноним 01/04/18 Вск 15:52:45  435692
Блядь, учусь на физмате но понять нихуя не могу
наливаю воду в кружку из металлического чайника, горлышко у него холодное, а сама вода -- кипяченая. когда вода идет по горлышку во все стороны летят капли кипящей воды.
Вопрос: за счет каких сил капли разлетаются во все стороны? что придает им импульс? из-за разности давлений? молекулы воды бьются об металлическую поверхность и срабатывает закон сохранения импульса или че? я вахуе
Аноним 01/04/18 Вск 16:01:47  435694
>>435687
И тут нам поможет теория относительности от Альберта Германовича.
Если до измерения Петя был в квантовой суперпозиции, т.е. одновременно пидор и не пидор, то после измерения ситуация кардинально меняется.
Всё относительно. Со своей точки зрения Петя не считает себя пидором. Но с точки зрения Васи Петя всё таки пидор. Т.е. уже имеем дело с релятивистскими эффектами.
Аноним 01/04/18 Вск 16:03:18  435695
>>435692
А не пиздишь ли ты часом?
Что-то я такого эффекта не наблюдал.
Аноним 01/04/18 Вск 16:07:54  435696
image.png (1124Кб, 1600x1130)
>>435685
Бамп вопросу. Ладно, хуй с ним, с падающим в дупу мужиком.

Что становится с веществом звезды во время превращения в дупу? Оно полностью замирает или же всё таки с ним что-то происходит?
Аноним 01/04/18 Вск 16:37:13  435701
>>435692
ты уверен что он холодный? те части кастрюли, например, которых не касается вода, т.е. выше её уровня, зачастую горячее 100 градусов
и если взять кастрюлю нагретой, но ещё не вскипевшей воды, и поболтать в стороны, то попадая на стенки, которых до этого не касалась, выше, вода будет вскипать на них и шипеть
Аноним 01/04/18 Вск 16:41:00  435702
>>435701
типа да, похоже на то когда масло шкварчит на разогретой сковороде
просто тут то вода, поэтому я удивлен
кстати возможно, типа горлышко >100 тепло передается льющейся воде, вода вскипает но в воздухе тут же конденсируется, хотя такие фокусы выглядят не очень правдоподобно как по мне
Аноним 01/04/18 Вск 16:41:53  435703
>>435696
>Что становится с веществом звезды во время превращения в дупу?
а уверен что есть вообще оно, это вещество?
кмк только или чистая энергия
или вообще ничего-ЧД разрушается сразу же, мгновенно после своего образования, но из-за остановки времени, для стороннего наблюдателя существуют вечно
Аноним 01/04/18 Вск 16:44:53  435704
>>435702
>типа да, похоже на то когда масло шкварчит на разогретой сковороде
>просто тут то вода, поэтому я удивлен
>вода вскипает но в воздухе тут же конденсируется
нахуй ей конденсироваться для таких эффектов? просто расширяющийся водяной пар, как пузыри в кипящей воде, только сильнее, т.к. очень быстрое испарение из-за малого объёма воды, раскидывает воду
Аноним 01/04/18 Вск 16:45:27  435705
>>435703
Что значит "разрушается"?
Я бы понял, что вещество переходит из одного фазового состояния в другое. Типа как вода в пар, пар в плазму. Но всё рано же при этом остаётся какое-то вещество, в котором происходят какие-то дальнейшие процессы.
Что значит "разрушается"?
Аноним 01/04/18 Вск 16:54:21  435706
>>435704
ну это правдоподобнее, пасибо, буду спать спать спокойно
Аноним 01/04/18 Вск 17:52:58  435710
>>435705
до каких пор вещество это вещество по-твоему?
до различимых кусков, типа "кусок железа"?
до молекул?
до атомов, типа "атом железа"?
до частиц? до каких? до нейтронов? до кварков?
может до струн?..
Аноним 01/04/18 Вск 17:54:52  435712
>>435706
>пасибо
пж
Аноним 01/04/18 Вск 18:06:46  435713
>>435710
Ну я не знаю, что там после протонов пишут, мол есть бозон какой-то, но пока у объекта есть масса покоя, я буду считать, что это вещество.
Как там с массой покоя в черной дупке? Дупки сливаются и излучают гравитационные волны. Значит масса покоя есть. LIGO не даст соврать.
Аноним 01/04/18 Вск 18:11:14  435714
>>435710
Или ты хочешь сказать, что там типа кварки становятся безмассовыми частицами, а масса покоя начинает существовать сама по себе не зависимо от наличия или отсутствия вещества?
Аноним 01/04/18 Вск 18:47:28  435715
>>435713
>>435714
Я сам не спец, но/и у меня всё очень красиво в голове было сложено, конечный личный мой вывод был сложен/произведён мной, который красиво все непонятки со всякими вечностями/бесконечно малыми точками и т.д. объяснял
Типа есть масса, она сжимается, сжимается, переходит из одного в другое, потом наступает такая граница, когда уже не во что переходить, эта граница совпадает с плотностью/гравитацией, при которой свет не может улететь разрушается, превращаясь в чистую энергию, в тот же момент, из-за отсутствия, теперешнего, массы, а как следствие и гравитции, разрушается чёрная дыра, но из-за замедления времени это разрушение/разлетание-вечное

Но потом мне тут сказали что гравитация воздействует не на массу, а на энергию.. и всё рухнуло...
Лол
Аноним 01/04/18 Вск 18:56:32  435716
>>435715
Ну блин. Черняты дупки сливаются за конечное время для стороннего наблюдателя. Значит масса покоя имеет место быть и как объект не останавливается во времени.
Вопрос же я сформулировал. Переформулирую, а то вдруг кто-то не понял. Полагаешь ли ты, что масса покоя становится самостоятельным объектом, доколе вещество уничтожается?
Аноним 01/04/18 Вск 19:20:33  435717
image.png (43Кб, 650x298)
Встречаются два спутанных фотона 1 апреля и один другому говорит: а у тебя спин белый.
Аноним 01/04/18 Вск 19:23:53  435718
>>435716
Я тут только манатеоретизирую, а всё моё образование в этой сфере-научпоп видосики на ютубчике (просто уточню, вдруг ты ещё не понял).

И я тебе не отвечают, а скорее говорю так, к слову
в дополнение к твоим вопросам

>>435716
>Полагаешь ли ты, что масса покоя становится самостоятельным объектом, доколе вещество уничтожается?
Получается что не масса, а искривление пространства остаётся.
Это типа как свет..
Хотя не как свет, тут не понятно. На него, как на энергию, таки действиет гравитация, или ппосто так кажется, потому что двигается он по искривлённому пространству(а а случае когда не может покинуть границы горизонта событий-по бесконечно искривленному. Ну или не бесконечно, а просто в круг/сферу, лол)

Да ещё видел видос, где говорилось что вроде как теоретически может быть создана ЧД чисто из света. Но раз он не имеет массы.. Как произойдёт начальное искревление пространства, для движения света по этому искривлению..
Выходит что таки на энергию воздействеет гравитация, а её воздействия на массу это так, частный случай..


Хм, вот эта мысль про пространство искривленное в сферу мне понравилась, что-то в ней есть.
Стабильно ли она/оно(пространство искривлённое в сферу) или нет, наоборот сразу же разрушается/разворачивается обратно в ровное чуть что..
Или абсолютно стабильно, и продолжает существовать как уже отдельный объект в пространстве, из которого и получился..
Типа как ты писал
>Полагаешь ли ты, что масса покоя становится самостоятельным объектом, доколе вещество уничтожается?


Ну и ещё.. Если чёрные дыры абсолютно стабильны, то как вообще, в принципе, мог произойти большой взрыв, если абсолютная сжатость вещества/материи-абсолютно стабильное состояние?
Ведь вся масса/энергия/пространство что вокруг, не появилось потом, а было изначально, только спрессовано ранее.

Хмм, хотя вот насчёт пространства.. Может та точка из которой и происходил большой взрыв и отличается от обычных чёрных дыр, что в ней, кроме массы и материи, было спрессовано и всё пространство, что есть вокруг, а не как чёрные дыры, которые просто существуют в этом пространстве, а не содержат его внутри/не сжали его внутрь себя, а состоят просто из массы/материи или энергии
Аноним 01/04/18 Вск 19:33:42  435719
>>435716
>Ну блин. Черняты дупки сливаются за конечное время для стороннего наблюдателя. Значит масса покоя имеет место быть и как объект не останавливается во времени.
И да, забыл написать. Хэзэ как, в моём теперяшнем понимании не могут сливаться, как и не могут ничего в себя втянуть. И чёрные дыры должны выглядеть как куча облепленного вокруг себя вещества, бесконечно близко приблежающегося к ней, но не могущего пройти внутрь
Аноним 01/04/18 Вск 19:34:28  435720
>>435718
>Ну и ещё.. Если чёрные дыры абсолютно стабильны, то как вообще, в принципе, мог произойти большой взрыв, если абсолютная сжатость вещества/материи-абсолютно стабильное состояние?
>Ведь вся масса/энергия/пространство что вокруг, не появилось потом, а было изначально, только спрессовано ранее.
>
>Хмм, хотя вот насчёт пространства.. Может та точка из которой и происходил большой взрыв и отличается от обычных чёрных дыр, что в ней, кроме массы и материи, было спрессовано и всё пространство, что есть вокруг, а не как чёрные дыры, которые просто существуют в этом пространстве, а не содержат его внутри/не сжали его внутрь себя, а состоят просто из массы/материи или энергии

Там такой прикол был, что массы покоя не было. Была лучистая энергия, типа электрмагнитной и всяких экзотических. И взрыва не было. Было расширение пространства. Такое же, как и сейчас, при котором якобы из-за тёмной энергии галактики разлетаются друг от друга.
Т.е. в чом прикол. Если взять вещество и антивещество и соединить, то они анигилируют, т.е превратятся в электромагнитное излучение. Так же возможен и обратный эффект - превращение излучения в плотное вещество. Только для обратного эффекта нужно удержать очень большую энергию в маленьком объёме. Вот как раз в первые мгновения так называемого "большого взрыва" который не взрыв как раз были подходящие условия для образования вещества, т.е. дастаточная мощность электромагнитных волн или фотонов в достаточно маленьком объёме.
А почему новорожденное вещество сразу же не сколапсировало в черняту дупку? А потому что пространство расширялось быстрее скорости света и новорожденные частицы тупо не могли догнать друг дружку.
Аноним 01/04/18 Вск 19:39:51  435722
>>435719
Ну, "сливаются" это я высказался утрированно. Они сближаются и вращаются друг вокруг друга. Т.е. мы имеем наблюдать два объекта с массами покоя, которые движутся во времени и пространстве. Т.е. время для массы покоя не остановилось.
Но при этом время будто бы остановилось для вещества, из которого получилась дупка..
От сюда и возникает вопрос: таки шо, масса покоя стала отдельным объектом, не зависящим от первичного вещества, из которого возникла?
Аноним 01/04/18 Вск 19:47:42  435723
>>435719
>И чёрные дыры должны выглядеть как куча облепленного вокруг себя вещества, бесконечно близко приблежающегося к ней, но не могущего пройти внутрь
И мы возвращаемся к исходному вопросу: а куда делось первоначальное вещество, которое прежде было звездой и которое после сжатия образовало черняту дупку?
Оно (хотя бы большая часть этого вещества) ведь изначально пространственно было расположено под горизонтом.
Аноним 01/04/18 Вск 20:06:02  435724
>>435723
Хм, тоже интересно, не задумывался об этом
Ведь сжатие не мнгновенный процесс, оно происходит откуда-то с центра, и, выходит, любая чёрная дыра изначально находилась внутри, в центре окружающего её вещества,
получается, образуется бесконечно малая(в плане массы из которой она получилась, бесконечно малого объёма вещества) чёрная дыра, и потом ничего уже не может в неё большие попасть.
Выходит и чёрных дыр существовать не может, лол
Аноним 01/04/18 Вск 20:07:14  435725
>>435723
А, ты там написал "под"
Или ошибся и хотел написать "над"?
Аноним 01/04/18 Вск 20:57:12  435726
Ладно. Будем исходить из фундаментальных свойств, что ничто не может двигаться быстрее скорости света и что всё, что под горизонтом, притягивается к центру.
Тогда, вещество, породившее черняту дупку и оказавшееся под горизонтом в момент возникновения дупки, может двигаться только в одном направлении - к центру и только с одной скоростью - света. Т.е. получается, что пространство под горизонтом становится:
(а) одномерным
(б) однонаправленным
Что это значит? Это значит, что частицы вещества не могут ни сблизиться, ни отдалиться друг от дружки. Потому что любое сближение или отдаление означало бы превышение скорости света хотя бы одной из частиц или же неподчинение этой частицы гравитационной силе. Т.е. частицы никак не могут друг на дружку воздействовать. И при этом их скорость равна скорости света и они двигуются в одном направлении.
Какие частицы обладают такими свойствами? Внезапно бозоны, в числе которых, например, фотоны.
Т.е. фактически всё вещество превратилось в силовые поля, электромагнитные и прочие, включая гравитационное, если допустить, что бозоны Хиггса таки существуют.
И что мы полуучааем?
Мы таки получаем массу покоя как самостоятельный объект, образованный сгустком полей Хиггса, которые застряли под горизонтом чернятой дупки.
Ахуеть!
Ну вроде бы. А с другой стороны, каждый их этих бозонов, образующих поля, находится под своём персональном горизонтом событий, т.е. не может распространять своё поле в пространстве. Что это значит? А это значит, что образуемая масса покоя не должна никак проявляться ни снаружи, ни внутри от горизонта. Т.е. снаружи чернятой дупы гравитационное поле не должно генерироваться веществом, оказавшимся внутри дупки. Но гравитационное поле снаружи таки есть и обнаруживается внешним наблюдателем..
И тогда мы получаем, что и гравитационное поле и масса покоя чернятой дупки существует не зависимо от породившего его вещества.
Ахуеть!
Аноним 01/04/18 Вск 20:58:05  435727
>>435725
Внутри. Так понятнее?
Аноним 01/04/18 Вск 21:07:32  435728
>>435713
Бозоны - это вообще про другое (тупо кратность спина - атомы бывают бозонами - спин целый, - а бывают фермионами - эти с полуцелым спином - в зависимости от суммы спинов составляющих частей). А насчёт того, что ЧД сливаются и излучают - ЧД гипотетически можно сформировать из фотонов, массы покоя не имеющих. А излучение Хокинга может проявляться любыми частицами (впрочем, когда это ЧД звёздных масс, а не микроскопическая дырочка, то практически всё излучение уходит в виде ультрахолодных ЭМ волн).
Аноним 01/04/18 Вск 21:09:40  435729
>>435728
>ЧД гипотетически можно сформировать из фотонов, массы покоя не имеющих
А не пиздишь ли ты? Как твои фотоны будут порождать гравитацию, если у них нет массы покоя?
Аноним 01/04/18 Вск 21:19:31  435732
image.png (269Кб, 500x436)
>>435718
> Как произойдёт начальное искревление пространства
Под действием собственно огромного количества энергии в ограниченном объёме пространства. Т.е. ещё до образования ЧД безумно огромное количество фотонов может гравитировать, искривлять пространство.
> а её воздействия на массу это так, частный случай
Полагаю, ты под массой имел в виду материю. Так вот, материя и есть энергия, просто в более связанном виде. Можно представить это как стоячую волну.
>как вообще, в принципе, мог произойти большой взрыв
Есть шанс, что мы на самом деле в ЧД.
Ну, точнее, мы ФАКТИЧЕСКИ внутри ЧД, возможно, даже бесконечной, - потому что с ростом массы ЧД её плотность уменьшается. И уже даже если взять массу видимой вселенной, её радиус Шварцшильда больше радиуса видимой вселенной (3.4×10^54 кг масса, радиус Шварцшильда - 5.05×10^24 км, радиус видимой вселенной - 4.4×10^23 км).
Физика
Аноним 01/04/18 Вск 21:20:50  435733
>>435718
>Может та точка из которой и происходил большой взрыв
за такое в физике бьют в морду. БВ не происходил в точке. Мы и есть та самая точка и никуда из неё не девались. БВ происходил везде - само пространство расширялось. Не откуда-то, а везде, в каждой точке появлялись новые точки, в которых ещё больше точек и т.д.
Аноним 01/04/18 Вск 21:23:25  435734
>>435732
>Есть шанс, что мы на самом деле в ЧД.
>Ну, точнее, мы ФАКТИЧЕСКИ внутри ЧД

Никак нет. Мы точно не в ЧД. В ЧД пространство искажено на столько, что является одномерным.
Я же очень чотко наблюдаю трёхмерность пространства. Значит я не в ЧД. Так же чотко твой высер. Значит ты тоже не в ЧД.
Аноним 01/04/18 Вск 21:26:45  435735
>>435726
>может двигаться только в одном направлении - к центру и только с одной скоростью - света
А вот тут ты накололся. Если бы что-то за пределами горизонта событий могло создать тягу, то оно могло бы двигаться и навстречу другим частицам (чуть-чуть медленней падая в центр). И, с другой стороны, частицы, одновременно прошедшие горизонт, будут двигаться навстречу друг другу - ведь обе они стремятся в центр, но вектор направления на центр у двух частиц будет разный и лучи, направленные от горизонта к центру, пересекаются собственно в центре.
Аноним 01/04/18 Вск 21:27:49  435736
>>435734
мы блять вообще можем быть на поверхности двумерного пространства, а третье измерение просто на голографическом принципе выводится. Так что попизди мне тут.
Аноним 01/04/18 Вск 21:28:17  435737
>>435734
И вообще, позитивизм в науке не уважается. Так что "я вижу" - не аргумент вот совсем.
Аноним 01/04/18 Вск 21:30:49  435738
>>435735
>могло бы двигаться и навстречу другим частицам (чуть-чуть медленней падая в центр)
Ты забыл про ускорение падения. Из-за ускорения каждая частица, как бы она ни старалась, всё равно не могла бы ни на йоту приблизиться к другой частице.
Аноним 01/04/18 Вск 21:33:04  435739
>>435736
Если бы мы жили в двухмерном пространстве, то мы не могли бы двигаться в трёх измерениях. И никакая голография нам не помогла бы. Но мы можем. Значит третье измерение всамделишное и не голографичное.
Аноним 01/04/18 Вск 21:36:26  435740
>>435738
ускорение - это производная от скорости, а не сама скорость. Так что вполне могла бы.
Но вообще рассуждать об этом как-то наивно, т.к. там даже геометрия ломается. В PBS SpaceTime было о том, что там время и пространство меняются местами.
>>435739
Ебать ты у мамки позитивист. Катись в свой 18 век и изобретай там свою логику исключительно на феноменологическом базисе.
Аноним 01/04/18 Вск 21:41:10  435742
>>435732
>Есть шанс, что мы на самом деле в ЧД.
>Ну, точнее, мы ФАКТИЧЕСКИ внутри ЧД, возможно, даже бесконечной, - потому что с ростом массы ЧД её плотность уменьшается. И уже даже если взять массу видимой вселенной, её радиус Шварцшильда больше радиуса видимой вселенной (3.4×10^54 кг масса, радиус Шварцшильда - 5.05×10^24 км, радиус видимой вселенной - 4.4×10^23 км).
Это работает вроде только если вся масса, образующая горизонт событий, сжата в ЧД
Т.е. вся масса видимой вселенной сжата в чд-её горизонт событий больше наблюдаемой вселенной.
Вся масса распределена по всему, или почти всему объёму-размер горизонта событий не больше размера наблюдаемой вселенной. Его вообще нет.

Ты пишешь про якобы малуую плотность больших чёрных дыр, но представляешь чёрную дыру как сплошной цельный объект, а это не так, чёрная дыра это условно точка, в которой вся масса, а вокруг неё пузырь, за стенку которого не может выйти свет. Плотность среднюю ты считаешь, а надо про плотность отдельных состовляющих этого объекта не забывать.
А то у тебя получается не мыльный пузырь, с воздухом внутри и очень тяжёлой пылинкой в центре, а сплошной шарик воды.
Аноним 01/04/18 Вск 21:44:59  435743
>>435740
>время и пространство меняются местами.
Про пространство то понятно. Оно становится одномерным и однонаправленным. Это прямо вытекает из основного постулата о невозможности превысить скорость света. И именно это подразумевалось, когда сказали, что меняется местами.
Однако я не понял про время? Какие свойства оно приобретает? Высказывалось мнение, что в чернятой дупке время становится двунаправленным. Но, это просто спизданули, не приведя никакой аргументации. Посему возникает закономерный вопрос: схуяли?!
Аноним 01/04/18 Вск 21:48:42  435744
>>435742
>не мыльный пузырь, с воздухом внутри и очень тяжёлой пылинкой в центре, а сплошной шарик воды.
А чому бы и нет. Схуяли масса должна вжаться в точку, если время остановилось? Не должна она вжаться. Массе некогда вжиматься, у неё нет на это времени.
Аноним 01/04/18 Вск 21:49:47  435745
>>435733
>БВ не происходил в точке. Мы и есть та самая точка и никуда из неё не девались.
Ну и
Раньше были маленькое точкой, теперь вроде побольше
Как находиться внутри взрыва, учитывая что вокруг взрыва ничего нет
Что тебе не понравилось, не могу понять
Аноним 01/04/18 Вск 21:50:00  435746
>>435742
Ну, кстати, да, бесконечная ЧД смысла не имеет... А вот если вселенная всё-таки не бесконечна, тогда она должна образовать горизонт событий.
С чего бы это ЧД - точка? ЧД характеризуется в первую очередь сферой Шварцшильда. Что там внутри - толком неизвестно. Но сжатие в точку явно противоречит уже принципу Паули, и очень сильно не факт, что гравитация этот принцип пересиливает (уже потому не факт, что ОТО и квантмех на эту тему не сходятся совсем).
Аноним 01/04/18 Вск 21:51:57  435747
>>435744
>Массе некогда вжиматься, у неё нет на это времени.
А планетки чё ж крутятся.. А сообщение ты это как настрочил, если время остановилось..
Аноним 01/04/18 Вск 21:54:05  435748
>>435747
Ну дык, я ж не в чернятой дупе. Я в трёхмерном пространстве. А кто не согласен с этой точкой зрения - пусть пруфает или катится к хуям.
Аноним 01/04/18 Вск 21:54:14  435749
И, кстати, на тему того, почему после БВ всё не сжалось обратно - на самом деле для формирования ЧД важно не столько количество энергии, сколько градиент её плотности. А поскольку вещество в начале БВ было распределено довольно равномерно, то таких градиентов, теоретически, не образовывалось.
Аноним 01/04/18 Вск 21:57:53  435750
>>435746
>С чего бы это ЧД - точка?
Чд я тут называл сингулярность в центре. Потому что это слово, как и то что оно означает, как и точка, мне не нравится, т.к. непонятно.
>С чего бы это ЧД - точка?
И не точка, а.. хз что, пусть будет шар конечного размера, не нулевого.
Ну или нулевого
Аноним 01/04/18 Вск 21:58:39  435751
>>435749
Ну, это ты как будто подразумеваешь, что вещество как будто имело бесконечный объём.
Да вот только одна проблема - не было бы градиентов, то не образовались бы и сгустки материи, и не было бы галактик. А они есть. Понимаш? Значит и градиенты были.
Значит объяснение несжатия другое. Например, дело может быть в очень быстром (быстрее света) расширении пространства на начальном этапе формирования вселенной.
Аноним 01/04/18 Вск 22:04:11  435755
>>435751
Градиенты были, разумеется, но сильно недостаточные для образования ЧД.
Аноним 01/04/18 Вск 22:04:37  435756
>>435748
А, я думал отвечал тот кто говорит что вселенная внутри чд. Я ему отвечал.

А в таком случае я выше писал, если учитывать замедление и остановку времени-образование чёрной дыры невозможно.

Хотяя, бле, я забыл и запутался, вроде для того что падает в чёрную дыру время идёт нормально, т.е. оно может в неё упасть.
Но сторонний наблюдатель не может увидеть это падение, т.к. оно будет происходить бесконечно долго.
Т.е. чёрные дыры могут существовать, Но и одновременно при этом не могут существовать для стороннего наблюдателя.

Огромныев смысле массы коты Шрёдингера, блять
Аноним 01/04/18 Вск 22:07:08  435757
>>435755
Тогда из твоих аргументов следует, вселенная изначально была бесконечная и изначально была на всей бесконечной протяжённости более-менее заполнена веществом. Ну... интересная точка зрения.
Аноним 01/04/18 Вск 22:09:34  435758
>>435749
Чёт бред какой-то. То что плотность чд не равномерна это не причина, а следствие. Т.е. то что у неё внешний размер определяется каким-то там горизонтом-пузырём это лишь следствие того что там где-то внутри как-то огромная плотная масса.
Вообще то что у неё есть этот внешний пузырь это следствие.
Или я не так понял твоё это "градиент плотности"?
Аноним 01/04/18 Вск 22:09:46  435759
>>435757
Не факт. Я ж говорю - неизвестно что происходит за горизонтом, поэтому нельзя утверждать, что мы никак не можем находится внутри ЧД и тем более нельзя на этом утверждении делать вывод о бесконечности вселенной.
Аноним 01/04/18 Вск 22:11:56  435761
>>435758
Это не ты ли тот самый позитивист? Подписывайся, чтоль, чтобы я сразу игнорил, а то какой смысл объяснять что-то чуваку, который верит только в то, что видит.
Аноним 01/04/18 Вск 22:12:20  435762
>>435756
>для того что падает в чёрную дыру время идёт нормально, т.е. оно может в неё упасть
А вот я чото сомневаюсь. Время на горизонте ВСЁ, кончается. Там, прямо на поверхности сферы Шварцильда ничего не двигается от слова совсем. Как мужик, упавший с ракеты, прорвётся сквозь этот барьер нулевого времени? Тут я даже сомневаюсь, что хотя бы тунельный эффект сработает.
Аноним 01/04/18 Вск 22:17:02  435763
>>435759
>неизвестно что происходит за горизонтом
Ну, у нас есть решения уравнений СТО для ЧД.
Это основной аргумент. А то без уравнений мы бы так же могли утверждать что-то наподобие "неизвестно что происходит на орбите МКС", ну и вообще вся беседа превратилась бы в срач аки "Вархаммер VS Массэффект".
Аноним 01/04/18 Вск 22:23:42  435766
>>435763
Уравнения ОТО может быть? В любом случае, они приводят к сингулярности. А в физике сингулярность не любят. Да, мы можем рассуждать (и уже нарассуждались вдоль и поперёк) на базе ОТО, но тем не менее, пользы от этого не особо, собственно, подход к ЧД на базе ОТО вообще устарел, т.к. там ничего нового не найти уже. То же излучение Хокинга как раз не на ОТО основано, а в большей мере на квантмехе. Но квантмех не может в то, что происходит за горизонтом событий или, тем более, в центре. Так что мы так и так вынуждены устраивать срач, потому что никто не знает. И самые образованные знают почему никто не знает.
Аноним 01/04/18 Вск 22:26:14  435767
>>435766
Ну блин. С геометрией пространства там всё ясно.
Вот кто мне за время тамошнее пояснит. Как оно вообще может работать в двунаправленности. Там блять чо, нет макрообектов штоли? Одни кванты, для которых похуй куда время текёт?
Аноним 01/04/18 Вск 22:34:39  435768
>>435755
На самом деле тут всё не так просто. И стоит вопрос о Большом Сжатии в будущем. То есть всё может быть устроено так, что расширение сменяется обратно сжатием и вопрос в том какое время процесс продолжается. И в основном ответ получается вроде бы только из наблюдений астрологов >:-)
А из этих наблюдений на текущий момент вроде бы видно, что сжатия быть не должно. Но это уже без объяснений почему...
Аноним 01/04/18 Вск 22:41:00  435769
>>435768
Из наблюдения астрологов становится видно, что Солнце в Овне. А из наблюдения астрономии - что расширение ускоряется. Вроде как вторая производная неизвестна, поэтому если она отрицательна (ускорение уменьшается), тогда когда-нибудь очень очень не скоро ускорение сменится-таки замедлением. Но тем не менее, нынешних наблюдений сейчас явно недостаточно. А точнее, из нынешних наблюдений вообще следует, что более вероятен большой разрыв, когда тёмная энергия станет превалирующей над всеми остальными, включая сильное взаимодействие.
Аноним 01/04/18 Вск 23:30:23  435772
>>435769
>Вроде как вторая производная неизвестна
Ну вообще-то ускорение это и есть вторая...
Ну там их ещё много, да... >:-)
>нынешних наблюдений сейчас явно недостаточно
Тут теория нужна. Которой похоже нет на этот случай... Кроме рассуждений про тёмные энергии...
Из наблюдений астрологов становится видно, что ждёт в будущем...
Аноним 01/04/18 Вск 23:35:25  435773
>>435772
А из астрономии - действительно, что Солнце в Овне :-)
Аноним 01/04/18 Вск 23:49:18  435776
>>435772
Ты наркоман или просто любитель повыёбываться? Ускорение - первая производная для скорости. Ускорение ускорения - вторая. Или же первая для ускорения.
Аноним 02/04/18 Пнд 00:10:46  435777
>>435776
Боюсь что ускорение ускорения будет всё же второй ускорения... >:-)
Аноним 02/04/18 Пнд 00:17:00  435778
>>435777
Не тупи и пиздуй учить алгебру.
Аноним 02/04/18 Пнд 00:27:57  435779
В каком виде информация хранится в генах?
Аноним 02/04/18 Пнд 00:56:43  435780
>>435779
В виде ДНК. А если ты хотел сказать в каком виде она хранится в ДНК - в виде оснований, четырёх подобных молекул. Они читаются белками транскрипторами (РНК-полимеразами), вроде бы один к одному, т.е. каждой молекуле ДНК соответствует молекула РНК, из них получается матричная РНК. Эта мРНК затем читается другими белками - рибосомами - в виде три к одному, т.е. из сочетания трёх различных молекул РНК получается одна аминокислота. А уже из цепочки аминокислот получаются белки. Многие РНК не являются матричными и из них белков не синтезируются, такие молекулы либо разбираются, как мусор, либо являются элементами сигнальной системы для регуляции синтеза белков или чтения ДНК.
Если тебя что-то другое интересовало, то уточняй.
Аноним 02/04/18 Пнд 02:44:01  435783
>>435780
>>435780
В каком именно виде информация в них хранится и как обрабатывается? В каком виде она переходит от ДНК к белку? Как молекулы хранят в себе информацию о безусловных рефлексах?
Аноним 02/04/18 Пнд 03:12:18  435784
>>435783
В каком виде хранится, как обрабатываются и как переходят к белку я уже рассказал. Или тебя интересуют прям малейшие детали? Это в книгах уже и статьях.
Как хранятся безусловные рефлексы, боюсь, это уже охренительно сложный вопрос. И ответ только на него, даже упрощённый, занял бы целую книгу, да и многие вещи были бы только гипотетическими, т.к. клеточные механизмы мы только начали понимать.
Если совсем поверхностно, то, разумеется, там не хранится некая компьютерная логика "если то, то это". Вместо неё хранятся тысячи и тысячи регуляторных последовательностей (они по большей части даже не белки и не РНК, а места, к которым крепятся белки и блокируют чтение генома в ближайшем окружении, либо наоборот, места, которые способствуют тому, чтобы этот участок более вероятно прочитался и как следствие чтобы он читался чаще других участков генома), которые регулируют работу белков под строго заданные условия. И через такую регуляцию каждая клетка в развивающемся организме находит своё место. Нейроны, начавшие своё развитие в одном месте, перекочёвывают в другое (для этого нужен градиент какого-нибудь сигнального вещества или даже нескольких, когда с одной стороны его мало, с другой много, и на основе этого градиента клетка решает, что ей надо сдвинутся в ту или иную сторону), по пути оставляя аксон или дентриды и сохраняя связь с предыдущим местом. Или же, например, другое сигнальное вещество, на которое клетка реагирует тем, что сокращает аксон или дендриты в его присутствии. И таким образом формируется общая архитектура нейросети. И затем, из-за особенностей работы нейронов, нейромедиаторов, синапсов, мембран, рецепторов и ещё кучи других вещей, сигнал от раздражителя в рамках этой архитектуры практически у всех особей приводит к определённой реакции - тому самому рефлексу.
Аноним 02/04/18 Пнд 04:08:39  435785
>>435783
Ну и, кстати, да, ещё немного - поскольку геном явно не божественного творения (если только эта сучьность не любитель сотворить какое-нибудь извращённое, кривое, нестабильное и запутанное дерьмо), то эволюцией в нём большая часть не играет лишь одну роль - всё, что может пригодится или оказаться полезным в другом месте - используется и в другом месте тоже. Так что в принципе не было бы ничего удивительного обнаружить ген, который бы, например, одновременно задавал форму пятки, производил зелёный пигмент и участвовал в рефлексе чихания, просто потому что так уж исторически сложилось, что именно этот ген в силу мелких мутаций оказался полезен для этих функций. А поскольку генов - десятки тысяч, а помимо них ещё сотни тысяч регуляторных последовательностей, то в каждой новой особи вся эта мешанина волею случайной мутации может проявить какую-нибудь ещё новую функцию.
Так что если рассматривать ДНК как язык программирования, то геном выглядел бы как самый извращённый, тупой и бессмысленный кусок говнокода, который не под силу было бы создать ни одному разумному существу; и там можно было бы наблюдать бесчисленные goto, спагетти код, полное остутствие масштабирования и костыль на костыле, придерживающий тадж-махал, отстроенный из костылей и велосипедов.
Аноним 02/04/18 Пнд 06:35:53  435787
>>435665
Если у тебя в коробке левый носок, то тем фактом что ты знаешь, что где-то далеко в коробке лежит правый носок, информацию ты не передашь. Квантовые носки в коробках состояния не меняют, как бы ты свои коробки не открывал, то у тебя всегда получится рандомная смесь из правых и левых. При этом ты каждый раз будешь знать что где-то там есть коробки с противоположными носками.
Аноним 02/04/18 Пнд 10:12:47  435797
id07[1].gif (7Кб, 228x171)
0e488d42188c[1].jpg (41Кб, 500x500)
>>435409 (OP)
Какая форма магнитного поля будет, если один из магнитов на первой картинке
развернуть сверху вниз и сцепить их всеми противоположными полюсами?
(Первый север одного к югу другого, другой юг другого - к другому северу первого.)
Будет что-то вроде второй картинки, или как?
Аноним 02/04/18 Пнд 11:34:33  435802
Стикер (0Кб, 512x512)
>>435758
>плотная масса
Аноним 02/04/18 Пнд 11:36:32  435803
>>435797
>развернуть сверху вниз и сцепить их всеми противоположными полюсами?
яннп. Нарисуй и проведи магнитные линии, сразу станет понятней.
Аноним 02/04/18 Пнд 12:38:18  435805
электронные генераторы вместе с фундаментальной частотой, f, производят гармонические частоты, 2f, ..., nf.
почему n, целое?

Аноним 02/04/18 Пнд 12:39:22  435806
>>435805
Потому что иначе это была бы не гармоника.
Аноним 02/04/18 Пнд 12:40:51  435807
>>435806
вопрос иначе: почему егнератор не производит НЕ гармоники?
Аноним 02/04/18 Пнд 12:46:31  435808
>>435807
Ну потому что у него вообще есть фундаментальная частота, напрямую связанная с частотой вращения, и куча разных процессов, погрешностей, неточностей и прочего, само собой связанных с тем, как ротор совершает оборот вокруг статора. Откуда в полностью циклическом процессе взяться частоте, которая с частотой оборотов не связана?
Аноним 02/04/18 Пнд 12:59:03  435811
>>435808
почему не гармоника не может быть связана с фундаментальной частотой?
к примеру, если f - фундаментальная, то частота f/3, значит, что вал задевает что-то 1 раз за 3 оборота.
Аноним 02/04/18 Пнд 13:03:28  435813
>>435811
А, бля, ты это имел в виду... Да, тогда странно. Так-то f/3 - тоже гармоника...
А, погоди, я догадываюсь. У тебя ниже f гармоник не может быть, потому что f - это не оборот вала, а оборот вала, домноженный на количество обмоток статора.
Аноним 02/04/18 Пнд 13:27:08  435814
>>435813
>>435811
я ошибся, видны и 3f и f/3 и f/n, n - целое.
я имел ввиду, к примеру 3/2f, но это тоже самое, что (1 + 1/2)f и такие гармоники есть.
2/3f = (1/3 + 1/3)f. и т.д.
Аноним 02/04/18 Пнд 13:30:00  435815
>>435814
т.е. спасибо. вроде бы понял.
это исходит из разложения Фурье. как-то. я плохо знаком с Фурье.
Аноним 02/04/18 Пнд 13:51:37  435817
magnetic[1].png (19Кб, 511x374)
magnetic.png (24Кб, 436x510)
>>435803
Нашёл в гугле первую картинку и сделал из неё вторую.
Надо посмотреть магнитной бумагой какой-нибудь.
Кажется, что силовые линии с южного полюса одного магнита
просто замкнуться на северный полюс другого магнита.
А по бокам ничего не будет, или будет очень слабое магнитное поле.
С другой стороны, вторая картинка предыдущего поста >>435797 показывает четверное поле.

Интересно будет ли магнитное поле у Земли если Марс упадёт на Землю?
https://2ch.hk/spc/res/370601.html#392774

И если будет, то какой формы оно будет?
Внутри Земли есть твёрдое ядро с остаточной намагниченностью.
У Марса оно тоже твёрдое, и если они притянутся внутри Земли,
тогда два магнитика могут склеится вот так,
и хрен поймёшь какое магнитное поле будет тогда.

Аноним 02/04/18 Пнд 14:21:19  435818
>>435817
У тебя будет то же, как если бы это был один магнит.
Аноним 02/04/18 Пнд 14:26:27  435819
slide17[1].jpg (38Кб, 800x600)
405280d545877c7[...].jpg (20Кб, 354x345)
D09FD0BED0BBD0B[...].png (46Кб, 500x355)
>>435818
Ну, если их соединить так, как на картинке 1 и 2, то да - будет один магнит.
Магнитное поле - на второй картинке. Но я же не так соединяю их.
И мне кажется, что силовые линии с северного полюса - просто уйдут в южный, и не будет магнитить.
Аноним 02/04/18 Пнд 14:30:34  435820
>>435819
Тебе кажется.
Аноним 02/04/18 Пнд 15:30:33  435821
201712041619271[...].jpg (21Кб, 680x485)
137412546714[1].gif (24Кб, 478x357)
magnitoplast-d4[...].jpg (58Кб, 500x416)
MPo-7[2].jpg (31Кб, 440x204)
>>435820
Смотри, если мысленно изогнуть стержневой магнит - до подковообразного, прямоугольного (картинка 1)
или до полукруглого (2 картинка) и сцепить их противоположными полюсами,
то силовые линии, будучи параллельными изгибу, из юга одного - быстрее зайдут в север другого и не разомкнуться во внешнее пространство.
Тогда, магнитного поля во вне - вообще быть не должно, и система поэтому - не должна проявлять свойства магнита.

На 3-й картинке видно конфигурацию магнитного поля у какого-то кольцевого магнита,
и как я понял из этого видео: https://www.youtube.com/watch?v=pUkozb1zRxc
на с 1:14 до 2:00,
>темные места соответствуют тем местам магнита, откуда магнитное поле выходит ПЕРПЕНДИКУЛЯРНО плёночке,
белые же полосы говорит о том, что в этом месте силовые линии ПАРАЛЛЕЛЬНЫ плёночке.
То есть, этот магнит, что на 3-й картинке, подобен магниту на 4-й картинке (там полюса по-другому),
но магнитам не таким, то есть разбитым не на 2, а на 4 части.

Но меня же интересует конфигурация магнитного поля у сборки, с полюсами как на картинке 2,
причём у сборки из полукруглых, подковообразных, квадратных, или стержневых магнитов,
а также у двух таких магнитов - но шарообразных, как два ядра у планет.

Если у кого-то есть плёночка такая, и магниты всякие - то можете нафоткать и напостить тут.
Аноним 02/04/18 Пнд 16:10:58  435823
>>435821
Магнит всегда будет проявлять магнитные свойства, как ты его ни сконфигурируй. Заставить его перестать это делать можно двумя способами - либо измельчив буквально до атомарного уровня, полностью уничтожив всю кристаллическую решётку, либо разогрев выше критической температуры.
Часть линий магнитного поля, конечно, замкнётся, но магнитные линии расходятся во все стороны, так что ничего принципиально не изменится как ты его ни замыкай.
Аноним 02/04/18 Пнд 16:31:18  435825
image031[1].jpg (16Кб, 500x274)
9d10981285b9545[...].png (13Кб, 673x522)
stock-photo-red[...].jpg (61Кб, 450x320)
>>435823
Ну, я думаю, магнитное поле подковообразного магнита ты уже видел?
Аноним 02/04/18 Пнд 16:52:15  435826
Метафизик кукарекнул со стороны параши:
>>435740
>Ебать ты у мамки позитивист.
Аноним 02/04/18 Пнд 17:50:51  435827
image.png (48Кб, 800x600)
>>435735
>Если бы что-то за пределами горизонта событий могло создать тягу, то оно могло бы двигаться и навстречу другим частицам (чуть-чуть медленней падая в центр).
Если бы да кабы. В случае с чёрнятой дупкой нечему создавать внешнюю тягу. В момент образования дупки вся-пре-вся масса исходной звезды оказывается под горизонтом. Можно конечно рассмотреть вариант слияния двух дупок. Но в таком случае просто произошло бы смещение центра тяжести и всё вещество, до момента слияния падавшее в одном направлении, вдруг стало бы падать в другом, но от этого частицы не стали бы двигаться на встречу друг другу. Они всего лишь все вместе изменили бы направление своего падения.

>И, с другой стороны, частицы, одновременно прошедшие горизонт, будут двигаться навстречу друг другу - ведь обе они стремятся в центр, но вектор направления на центр у двух частиц будет разный и лучи, направленные от горизонта к центру, пересекаются собственно в центре.
Так то оно как буд-то бы так. Но ты забыл про сингулярность. Частицы двигаются не друг к другу. Частицы двигаются в направлении сингулярности. Под горизонтом вектор гравитационного ускорения каждой частицы направлен строго в сторону сингулярности, ни на йоту не отклоняясь в другую сторону. Прям совсем не отклоняясь.
Выходит, что частицы под горизонтом могут на начальных участках своих траекторий несколько сблизиться, достаточно чтобы провзаимодействовать с друг дружкой. Но только на короткое время. Под горизонтом частица не может отклонить свой вектор ускорения. А это значит, что в конечном итоге траектории падения двух частиц, какими бы близкими они ни были, всё же будут стремиться стать параллельными. И чем дальше, тем сильнее это стремление, так что в конечном итоге на какой-то глубине под горизонтом возникает условная граница, после прохождения которой у частиц уже не хватает никаких сил взаимодействия, ни сильных, ни слабых, ни электрических, чтобы сближаться друг с дружкой. После прохождения этой условной границы каждая частица становится полностью изолированной от любых других частиц.
Аноним 02/04/18 Пнд 18:17:21  435828
>>435827
Ну и естественный вывод из этого:
После прохождения условной границы понятие пространства перестаёт иметь смысл, потому что т.к. частицы никак между собой не взаимодействует, то отменяется принцип относительности движения, т.е. движение становится равнозначным стоянию на месте. По сути перестаёт существовать даже одномерное пространство. Т.е. математически оно может быть всё ещё существует, а физически его уже нет. А вместе с пространством перестаёт существовать и время, т.к. это две неразрывно связанные сущности.
Аноним 02/04/18 Пнд 18:22:07  435829
>>435828
ПыСЫ: Я сам того не заметил, как описал в обратной последовательности начальные стадии большого взрыва. Но всё это может быть верным только если под горизонтом чернятой дупки таки есть время. В чом я не уверен.
И ещё остаётся нерешённым вопрос: могут ли нейтроны взаимно анигилировать при достижении достаточной плотности?
Аноним 02/04/18 Пнд 18:39:59  435832
>>435829
Так ладно. Отвечу сам себе. Нейтрон должен был бы самопроизвольно распадаться на протон, электрон и хуиту с периодом полураспада 15 минут после прохождения условной границы. Т.е. под условной границей начинается область распада протонов на взаимно изолированные частицы, которые сразу же в момент распада теряют связь с друг дружкой. Т.е. время там ещё не остановилось. Но при этом каждая частица существует в своём времени, никак не связанном с временем других частиц. Т.е. частицы существуют сами по себе, как волны.
Аноним 02/04/18 Пнд 19:02:56  435833
Молодец, теперь создай себе уютный тредик и общайся там сам с собой сколько влезет.
Аноним 02/04/18 Пнд 20:47:22  435836
Поясните за комбинированы степени преувеличения. или как они называются? Типа есть супер, мега, гига и так далее. Эти слова достаточно научны т.к. легко можно вспомнить их применения в науке. Но вот чего я не могу вспомнить так это их комбинирования. Например супермега используют в обычной речи, но не используют в науке. мегагига вообще не используют скорее всего из-за того, что гига не очень популярное слово. Почему в науке отказались от использования комбинированных степеней преувеличения типа супермега?
Аноним 02/04/18 Пнд 20:54:31  435837
>>435823
>либо измельчив буквально до атомарного уровня

Вот это как раз немножко безнадёжно...
Аноним 02/04/18 Пнд 20:57:37  435838
>>435778
Ваше мнение было принято к рассмотрению и должным образом обработано :-)
Аноним 02/04/18 Пнд 21:25:38  435840
>>435787
Т.е. от воздействия он принимает рандомное состояние, и подействлвав на него определённым образом нельзя заставить его определённое состояние?
(Которое мгновенно но наоборот передастся его паре)
Аноним 02/04/18 Пнд 21:27:18  435841
>>435802
Всё ж норм, звучит немного непривычно, но никаких противоречий нет
Аноним 02/04/18 Пнд 22:02:42  435844
>>435837
Вот именно.
Аноним 02/04/18 Пнд 22:04:47  435845
>>435840
Разумеется нельзя. Неужели ты думаешь, что за годы с момента открытия этого эффекта никто не пытался использовать его?
Аноним 03/04/18 Втр 00:54:09  435849
>>435845
>Неужели ты думаешь
В том сообщении про носки есть приписка что "судя по тому как это описывает тут анон, нет никаких препятствий передавать так информацию"
Аноним 03/04/18 Втр 01:01:22  435850
>>435785
>>435784
А как это всё работает? Почему система и взаимодействия веществ и реакций определяются именно так? Что нужно знать, чтобы понимать принцип и осуществления чтения? Или это просто запущенная череда взаимодействий химических реакций, обусловленных веществами?
Аноним 03/04/18 Втр 01:32:04  435851
>>435849
Ты про то, чтобы надевать носок именно на правую ногу? Справедливо, тут я накололся... Надо будет в следующий раз в этой аналогии уточнить, что выбрать ногу нельзя.
>>435850
По большей части потому что так исторически сложилось. У нас и в геноме, и в протеоме (это все белки в клетке) плавает куча мусора (предположительно мусора; время от времени мы узнаем, что что-то из этого, оказывается, играет свою роль, но пока что судя по темпам, с которыми мы находим среди мусора что-то полезное, там всё-таки большая часть - это именно мусор). И из-за случайных мутаций что-нибудь из этого мусора может стать немного полезным, а затем будет подхвачено эволюционными процессами и рафинировано до наибольшей эффективности. У природы были буквально миллиарды лет, чтобы сформировать тысячи разных механизмов рецепции всевозможных стимулов: фоторецепторы, хеморецепторы, терморецепторы, etc, etc. По большей части можно относительно просто продублировать существующий рецептор и модифицировать одну из копий под новые нужды. Ну а поскольку это процесс весьма случайный, то обычно нет какого-то серьёзного обоснования почему была выбрана именно та или иная сигнальная молекула.
К слову о том, как работают рецепторы - большинство из них, простите за вульгарность, работают на ощупь. Т.е. этот самый линейный белок, когда сворачивается в правильную форму, образует внутри себя полость определённой формы. Плюс к тому, там ещё болтаются всякие разные аминокислотные остатки - то, чем одна аминокислота отличается от другой. И эти остатки по-разному заряжены, так что в этой полости ещё и образуется определённый узор положительных и отрицательных зарядов. И когда к этому белку подплывает какое-нибудь вещество, если это левое вещество, то оно тупо не подходит - у него не та форма и не тот рисунок полярности. А сигнальное вещество - оно именно той формы и полярности, чтобы войти в белок, как ключ в замочную скважину. И когда оно входит в белок, оно приводит к тому, что этот самый рецепторный белок перестраивается. И там уже либо он выпускает из себя какую-нибудь ещё сигнальную молекулу, либо с ним связан ещё какой-нибудь белок, который из-за этой перестройки освобождается и начинает свободно плавать, или наоборот какой-нибудь белок может присоединиться к рецептору только когда с рецептором соединена сигнальная молекула. В общем, тут много разных способов и скорее всего мы ещё далеко не все их знаем. В любом случае, происходит нечто. Это нечто может в результате каскада разных реакций приводить к ответной реакции на то, что почуял этот рецептор - или изменить мембранный потенциал, как это делают нейроны, или начать выработку какого-нибудь белка, или сдвинуть цитоплазму клетки в какую-нибудь сторону. Опять же, возможных реакций очень много. И вся эта мешанина миллионами лет оттачивалась так, чтобы работать. Тут нет какой-то общей системы, нет единообразия, кроме законов химии. Всё работает потому, что... просто так сложилось.
Что нужно знать, чтобы осуществлять чтение чего? Генома? Ну.. более-менее всё, т.к. там всё завязано друг на друга. Белки читают геном, а геном создаёт белки. Так что нельзя просто так взять ядро из одного организма и вставить его в клетку другого, надеясь получить первый - там не тот протеом, не тот химический состав и много чего отличается. У прокариотов (бактерий) с эукариотами вообще принцип работы с ДНК различный - прокариоты ДНК держат в виде множества колец и постоянно копируют эти кольца, не парясь особо ни об их количестве, ни о том, что на них находится. Эукариоты хранят ДНК огромными длиннющими линейными клубками (хромосомы; не лучшее, кстати, решение, т.к. из-за этого при копировании начало и конец ДНК тупо не помещаются в реплисоме и потому не копируются; чтобы это пофиксить, эукариоты придумали теломеры, не несущие полезной информации, а предназначенные именно для того, чтобы их не жалко было при каждом копировании отрезать по чуть-чуть). К тому же, эукариоты по большей части стремятся всё держать в двух экземплярах, от обоих родителей по хромосоме. Зато эукариоты могут в охренительно точную настройку того, сколько и в каких условиях должно производится того или иного белка - собственно благодаря этой фиче мы, многоклеточные и сложные организмы, и существуем.
Аноним 03/04/18 Втр 02:54:13  435853
>>435840
Блджад, пользуйся нормальной терминологией. Нет никакого нахуй воздействия, есть измерение. Какое нахуй "задает" при измерении? Я меряю длину своего хуя и задаю ему величину 20см, хотя на самом деле там 3мм? Если ты как-то там будешь воздействовать на один носок, то другому будет глубоко похуй, на нем эти воздействия никогда не отобразятся.
Аноним 03/04/18 Втр 06:53:26  435855
1332193246601.jpg (1242Кб, 1600x900)
Какая максимальная температура была в эпицентре падения динозаврьего метеорита? И правда ли, что в центре бабаха царьбомбы было 10^10 грабусов?
Аноним 03/04/18 Втр 14:58:12  435889
Знатоки, внимание ВОПРОС!
Допустим у меня дома есть беговая дорожка. В день я пробегаю 10км со скоростью 10 км/ч. Возможно ли как-то засэйвить эту энергию? Не знаю как динамо машина или в аккум каокой и о каких цифрах идёт речь?
Аноним 03/04/18 Втр 15:02:48  435890
>>435889
Только если пересядешь на велосипедный тренажёр. Да и энергии-то там - не хватит даже вскипятить кружку. Большая часть уходит в тепло.
Аноним 03/04/18 Втр 15:57:27  435900
>>435889
самый простой способ - аккумулировать в сверхпроводнике, конечно же.
на самом деле - прикручиваешь генератор - к нему кипятильник - греешь воду - сливаешь в термос.
Коди Аноним 03/04/18 Втр 18:04:04  435902
Аноны, объясните что за хрень на этом видео?
https://www.youtube.com/watch?v=gHoUCyQR7h8
Видео-то первоапрельское, но он потом действительно прополоскал рот, тем что намешал, иначе бы такое на ютубе не пропустили, если бы действительно было опасно.
Вода + калий + метанол + перекись водорода + немного плавиковой кислоты - что в итоге получилось?
Аноним 03/04/18 Втр 18:20:27  435903
>>435902
Если смешать в правильной пропорции, то ничего страшного не будет, главное не глотать, остальное взаимно нейтрализуется: калий + вода = гидроксид калия + водород (улетучивается сразу). Гидроксид калия + плавиковая кислота - фторид калия (вреден, но его надо съесть несколько граммов; плюс, очевидно, именно он используется для фторирования зубной эмали). Перекись вроде ни с чем особенно не реагирует и не особо вредна, опять же, если не пить её. Метанол так же не сильно опасен, если не пить.
Аноним 03/04/18 Втр 20:51:23  435905
15036461359140.png (2Кб, 400x400)
Проверялось ли опытным путём равенство гравитационной и инерционной масс?
Аноним 03/04/18 Втр 21:24:12  435906
>>435905
Конечно
Аноним 03/04/18 Втр 21:47:55  435907
>>435906
Каким образом и где можно почитать про эксперименты?
Аноним 04/04/18 Срд 01:06:21  435909
lunatism-b.jpg (41Кб, 600x443)
Поясните за сомнамбулизм/лунатизм - почему появляется, как работает, болезнь ли это и можно ли доверять научпопу типа National Geographic в этом вопросе (то что я видел очень сомнительно).
Аноним 04/04/18 Срд 06:40:18  435912
IMG1320.JPG (108Кб, 670x463)
Нужно сделать контейнер с такой крышечкой, что бы держала 30-40Мпа если можно больше, еще лучше что мне гуглить, что бы понять как устроен узел, который держит такое давление?
Аноним 04/04/18 Срд 14:00:45  435925
z25114c1d.jpg (70Кб, 1215x1024)
У меня тупой не научный вопрос.
В каком году мы живём то?
Вот если мне месяц назад исполнилось 20, то сейчас я уже живу двадцать первый год.
А с земными годами как? Сейчас 2018й или уже прошло 2018 и сейчас уже идёт 2019й?
Не понимат.
Отсчёт начался с нулевого или с первого года?
Аноним 04/04/18 Срд 14:23:24  435927
>>435925
Лолд...
>>435907
Большинство результатов предсказаний ОТО поедет по пизде, если эти массы не совпадают.
Поэтому заходи на гугл академию и ищи там любые статьи какие хочешь.
Аноним 04/04/18 Срд 14:31:19  435928
>>435912
автоклав Флагман до 4 атм
Аноним 04/04/18 Срд 14:58:01  435930
IMG1502.JPG (66Кб, 811x541)
>>435928
Бро, мне бы что бы 300атм держала крышечка
Аноним 04/04/18 Срд 16:01:17  435936
>>435930
Ты чё там собираешь?
Аноним 04/04/18 Срд 16:44:43  435938
Да так, есть зорошие зимические идеи. Но нужно понять как нагуглить крышечку к этим атмосферам и паскалям
Аноним 04/04/18 Срд 17:55:31  435942
>>435938
Город назови, будем ждать новости про умершего при взрыве самодельного баллона на 300 атмосфер.
Аноним 04/04/18 Срд 20:26:16  435945
>>435409 (OP)
Блин аноны завис и не могу решить вроде бы простое задание по теории вероятности
У нас есть 20 шаров - 10 белых и 10 черных
Какова вероятность случайно выбирая 15 шаров - выбрать 10 белых и 5 черных.
Аноним 04/04/18 Срд 21:01:41  435948
>>435945
условие кстати неоднозначно. Шары выбираются с возвратом или нет...
Аноним 04/04/18 Срд 21:09:15  435949
>>435948
Скорее всего нет.
В таком случае p = шанс вытащить 5 чёрных шаров (ибо изофалично, вытащить 10 белых и 5 чёрных (тогда в коробе остаются 5 чёрных как ни крути), либо вытащить 5 чёрных (и в коробке будут 10 белых и 5 чёрных).
p1 для 1 ч. шара = 0.5
p2 = 10/20 9/19 ~= 0.236
...
p5 = 10/20
9/19 8/18 7/17 * 6/16 ~= 0.0162, если нигде не ошибся в рассчётах.
Аноним 04/04/18 Срд 22:11:07  435952
>>435912
берёшь в солиде хуяришь модельку, задаёшь материал и прикладываешь в исследовании распределённое усилие на всю внутреннюю поверхность

или тупо находишь в гугле два калькулятора
1)расчёт труб на давление
2)расчёт резьбы
усилие на резьбу-посчитаешь крышку сосуда как поршень-давление на площадь крышки, получишь срывающее усилие

т.к. это самый простой способ сделать такой сосуд-труба с глухим дном на одном конце и резьбой на другом, и резьбовая крышка
а другой конструкции сосуд ты хуй сделаешь, то тех двух калькуляторов хватит, с солидом даже нечего заморачиваться

на глаз сходу там труба со стенкой мм 15 где-то толщиной, при внутреннем диаметре около 200мм
Аноним 05/04/18 Чтв 09:11:59  435966
Зачем Clostridium botulinum'ям нужен ботулотоксин и как он в них появился? Кого они собрались им убивать кроме домашних консерваторов?
Аноним 05/04/18 Чтв 10:28:43  435967
>>435966
>ботулотоксин
Могу лишь предположить два возможных сценария: либо случайно, что вряд ли, либо они так обеспечивают себе размножение - выделяют токсин, время от времени убивают своего носителя, пожирают его труп, плодятся и рассчитывают, что кто-нибудь ещё позарятся на этот труп и тоже поможет в размножении.
Аноним 05/04/18 Чтв 15:45:30  435978
>>435409 (OP)
Блять двач, почему 0/2=0, а при проверке же 2≠0/0, получается 0/2≠0?
Аноним 05/04/18 Чтв 20:50:30  435992
IMG1548.JPG (88Кб, 811x730)
>>435952
>>435912

О калькулятор есть! Шикарно, держи няшу

Скажи, а ты не хотел бы заработать денюжку сделав небольшой расчет?
мне нужна, грубо говоря, стальная банка, которая может держать 300-400 в идеале 500 атмосфер горячей жижи. Нжно понять как выполнить узел, через который в в банку в холодном виде(!) будем загружать жижу в контейнере. Расчет мне нужен лично для себя, так что оформлять красиво нинужна если да то %%forfamilyady@yandex.ru
Аноним 05/04/18 Чтв 20:51:36  435993
IMG1553.PNG (490Кб, 674x1482)
Аноним 05/04/18 Чтв 20:56:00  435994
>>435978
0 / 0 в некоторых случаях равен двум. Есть еще отношение 0 * бесконечность, бесконечность / бесконечность. Они называются неопределенностями
Аноним 05/04/18 Чтв 21:08:00  435995
>>435836
потому что для всех степеней есть свои слоги, начиная от атто- и заканчивая экза-.
Отказались потому что ученым так оказалось удобнее и проще.
Аноним 05/04/18 Чтв 21:32:58  435996
>>435409 (OP)
У меня тут на дваче, будет Оксана.
Аноним 05/04/18 Чтв 22:38:54  435998
>>435992
Если 500-необходимое рабочее давление, то учитывая что это для газа/пара(а газ под большим давлением это натуральная бомба) да плюс нагрев.. нужно брать запас раза в 2 минимум, я бы взял в 3.
Какой необходим внутренний объём тары?
Какая температура нагрева?
Возможно придётся применить что-то типа жаропрочной нержавейки, да и она будет наверное предпочтительна, чем просто сталь, учитывая что ты в этом котелке чё-то химичить собрался.
Кстати как то что ты туда загружать собрался взаимодействует с металлами вообще, и при тех условиях(температуре и давлении) в частности?
Может тебе вообще неметаллический сосуд нужен.
Аноним 06/04/18 Птн 00:09:21  436005
>>435998

>>435998

Желательное рабочее давление 207атм или 21 000 кПа , но если можно будет без существенных затрат его повысить, то это очень здорово

Температура пока 370с , потом посмотрим

Обьем. Внутренний диаметр полости 16 см высота сантиметров 10. Эти параметры можно варьировать.

>Кстати как то что ты туда загружать собрался взаимодействует с металлами вообще, и при тех условиях(температуре и давлении) в частности?
Мы там все фторопластом внутри фундируем. Фторопласт нейтрален до 450с

>Может тебе вообще неметаллический сосуд нужен.
Вопрос стоимости. Скланяюсь к металлу, его легче обрабатывать, если надо будет допилить или повторить прибор своими силами




Аноним 06/04/18 Птн 00:10:41  436006
>>436005
>склоняюсь
Быстрофикс
Аноним 06/04/18 Птн 09:04:05  436015
>>435409 (OP)
Частота электростатического поля равна нулю, тогда и энергия фотона электростатического поля равна нулю?
Аноним 06/04/18 Птн 20:11:00  436068
>>436005
Метадон шоль синтезируете, готов стать тестером, пожизненно.
Аноним 07/04/18 Суб 02:16:34  436073
/
Аноним 07/04/18 Суб 13:48:57  436085
>>436015
>тогда и энергия фотона электростатического поля равна нулю
Хуже, он виртуальный.
Аноним 07/04/18 Суб 16:09:37  436088
>>436085
Подробнее?
Аноним 07/04/18 Суб 17:19:34  436092
А есть где-нибудь база данных всяких полезных фактов, вроде плотностей и теплоемкостей разных веществ, к которой бы можно было обращаться на SQL? Желательно через веб-интерфейс.
Аноним 07/04/18 Суб 18:37:18  436093
>>436092
В ниишных сетках наверняка много таких баз пылится, но чтобы в доступе с постоянной возможностью подключения вряд ли, тебе надо, может, с каким-нибудь сотрудником профильного нии скорефаниться, ну а SQL запросы сейчас любой школьник умеет делать.
Аноним 07/04/18 Суб 23:39:46  436097
Что значит магнитное поле замкнуто а электрическое нет?
Если ты создаешь разность потенциалов, то где-то будет плюс, а где-то обязательно минус, и эти поля скомпенсируют себя - то есть замкнут.
С другой стороны, если взять магнит в виде длинной проволоки, мы получим аналог электрического поля: с одной стороны будет практически полностью северный полюс, с другой южный. Если представить магнитное поле как разность магнитных потенциалов, то по сути происходит то же что и с электрическим.
Аноним 08/04/18 Вск 02:17:52  436100
>>436097
Существуют заряды, от которых идут силовые линии.
Не существует магнитных зарядов. Силовая линия замкнутая, нет не "истоков" не "стоков".
Аноним 08/04/18 Вск 02:30:43  436102
>>435994
Количество ебырей которые абли твою мамку когда она тобой залетела равно двум. Не путай предел и равенство.
Аноним 08/04/18 Вск 02:41:51  436105
>>436100
>Существуют заряды, от которых идут силовые линии.
И эти линии идут в сторону противоположного заряда.
Или все дело в том что конкретный заряд идет в сторону любого противоположного а не конкретного? Ну так это ж вроде градиент, а силовых линий не существует?

>Силовая линия замкнутая, нет не "истоков" не "стоков".
Ну смотри: вот есть батарейка, у нее есть + и -. Из - вылетают электроны, заряд передается через цепь и возвращается добавляя электроны в + - цепь замкнута. Даже правило такое есть - ток течет только в замкнутой цепи.

Получается и магнитное поле - это 2 параметра(аналога зарядов), разность между которыми дает градиент магнитного поля - те самые силовые линии.

Если отойти от доменов дипоей: вот в вакууме катушкой создается магнитное поле и электрическое, оба представляют собой градиент своего параметра, при чем оба имеют 2 параметра-максимума.



Аноним 08/04/18 Вск 02:47:52  436106
>>436105
Попробуй забыть аналогии и просто посмотреть математику.
Аноним 08/04/18 Вск 11:50:27  436113
Что такое 0, это полное отсутствие или минимально возможное наличие и есть ли разница?
Аноним 08/04/18 Вск 12:32:23  436115
>>436113
Нулей целый вагон и маленькая тележка.
В теории групп есть ноль.
В линейной алгебре. В полях есть ноль. и т.д.
Аноним 08/04/18 Вск 12:32:32  436116
>>436106
Математика не даст мне представление физики, это лишь метод вычисления но не понимания сути.
Аноним 08/04/18 Вск 12:44:12  436117
>>436115
Это по физике был вопрос, а не по математике - забыл сказать.
Аноним 08/04/18 Вск 12:50:44  436118
>>436113
>Что такое 0
https://www.youtube.com/watch?v=f8r2J0pCi8Q
https://www.youtube.com/watch?v=9UgSDGrxlAE
Аноним 08/04/18 Вск 13:01:03  436119
>>436116
>это лишь метод вычисления но не понимания сути
Не соглашусь.
Во многих вещах, пока мат. модель не построишь, в сути вещей не разберешься.
Простой пример, теория стабилизации кристаллическим полем, в комплексах переходных элементов снимается вырождение d подуровня, и математика дает прикинуть как именно и от чего зависит это. После чего прекрасно объясняет миллион вещей от цветов до устойчивости некоторых веществ.
Аноним 08/04/18 Вск 13:40:07  436120
>>436118
Класс
Аноним 08/04/18 Вск 14:00:19  436121
>>436120
https://www.youtube.com/watch?v=kzV4KuZPQWk
Аноним 08/04/18 Вск 14:02:31  436123
>>436120
https://www.youtube.com/watch?v=VnKjGF1RQpI
https://www.youtube.com/watch?v=c4dFqRFApls
Аноним 08/04/18 Вск 17:22:59  436132
Существуют проекты по опреснению морской воды за счёт перегонки. Почему именно перегонка? Не эффективнее ли (энергетически) заставить эту же воду закипеть при пониженном давлении, откачивая её пары и оставляя соль?
Аноним 08/04/18 Вск 17:24:38  436133
>>436132
Зачем, если можо при помощи того же понижения давления очищать воду обратным осмосом?
Аноним 08/04/18 Вск 18:00:17  436135
>>436132
Я тебе так скажу.
Ты хочешь...
Соленая вода===> какие-то манипуляции ====> соль + чистая вода
Так вот, от пути процесса, энергозатраты не зависят. Основы термодинамики. Затраты ниже
изменения энергии Гиббса в этом процессе не сделать.
Аноним 08/04/18 Вск 19:48:17  436148
Сколько пожарных извещателей нужно ставить в помещение (читай в комнату) и в каждый отсек конкретно?
Аноним 08/04/18 Вск 21:28:41  436155
>>436135
>Так вот, от пути процесса, энергозатраты не зависят. Осно
Зависят ещё как! Ты на много порядков ошибается.
Обратный осмос - ты создаёшь давление, тратишь ресурсы на создание фильтров, получаешь соль, чистую воду и непригодные фильтры. Второй вариант - ты создаёшь пониженное давление, ты греешь воду (тепло уходит в окружающее пространство), получаешь соль и пар. Третий вариант - ты просто греешь воду до кипения, огромное количество тепла уходит в окружающее пространство, получаешь соль и пар.
Насчёт кипячения при пониженном давлении не скажу, но есть у меня подозрения, что это всё-таки менее выгодный процесс, чем фильтры. Очевидно, иначе бы кто-то уже додумался и использовал этот процесс.
Аноним 09/04/18 Пнд 00:12:14  436162
>>435889
Понадобится обычный велосипед. Снимаем всё ненужное нахуй, к задней оси крепим насос. На крышу дома впидорашиваешь бочку, в неё нагоняешь воды, и закрываешь клапан. Потом в любой нужный момент, открываешь клапан, вода стремится вниз, крутит генератор, дает энергию.
Можно понаставить автомобильных аккумуляторов, и заряжать напрямую.
Аноним 09/04/18 Пнд 02:22:02  436168
Почему математичка так сложна для нашего мозга? Например, я спокойно представляют круг, радиус, треугольник, и примерно определяю длину отрезка в зависимости от угла. Синус в голове я бы хуй вычислил без листка бумаги даже с такой погрешностью.
Мозг может обучаться и затем обрабатывать информацию уже на бессознательном уровне, но с математикой такое не выходит.
Мозг является некой вычислительной машиной, математика была изобретена человеком для анализа мира. Есть ли альтернативы математике, более совместимые с "архитектурой" мозга?

Или просто альтернатива обычной математике без линейных вычислений и позиционных систем счисления?
Аноним 09/04/18 Пнд 02:42:18  436170
>>436168
Мозг вполне обучается математике и выдаёт информацию на уровне интуиции, просто нужно больше опыта и результат часто менее применим. Точный синус ты, конечно, не вычислишь, но прикинуть каой он примерно должен быть можешь. А это и есть интуиция. В остальной человеческой деятельности по большей части тоже интуиция тебе не даст сколько-нибудь полезного и точного решения - надо проверять, думать, считать (хотя тут опять приходим к математике). Однако без интуиции хуй что-то сложнее школьных задач - ведь нужно понять каким образом это решать.
Если же тебе интересно почему мы не можем научиться в уме вычислять пи до миллионного знака - так это потому, что каждая цифра с т.з. мозга - это объект, и у каждой элементарной операции т.о. будут десятки объектов, и самих операций куча, и операции эти - тоже объекты, которые тоже нужно удерживать в голове, тогда как наш мозг едва способен удерживать в активной памяти и как-то оперировать только с тремя-пятью объектами. Поэтому что-то сложнее 2+2 уже требует активной мыслительной деятельности, в подкорку это не прошьёшь. Числа, конечно, можно представлять и не отдельными цифрами, а величиной, но в таком случае получается очень уж неточно - сигналы нейронов в принципе не способны к точности сильно выше "один-два-много".
Аноним 09/04/18 Пнд 03:10:29  436171
>>436170
>Точный синус ты, конечно, не вычислишь, но прикинуть каой он примерно должен быть можешь.
Однако расчет а математике это цепь из вычислений, и есть для расчета 1 синуса достаточно интуиции, то следующее же действие с этим примерным результатом полностью обесценивает результат. Для такого подхода годится только 1 действие. В то же время проследить зависимость и выстроить некое внутреннее сопоставление "входные данные => результат" мозг может, а значит возможно некое иное описание в 1-2 действия для интуитивного понимания. Тот же график функции позволяет вычислить примерный результат полностью в уме(представляя график).

>что каждая цифра с т.з. мозга - это объект, и у каждой элементарной операции т.о. будут десятки объектов, и самих операций куча, и операции эти - тоже объекты, которые тоже нужно удерживать в голове, тогда как наш мозг едва способен удерживать в активной памяти и как-то оперировать только с тремя-пятью объектами.
Вот и я о том же, обычная позиционная СС тут не подходит. Ну или правила вычисления нужно изменить. Даже обычная сумма большинством вычисляется как представление сложения "на листочке". Если мозг не может держать больше 3 объектов, нужна модель вычисления с таким условием.
>Числа, конечно, можно представлять и не отдельными цифрами, а величиной, но в таком случае получается очень уж неточно - сигналы нейронов в принципе не способны к точности сильно выше "один-два-много".
Тут еще смотря как из представлять. Как я понял, даже единую величину мозг воспринимает как относительную(напр., 5 это то что больше 1 но меньше 10), возможно от этого стоит отталкиваться. Вообще, в природе тоже нет абсолютных констант, все относительно чего-то.

>"один-два-много".
То есть мозг таки с троичной СС работает?
Аноним 09/04/18 Пнд 03:38:31  436173
>>436171
> проследить зависимость и выстроить некое внутреннее сопоставление мозг может
Как сказать. Вот смотришь ты такой на график синуса и вроде бы понимаешь какой он. Но стоит спросить чему будет равно значение этого графика по y при x=30 - и уже почти без шансов. С большинством других графиков тоже самое, мы можем запомнить пару изгибов, пару "якорей" - значений функций в заданной точке, да примерный масштаб значений, а чуть сложнее и мы уже даже примерно не можем угадать.
> нужна модель вычисления с таким условием.
удачи, что я могу сказать. Только не удивляйся, если ничего не придумаешь - всё-таки это невозможно, уже по той простой причине, что для этого тебе нужно иметь цепочку простых операций, но этих операций будет сильно больше, чем ты можешь запомнить, и уже даже для того, чтобы такую цепочку составить, тебе нужно преобразовать изначальные вычисления, содержащие опять же слишком много объектов, чтобы ты с ними мог оперировать одновременно.
> 5 это то что больше 1 но меньше 10
только пока ты об этом думаешь или когда тебе нужно сравнение. Иначе - формат представления будет другим (и скорее всего именно как какой-нибудь объект или группа связанных объектов). Т.е. когда ты считаешь сумму и не думаешь, что 5 больше 6, то 5 не больше 6, 5 - это 5, которое в сумме с 1 даст 6, в сумме с 2 - 7 и т.д (я полагаю большинство людей хранит сумму как выученную таблицу, так же как и умножение).
> То есть мозг таки с троичной СС работает?
Нет, мозг вообще не работает в какой-либо системе счисления, он работает как система активаторов, тормозов и рандома. Приходит на синапс много активирующих сигналов и мало тормозящих на дендрит - нейрон начинает "мигать" сигналами. Приходит немного сигналов или просто в силу случайных химических процессов - он мигает один-два раза и умолкает. Частота мигания - это некое подобие силы сигнала. Только нейроны не очень-то могут эту частоту регулировать, там очень много случайных срабатываний или не срабатываний и хаотичных процессов. Поэтому иногда это "Ноль-много", иногда "ноль-один-много", иногда "ноль-один-несколько-много", и т.д., но общей системы как правило нет.
Аноним 09/04/18 Пнд 03:39:09  436174
>>436173
>при x=30
Радиан, не градусов. В градусах - ну, например, тысяч пять.
Аноним 09/04/18 Пнд 06:54:39  436182
>>435409 (OP)
Суп, /sci/ суть такова. Имеется массив из овердохуя рядов связанных между собой значений (характеристик изображений и сопутствующих им данных) вида

#1 - куча характеристик - ОК
#2 - куча характеристик - НЕ ОК
#3 ...

Решение ОК/НЕ ОК выносится визуально, т.е. мне как бэ понятно что норм, а что не норм. Вопрос в том, как это моё субъективное решение увязать с имеющимися объективными характеристиками - нужно понять, насколько сильно ОК/НЕ ОК зависит от того или иного значения, и зависит ли вообще.

Сам я не то, чтобы дохуя инженер, поэтому я нихрена не понимаю, куда мне дальше двигаться с этим, кроме как нахуй. Может мне какие-то эти ваши нейросети нужны или вроде того?
Аноним 09/04/18 Пнд 11:33:39  436190
>>436182
Я сильно не уверен, что вообще понял о чём ты, но по идее свёрточные сети тебе помогут.
Аноним 09/04/18 Пнд 11:40:30  436192
Как вывернуть с[...].webm (19004Кб, 256x144, 00:21:27)
>>436171
Ты почему-то ограничиваешь математику только вычислительной математикой. Почему мы не можем синусы считать в уме вот этот анон уже подробно ответил >>436170 и твои утверждения про цепочки операций там совершенно валидны. Только математика на этом не заканчивается, математика в целом - это наука о закономерностях и абстракциях, а для создания интуиции об абстракции точные вычисления нужны далеко не всегда, пример тому топология, в основе которой лежит как раз полная свобода того как ты можешь деформировать там свои множества, массируя их как астронавты Аполлона наминали собственный кал в пакетике со стрептоцидом. Математики не дураки и часто делают то что наоборот помогает создавать интуитивное понимание того или иного объекта или процесса.
Аноним 09/04/18 Пнд 11:59:10  436194
>>436192
Я тот анон и, кстати, меня всегда интересовало как работает топология? Т.е. все эти трансформации - как они описываются? Какая за ними вычислительная математика? Или сидят с нитками-блендером-майа и мнут?
Аноним 09/04/18 Пнд 12:14:18  436195
>>436194
>Т.е. все эти трансформации - как они описываются?
Ну в целом никак, топологам это не нужно. Цель топологов - дырки в пространстве сосчитать, а все эти трансформации - это всего-лишь способ определить эквивалентность пространств, т.е. если одно пространство можно деформимровать в другое как на вебмрилейтед - то с точки зрения топологии это одно и то же пространство. Но если очень хочется, то трансформации описываются через гомотопии - это как раз та самая непрерывная трансформация сферы к примеру.
Если очень хочется именно что-нибудь посчитать - есть подход симплициальных комплексов, когда гамалогии считают разбивая пространство на симплексы как треугольные полигоны в 3d-графике после чего можно отдельно для каждого посчитать что-то и потом их объединить для всего пространства. С гомотопиями там можно определить фундаментальную группу и уже с пространством как с группой работать. Ну и соответственно в обоих случаях (в случае симплициального комплекса или в случае фундаментальной группы) можно легко сосчитать количество дырок в пространстве.
Аноним 09/04/18 Пнд 14:48:41  436203
>>436182
Да, сети один из вариантов. Я так понял у тебя классичская задача распознавания образов на картинке. Тут две группы подходов, если характеристики в левой части осмысленные, то ты можешь и сам вручную подобрать формулу от них которая будет говорить результат да или нет. И, конечно, нужен способ считать эти характеристики по изображеням. Второй вариант сети - они сами найдут необходимые характеритики в процессе обучения и сами выведут формулу. Также можно использовать сеть, которая будет распознавать не картинки а вот эти наборы характеристик, это если они у тебя заданы изначально: т. е. обучишь сеть характеристиками с результатми и она построит формулу за тебя.
Аноним 09/04/18 Пнд 16:09:36  436220
>>436203
Сети вообще не с картинками работают, это картинки перекодируются в удобоваримый формат. А так - они со значениями работают, преимущественно с нормализованными значениями, а там уж картинка или характеристики - по барабану.
Аноним 09/04/18 Пнд 20:55:56  436257
5120.png (579Кб, 638x609)
Можно ли снять статическое напряжение с тела помыв руки?
Аноним 09/04/18 Пнд 23:31:05  436267
>>435409 (OP)
Почему красный фотон легче синего?
Аноним 10/04/18 Втр 00:48:27  436270
>>436267
Мозг у тебя легче, а красный фотон имеет меньшую энергию, т.к. частота - это и есть его импульс, это считай что одинаковые понятия с т.з. квантмеха.
Аноним 10/04/18 Втр 00:56:05  436271
Почему после смерти мозга оставшееся тело не может самостоятельно дышать и стучать сердцем? Для выполнения этих рутинных операций тип строго обязательно иметь мозг?
Аноним 10/04/18 Втр 00:57:55  436272
На хабре нашел такую дичь:
>Деление можно заменить умножением на обратный (его для точной арифметики получают алгоритмом основанным на методе Ньютона для приближенного решения нелинейных уравнений) и тогда асимптотическая сложность деления равна сложности умножения

Но я не нашел этого конкретного алгоритма, или просто не понял нихуя. Но, тем не менее, дай наводку, наукач.
Аноним 10/04/18 Втр 01:13:40  436273
>>436271
Должен же кто-то электричество им слать. Без электричества человек не работает.
Аноним 10/04/18 Втр 01:25:34  436274
>>436272
>Деление можно заменить умножением на обратный
В случае с полем рациональных чисел например, возьмем два числа a и b, найдем их деление, каждое число представимо в виде дроби a = p1/q1, b = p2/q2. Вместо нахождения a/b можно найти a*b⁻¹, где b⁻¹ - обратный элемент, b⁻¹ = q2/p2. В случае если известен алгоритм поиска обратного элемента (и он сщуествует для всех b ≠ 0), деление сводится к умножению на обратный.
Аноним 10/04/18 Втр 01:57:44  436278
Где-то в выживач-разделе высказывалось мнение, что зимой человек может спать на улице хоть в -20°, если одеть 100500 всяких шуб на себя. Возможно ли это? (А с современными нанотехнологиями там явно меньше 100500 слоев понадобятся).
Инбифо: если бы было возможно, щас бы все охотники так делали.
Аноним 10/04/18 Втр 01:58:40  436279
>>436273
Цыплята некоторые работают без головы, а человек что, особенный какой?
Аноним 10/04/18 Втр 01:59:37  436280
>>436279
Остаточные конвульсии мышц. Человек тоже так может работать какое-то количество секунд.
Аноним 10/04/18 Втр 02:20:50  436281
>>436271
Мышцы лёгких (грудной клетки и диафрагмы, фактически) управляются из мозга, поэтому они перестают работать сразу - им просто не поступает сигнал. Сердце управляется собственным ритмоводителем и ещё какое-то время может продолжать биться, но сам ритмоводитель так же управляется из мозга и без сигналов приходит в уныние и работать отказывается. Почки, печень и другие органы более независимы и работают немного дольше - до тех пор, пока из-за гипоксии не теряют всю энергию в клетках. Дольше всего "работают" клетки мышц и кожи, но первые опять же ничего не делают без приказа сверху, а вторые в принципе ничего не делают.
>>436279
У птиц больше функций парасимпатической нервной системы делегированы спинному мозгу, который без начальства начинает паниковать и слать хаотичные сигналы, что выглядит как какая-никакая деятельность. А если же ты про того несчастного петуха, которому отрубили глаза и клюв, назвав это "без головы", то ему мозг не отрубили, по большей части.
>>436278
Конечно возможно! Пингвины, белые медведи и прочие любители закалки же живут и, очевидно, спят иногда, и им достаточно даже одной шубы. Всё дело только в теплообмене - человеку будет достаточно даже +20, чтобы окочуриться в какой-нибудь медной фольге в обтяжку, если она с радиаторами, и точно так же ему будет достаточно лёгкой курточки из аэрогеля, имеющего рекордно низкую теплопроводность.
Аноним 10/04/18 Втр 05:50:39  436282
image.png (319Кб, 900x555)
Читал очередное пережёвывание старых фильмов и обратил внимание на драмму в фильме «Космическая одиссея 2001 года».
Там компьютер с искусственным интеллектом решил убить всех человеков как бы не со зла, а потому что в него заложили взаимоисключающие параграфы полностью информировать экипаж космолёта об экспедиции @ скрывать от экипажа настоящую цель полёта.
И вот у меня теперь вопрос: а как обстоят дела ныне, в 2018?
А то так много сообщают об успехах ИИ вообще, и модного ныне направления - нейросети - в частности. Разрабатывают ли нынче такой ИИ, чтоб он решал проблемы взаимоисключающих параграфов. И если да, то какие пруффы и какие успехи?
Аноним 10/04/18 Втр 06:14:02  436283
>>436282
Нейроночки умеют сомневаться. Не бинарные. У них не жесткая логика.
Аноним 10/04/18 Втр 06:22:51  436284
image.png (281Кб, 418x626)
>>436283
То-то я вижу, как гугловская нейросеть требует от меня указать дорожные знаки и витрины там, где их нет и в качестве решения парадокса пойти нахуй.
Аноним 10/04/18 Втр 07:56:28  436287
>>436282
Вот тут почитай: https://ru.wikipedia.org/wiki/Продукционная_модель_представления_знаний
Там из-за необратимости дизъюнкций, продукции в семантической сети продукционных правил - начинают противоречить друг-другу.
Для снижения роста противоречивости - применяются два маханизма - это механизм исключений и механизм возвратов.
Для выявления противоречий - используется автомтаческое доказательство теорем, и механизмы перебора с возвратами.
Аноним 10/04/18 Втр 07:57:38  436288
>>436284
Разве это не один большой дорожный знак?
Аноним 10/04/18 Втр 13:32:17  436308
image.png (73Кб, 300x159)
>>436282
В фильме был если и не сильный ИИ, то на один шаг позади сильного, т.е. он как минимум способен принимать осмысленные решения. У нас сейчас до этого ещё далеко. Промышленные системы где-то на уровне чуть выше муравья, экспериментальные... ну, не знаю, ящерицы?..
Однако вангую, что проблема взаимоисключающих параграфов крайне раздута без повода. Это всё равно что бояться, что если поделить на ноль, то весь мир полетит в пизду. Потому что даже слабый узкоспециализированный ИИ периодически должен иметь дело с противоречиями и обычно всё сводится к взвешиванию.
С другой стороны, по этой же причине в ИИ не прошьёшь законы Азимова, это скорее просьбы Азимова будут.
Аноним 10/04/18 Втр 14:10:55  436312
>>436105
>Ну смотри: вот есть батарейка, у нее есть + и -.
Есть "батарейки" только с "+". Или минусом...
Если потереть... >:-)
Магнитных таких есть нет.
Аноним 10/04/18 Втр 14:34:07  436314
>>436274
Это понятно. Просто мне нужно получить, к примеру, значение 1/17 до определенного числа знаков после запятой. И насколько я понял из метода касательных, мне все равно надо делить. А моя задача стоит в делении очень длинного числа на очень длинное число. То есть я не могу использовать операцию деления для расчёта обратного элемента. Мне надо как-то получить обратный элемент без деления и потом умножить его на свое делимое, перенеся потом запятую.
Аноним 10/04/18 Втр 14:47:20  436315
>>436308
>Однако вангую, что проблема взаимоисключающих параграфов крайне раздута без повода. Это всё равно что бояться, что если поделить на ноль, то весь мир полетит в пизду.
С поводом.
Если у вас взаимоисключающие параграфы (или деление на нуль), то у вас нет алгоритма. А это значит, что по вашему алгоритму система уже не заработает. Если кто-то другой о вас позаботился и сделал таки алгоритм (без противоречий) на этот случай, то может быть катастрофы и не будет, если ОНИ её не планировали. Если ОНИ что-то вообще планировали.
Вы просто не контролируете ситуацию. И если система чем-то управляет... на самом деле...

То исход дела только случайно может оказаться сколько-то в вашу пользу. Или чью-то. Вопреки некоторым мнениям ИИ довольно легко "расстаётся с жизнью" просто чтобы стукнуть вас по голове... (и засунуть в вакуум). Получаем непрерывную русскую рулетку с полным так сказать барабаном. Почти...
Аноним 10/04/18 Втр 15:25:51  436316
>>436132
Это и есть оно. Перегонка. Вариант.
Аноним 10/04/18 Втр 15:46:26  436317
>>436308
>С другой стороны, по этой же причине в ИИ не прошьёшь законы Азимова, это скорее просьбы Азимова будут.
Насколько всё же Азимов был гуманитарен...
Аноним 10/04/18 Втр 15:58:56  436318
>>436317
Гуманитарен и излишне моралфажен. К сожалению, других писателей не завезли. Можешь подрочить, читая мануал к стиралке, его точно технари атеисты писали.
Аноним 10/04/18 Втр 16:13:44  436320
>>436315
Ты ИИ незаалгоритмизируешь, балда, а если попытаешься, то в лучшем случае получишь экспертную систему, которая от ИИ далека настолько же, насколько большая советская энциклопедия далека от Хокинга.
Сильный ИИ тем и отличается от той же экспертной системы, что работает не по строгим тупым и неповоротливым алгоритмам, а как гибкая перестраивающаяся динамическая система, способная не только к усвоению новых навыков или информации, но и в какой-то мере к перестройке собственной системы усвоения - так же, как и нам, людям, в школе с трудом даётся даже таблица умножения, а лет в 25 мы можем запомнить десятки адресов местной локалки, пару сотен веб-адресов и т.д. и т.п. - мы способны учиться учиться. Без этой функции объём работ по алгоритмизации и обучению всему и вся будет чуть больше, чем десять жизней вселенных трудолет, и даже если бы это можно было реализовать, один хуй какой-нибудь сильный ИИ сможет однажды покопавшись во всём этом говнокоде найти уязвимости и поставить такой алгоритмический ИИ раком в любом вопросе. Не потому, что он реально умнее или лучше знает, а именно потому, что он динамичен.
Аноним 10/04/18 Втр 17:05:55  436321
>>436315
Посмотри как работает тот же alfa zero.
Шахматный гений на нейросетях.
У него не игры, а произведения искусства. При этом давай ему одну и ту же позицию, он будет не всегда играть один и тот же ход, особенно если позиция очень сложная.
Они умеют сомневаться.
Аноним 10/04/18 Втр 17:20:26  436323
>>436320
>работает не по строгим тупым и неповоротливым алгоритмам, а как гибкая перестраивающаяся динамическая система, способная не только к усвоению новых навыков или информации, но и в какой-то мере к перестройке собственной системы усвоения
Почему?
>Ты ИИ незаалгоритмизируешь
Аноним 10/04/18 Втр 17:21:09  436324
А можно на авторотации посадить спускаемый аппарат с орбиты? Типа тормозной конус чтоб потом как цветок разворачивался в лопасти.
Аноним 10/04/18 Втр 17:25:35  436325
Есть ИТТ биологи, ботаники ? Я читал про способы генерации фракталов, в частности L-системы. Так, первым приложением L-систем была ботаника, а не генерация деревьев для 3Д визуализации. Мне стало интересно, зачем конкртено это нужно ботаникам. Ну нашлась модель описания роста деревьев, а потом что с ней делать ? В каких задачах это может быть применено ?
Аноним 10/04/18 Втр 17:31:21  436326
>>436323
Именно потому, что невозможно закодировать все возможные ситуации, предусмотреть все противоречия. Это как задача коммивояжёра, только бесконечномерная и с неизмеримым астрономически большим количеством точек. А с учётом того, что вселенная не стоит на месте, всё вокруг меняется и эволюционирует во времени, то ещё и точки в этой задаче постоянно переползают с места на место.
То, что ты предлагаешь - это по сути решение её перебором в попытке найти идеальное решение. Гипотетически это выглядит возможным, фактически - нет, т.к. это требует столь же неизмеримых астрономически больших затрат.
Неалгоритмический ИИ вместо этого работает как самообучающийся локально-оптимальный агент, который выдаёт не идеальный, но как правило вполне приемлемый вариант и старается обучиться выдавать лучший результат.
Единственный известный на сегодняшний день способ реализации такого агента - это нейросети, по сути то же, что работает у нас в мозгу.
Аноним 10/04/18 Втр 17:33:31  436327
>>436326
И что, нельзя сделать алгоритм с участием нейросетей?
Аноним 10/04/18 Втр 17:43:32  436328
>>436327
Алгоритм подразумевает непосредственные инструкции и строгий порядок действий по заданным критериям.
Единственный алгоритм в нейросетях касается только базиса их работы, типа, считать входы, просуммировать, домножить, выдать на выход. Всё.
Нейросети настраиваются не вручную, а самостоятельно (сиречь, произвольно), поэтому предсказать каков будет результат их работы до обучения практически невозможно (т.к. с точки зрения математики это в лучшем случае система из сотен линейных уравнений с десятками тысяч переменных для довольно небольшой нейросети, и сотнями миллиардов уравнений с триллионами неизвестных для нашего мозга). Можно как максимум лишь направить их в нужное русло. Ну и наконец самое главное - там нет чётких алгоритмов, особенно если это активные сети, которые обучаются в процессе использования. Там всё постоянно меняется и перестраивается.
Аноним 10/04/18 Втр 17:57:02  436329
>>436327
Можно их подшить, как подшивают алкашей. Типа, нейросеть принимает неверное решение = самоустраняется.
Проблемка в том, что у нейросети для этого должно быть самоосознание, самоответственность, мораль и этика. А такие нейросети нам не нужны, т.к. и так по улицам ходят в огромных количествах.
Аноним 10/04/18 Втр 18:02:04  436330
>>436329
Вообще, мы ещё толком не знаем что такое самоосознание и прочее, мы только считаем, что это непременно должно быть что-то особенное, какой-то рубеж.
Я бы не удивился, если обнаружится, что никакой четкой черты в этом деле нет: чуть меньше нейронов и связей - и будет чуть меньше того, что мы могли бы принять за самоосознание. И пришлось бы думать где провести границу - то ли вытолкнуть миллионы людей за черту разумных существ, то ли включать дельфинов, китов, высших приматов и некоторых пернатых в число разумных.
Аноним 10/04/18 Втр 18:39:02  436332
>>436330
>что такое самоосознание и прочее
Охуенно, наверное, самому вводить определение и самому не знать, что оно означает.
Аноним 11/04/18 Срд 00:07:01  436341
>>436314
>Мне надо как-то получить обратный элемент без деления и потом умножить его на свое делимое, перенеся потом запятую.
Приведенный тобой метод замены деления на умножение на обратный применим только в случае если обратный элемент найти не составляет труда. Примером таких ситуаций является деление рациональных чисел или какие-нибудь поля вычетов по модую, в твоем же случае как я понимаю это вещественные числа, в этом случае подобный метод себя не оправдывает.
Аноним 11/04/18 Срд 01:20:05  436343
>>436324
Если в самом начале ты гасишь скорость более традиционными методами (читай абляционная защита или быдлоплитка как у шаттла) до терминальной, то не вижу теоретических причин почему ближе к поверхности не использовать авторотацию. Думаю никто так не делает только из-за дороговизны и ненадежности конструкции. Паращюты куда пизже. Ну или в крайне случае крылья как у шаттла.
Аноним 11/04/18 Срд 03:27:57  436345
>>436324
>>436343
Я думаю, проблема может быть в размере лопастей и их массе.
Аноним 11/04/18 Срд 04:00:31  436346
Почему ракетный двигатель менее эффективен в атмосфере? Интуитивно кажется, что должно быть наоборот - выхлоп двигателя, сталкиваясь с воздухом, будет создавать большее давление в камере сгорания и повышать эффективность реакции. Но факт есть факт.
Аноним 11/04/18 Срд 06:32:51  436348
Можно ли вычислить, сколько стоит жизнь определённого человека, прибегнув к эволюционной и молекулярной биологии? Как?
Аноним 11/04/18 Срд 07:58:28  436357
>>436348
Сколько стоит чего?
Аноним 11/04/18 Срд 08:00:53  436358
>>436345
Я не думаю что рандомный вертолет сильно легче капсуы союза скорее наоборот и ему не нужны массивные и большие лопости для посадки на авторотации.
Аноним 11/04/18 Срд 08:05:50  436359
>>436346
Эффективность камеры сгорания зависит не от давления внутри а от разницы давлений. Думай об этом так: реактивная струя вместо того чтобы улетать от ракеты с максимальной скоростью и следовательно передавать максимум импульса ракете (согласно закону сохранения импульса), сталкивается с воздухом который тормозит её и запихивает обратно в камеру сгорания, скорость у неё уже будет совсем не та что в безвоздушном пространстве и таким образом ракета будет получать меньше импульса на единицу потраченного топлива.
Аноним 11/04/18 Срд 12:46:16  436371
>>436358
Ну, ему нужен хвост для стабилизации полёта по рысканию.
Аноним 11/04/18 Срд 17:27:20  436381
Как в мозге могут быть железно прописаны мотивы поведения, такие как возбуждение от созерцание тянских сиськожоп? Или хищники агрятся, когда в глаза им смотришь - откуда они знают, что именно глазами все видят? И ещё вроде как у слепого зрительная зона в мозга занята обработкой другой информации. Допустим, затем он прозрел, но оглох, и тогда уже слуховая зона занята обработкой в т.ч. зрения. А затем слух вернулся, но слуховая зона переместилась в зрительную. Такое может? И почему нет?
Как-то очень неестественно такая запрограммированность кажется, будто в пластичную нейронную систему асик встроили.
Аноним 11/04/18 Срд 17:34:08  436382
image.png (67Кб, 300x168)
>>436381
>Как в мозге могут быть железно прописаны мотивы поведения, такие как возбуждение от созерцание тянских сиськожоп?
Они там не прописаны. Всё это генерируется на лету из подручного материала. Будешь воспитываться среди собак - на собак будет вставать.
>Или хищники агрятся, когда в глаза им смотришь - откуда они знают, что именно глазами все видят?
Они не дебилы.
>А затем слух вернулся, но слуховая зона переместилась в зрительную. Такое может? И почему нет?
Может. Но на этот процесс уйдёт до десятка лет, в зависимости от возраста пациента, и титанические усилия.
>Как-то очень неестественно такая запрограммированность кажется
Это для того, чтоб у тебя сердце не отказывало каждый раз, когда наступаешь мимо цветной плитки. Разум, сознание - шестёрки на побегушках у тела.
Аноним 11/04/18 Срд 18:08:57  436383
Пишу НФ-бестселлер про плоский мир. Вопрос - если предметы не скрываются за горизонтом, то какова будет дальность обзора с какой-нибудь вышки? Ну, я имею ввиду на каком расстоянии нельзя будет тупо различить что-то из-за ширины, так сказать, воздушного столба?
Аноним 11/04/18 Срд 18:32:49  436386
Сам нагуглил, что "Вода, даже оптически чистая, примерно в 1000 раз сильнее воздуха ослабляет видимый свет". То есть, если в воде что-то можно разглядеть в нескольких десятках метров, то в воздухе, стало быть, на десятки километров?
Аноним 11/04/18 Срд 18:48:58  436390
>>436383
>НФ-бестселлер
>плоский мир
>обычное зрение
https://vk.com/video-2675_146119305
Аноним 11/04/18 Срд 19:16:00  436391
Что есть эфир и почему отказались от идеи некой основной среды, поверх которой происходит все остальное? Ведь насколько я понял, в конечном итоге все частицы, вся материя, волны и все такое представляет собой электромагнитные волны различной конфигурации. Логично предположить что все эти волны идут по некой среде.
Аргумент "эфир должен вращаться вместе с Солнцем" и эксперимент, опровергающий это, лишь говорит что эфир не привязан к какой-то мат. точке, а так же не является материей или полем.

Только не надо сразу посылать в /re/, я исхожу просто из логики.
Аноним 11/04/18 Срд 19:21:19  436392
Почему в физическом вакууме постоянно и равномерно возникают и исчезают частицы, что их порождает и как?
Являются ли эти частицы причиной неопределенности и конечности точности измерений, ограничением скорости света а так же основой любого рандома в этом мире?
Аноним 11/04/18 Срд 19:24:09  436393
>>436390
А какое нахрен зрение должно быть в плоском мире? Мир-то такой же, 3D, просто Земля плоская, а не шар.
Аноним 11/04/18 Срд 19:29:04  436394
>>436382
> Они не дебилы.
Ну да, а в зеркале себя не узнают. И вообще представление о зрении и способсности других видеть предполагает понятийное мышление и эмпатию, ну я хз.
> Разум, сознание - шестёрки на побегушках у тела.
Ну это больше в отношении гормонов/нейромедиаторов правило.
Аноним 11/04/18 Срд 19:31:01  436395
>>436393
>Мир-то такой же, 3D, просто Земля плоская, а не шар
А чего она плоская?
Аноним 11/04/18 Срд 19:33:40  436396
>>436395
Это виртуальный мир. Сгенерировали вот такой, не подумав.
Аноним 11/04/18 Срд 19:43:17  436397
>>436381
Выше уже часть рассказывал: >>435784
Если дополнить под конкретно твой вопрос - за счёт генетически предопределённой морфологии, клеточных миграций и схемы развития нейронных связей у тебя практически всегда мозг работает как запрограммированный. Т.е. зрение идёт в зрительную кору, слух в слуховую. Ты гипотетически можешь перестроить это, но это будет вариант через жопу и работать он будет соответственно. Т.е. то, что у слепых зрительная кора работает - не значит, что они полноценно её используют и ей вообще насрать какую информацию как обрабатывать (иначе бы слепые были бы поголовно гениями). Просто нейрончикам там скучно сидеть без информации и поэтому они пускают свои дендриты в поисках сигналов в соседние области, а когда находят и формируют новые связи, то пытаются работать по генетически заложенной схеме. При этом получается у них крайне хреново и по большей части бесполезно, но иногда может оказаться и полезным.
Хищники жили среди животных и знают как те выглядят и вполне могут понять где у кого глаза. Некоторые животные этим пользуются и делают ложные глаза, из-за чего хищнику кажется, что жертва его палит как бы ни повернулась и сцут нападать.
Аноним 11/04/18 Срд 19:48:42  436398
>>436383
В бесконечно плоском мире, то дальность ограничена:
- прозрачностью среды(воздуха)
- разрешением и оптикой глаза
- высотой вышки: в какой-то момент угол линии направления зрения и плоскости будет стремится к нулю, то есть линия будет практически параллельна плоскости.
Без третьего пункта твоя задача аналогична дальности обзора Вселенной с телескопов в реальном мире.
Аноним 11/04/18 Срд 19:48:44  436399
>>436391
>Ведь насколько я понял, в конечном итоге все частицы, вся материя, волны и все такое представляет собой электромагнитные волны различной конфигурации, остальные частицы к электромагнетизму отношения не имеют кроме того что некоторые из них могут нести заряд. Электромагнетизм - это всего-лишь одно из четырех фундаментальных взаимодействий.
Ты неверно понял. Электромагнитные волны - это только фотоны.
>Логично предположить что все эти волны идут по некой среде.
Волна в квантмехе волна - это математический объект, описание пространственных осцилляций энергии. Для того чтобы одна форма энергии преобразовывалась в другую и обратно не обязательно иметь какую-то среду.
Аноним 11/04/18 Срд 19:50:20  436400
>>436391
Ты встал на очень опасный путь и уже даже немного продвинулся по нему.
> все такое представляет собой электромагнитные волны различной конфигурации
Нет. Электромагнитное взаимодействие - самая очевидная и весомая сила в нашем, человеческом, мире, потому что мы можем ощущать предметы, а не проваливаться в текстуры. Но без гравитации, сильного или слабого взаимодействия нас бы не было.
Впрочем, это не имеет отношения к делу. А дело в том, что вместо того, чтобы лезть в псевдонауку, которую практически все уже знакомые с темой люди покрывают хуями, почему бы тебе не попытаться разобраться в науке? И уже после этого, с каким-никаким знанием дела смотреть в сторону альтернативы или, тем паче, псевдонауки.
Аноним 11/04/18 Срд 19:51:13  436401
>>436392
>Почему в физическом вакууме постоянно и равномерно возникают и исчезают частицы, что их порождает и как?
>Являются ли эти частицы причиной неопределенности
This. Это прямое следствие неопределенности Гейзенберга.
Можно покукарекать про всякие энергии вакуума, но это все непроверяемо на данном этапе.
Аноним 11/04/18 Срд 19:53:01  436402
>>436397
>Просто нейрончикам там скучно сидеть без информации и поэтому они пускают свои дендриты в поисках сигналов в соседние области, а когда находят и формируют новые связи, то пытаются работать по генетически заложенной схеме.
Что значит "им становится скучно"? Когда это происходит и как? Можно ли повлиять на этого процесс, усилить или уменьшить "скукоту" отдельных нейронов?
Можно ли таким образом программировать мозг, оптимизировать его под конкретную задачу?
Аноним 11/04/18 Срд 19:59:39  436403
>>436392
Никто не знает. Поэтому - потому что так устроен мир. Частицы эти в общем-то ничего не порождает, они всегда как бы есть и как бы нету. Это как рябь на поверхности воды - иногда она может быть выше среднего уровня, иногда ниже, но каждый бугорок не появляется спонтанно, а несёт энергию колебаний поверхности и причина его "проявления" - это история сотен тысяч миллиардов взаимодействий воды с чем либо, что в результате порождает рябь.
Насчёт неопределённости - тут как с курицей и яйцом. В конечном счёте все настоящие частицы - часть тех же флуктуаций (т.е. все говорят, что в протоне и нейтроне по три кварка, но мало кто говорит, что их там на самом деле могут быть десятки, сотни, миллиарды - просто все остальные, кроме трёх, взаимно уничтожаются, а в сумме получается именно то, что описывается тремя кварками) и единственная разница между реальными и виртуальными частицами в том, что реальные - это совокупность виртуальных, распределённых во времени в соответствии с импульсом и положением. Опять же, если рассматривать это как волны на озере, то реальные частицы будут выглядеть как стоячая волна, образованная из той же самой ряби.
Аноним 11/04/18 Срд 20:06:14  436404
>>436402
> Что значит "им становится скучно"?
Это значит, что нейрон без какого-то минимального уровня стимуляции либо отмирает, либо переключается в режим формирования новых связей. При этом дендриты нейрона могут по содержанию различных сигнальных веществ и в зависимости от их генетического кода и эпигенетического состояния устремляться преимущественно в определённом направлении, либо наоборот просто разрастаться вширь.
>Можно ли повлиять на этого процесс
Гипотетически можно, даже многие психоделики в некоторой степени влияют, но...
> Можно ли таким образом программировать мозг
Практически нет. А точнее, это примерно равносильно тому, чтобы засунуть мозг в блендер, разбить его на клетки и собрать заново. С вероятностью 99.99999(9)% ты получишь просто суп или рагу из мозгов, как бы ни старался. Впрочем...
> оптимизировать его под конкретную задачу
можно - просто занимайся этой задачей очень долго, много, часто и мозг сам оптимизирует себя под неё.
Аноним 11/04/18 Срд 21:01:29  436412
>>436391
> Логично предположить что все эти волны идут по некой среде.
Современная физика ориентирована на построение работающих математических моделей, а не картины мира, основанной на метафорах и аналогиях. Эфир - это чуть усложнённое подобие поверхности воды, по которой бегут волны. Он был выведен философами, когда научное мировоззрение только зарождалось. Поскольку вплоть до 20века полноценной научной картины не было, учёным для объяснение физического смысла приходилось прибегать к абсолютным метафизическим субстанциям и аналогиям, подгонять под них/их под математику и эксперименты. У эфира изначально были проблемы с подгонкой под матан, и, не понадобись он Максвеллу, от него отказались бы и раньше. Главное его достоинство - образная простота и наглядность, быстро исчезали в любой тогдашней теории.
В современной физике у метафоры "мир подобен волнующейся поверхности океана" нет конкретного матанового содержания, потому и эфира нет.
Аноним 12/04/18 Чтв 04:56:07  436442
Как бактерии постоянно прокачивают резисты к новым лекарством? Что за ебучее читерство? Они и к ядерке, и к сверхновой иммун сделать могут? Хотя лучше спросить - как они при этом сохраняют свою сущность, не становятся совсем другими, так что гриппы разных поколений похожи по симптомам. Нельзя сделать лекарство, которое полностью забанит её генотип, потребует его полнейшей глобальной перестройки, так что проще заново создать?
Аноним 12/04/18 Чтв 04:58:07  436443
>>436442
> бактерии
> грипп
бля, ну и вирусы тоже, несуть
Аноним 12/04/18 Чтв 06:53:33  436445
>>436442
>Как бактерии постоянно прокачивают резисты к новым лекарством? Что за ебучее читерство?
Эволюция, чем проще организм тем быстрее она идет. Эволюцию одноклеточных можно наблюдать в риалтайм, для млекопитающих же этот процесс занимает миллионы лет.
>Они и к ядерке, и к сверхновой иммун сделать могут?
Некоторые вирусы способны жить при высокой радиации из-за ускоренного деления, например.
>Хотя лучше спросить - как они при этом сохраняют свою сущность, не становятся совсем другими, так что гриппы разных поколений похожи по симптомам.
Не настолько радикальные изменения.
>Нельзя сделать лекарство, которое полностью забанит её генотип, потребует его полнейшей глобальной перестройки, так что проще заново создать?
Теоретически - да, но тут вопрос еще и в безопасности для других клеток, например наших. Суть лекарства в том чтобы не трогать наши клетки а выпиливать грипп.

Аноним 12/04/18 Чтв 13:36:05  436453
>>436442
Во-первых, у них очень большая вариабельность, так что какие-то из бактерий могут быть резистентны к антибиотикам (напоминаю, что к антибиотикам восприимчивы только бактерии, а не вирусы), которых мы вообще ещё не придумали. Во-вторых, у них есть клеточные каскады, которые повышают мутагенез в неблагоприятной среде, поэтому лучший способ сделать резистентные бактерии - пить антибиотики в небольших дозах и не пропивать курс до конца.
В-третьих, ДНК бактерий находится не как у нас в хромосомах, а в виде небольших колечек, которыми они могут и обмениваются между собой, поэтому бактерии могут не только сами обрести резистентность, но ещё и подарить её другим бактериям (своему или даже другому виду). К радиации они примерно так же восприимчивы, как и многоклеточные, и у них примерно так же, как у тихоходок, могут выработся механизмы защиты и репарации ДНК при повреждении радиацией, что немного повышает их устойчивость, но это всё всё равно не так уж эффективно и от больших доз не спасёт.
Лекарство сделать можно, но сложно. Наиболее эффективный метод в этом плане - массовая атака на все системы клеток, коктейль из кучи разных антибиотиков. Выработать резистентность к одному из них бактерии могут легко и непринуждённо, но сразу к десятку - без шансов. Но в таком случае понадобится строгий контрль, чтобы убедится, что этот коктейл прописывается в полном составе.
Это всё относится к бактериям. Вирусы - это совсем другое. Им похуй на антибиотики, на бактерий и даже на нас. Поэтому у нас нет противовирусных препаратов даже для столь старых друзей как папилломавирус или герпес и отчасти поэтому вакцина против ВИЧ так сложно даётся. Вирусы тоже очень вариабельны и мутагенны, но в общем-то это всё. Реальная проблема с ними в том, что в отличии от бактерий они убер-маленькие и не живые. Поэтому у нас крайне мало механизмов, атакующих непосредственно вирусы - разве что антитела, блокирующие вирусные капсиды. С бактериями можно ожидать, что она сама проглотит всё, что нужно и там уже внутри вещества могут действовать на что угодно: на ДНК, его считывание, на репликацию, биосинтез белков, разрыв мембран и прочее и прочее. С вирусами - по большей части это только залить организм каким-то веществом, способным запечатать капсид снаружи. Ещё можно проникать в клетки потенциальных жертв и там что-то настраивать на противодействие вирусу, но это охеретительно сложно, т.к. нужно при этом ничего не сломать и не оказаться хуже, чем сам вирус.
Аноним 12/04/18 Чтв 15:07:23  436456
Вопрос. Куда девается энергия при деструктивной интерференции света?
Аноним 12/04/18 Чтв 16:13:48  436459
>>436456
Никуда не девается - где есть деструктивная интерференция есть и усиливающая.
Аноним 12/04/18 Чтв 20:40:41  436463
Посоветуйте почитать что-нибудь простое про мутагенные свойства металлов.
Аноним 12/04/18 Чтв 22:29:33  436469
image.png (56Кб, 1134x172)
Да пиздец, я уже нашел правильный ответ, но вот, блять, прочитайте это постановку, ведь так и читается:

пусть x - ширина
длина на 5 больше, тогда x+5 - длина
длина на 5 меньше высоты, то (x+5)-5 высота, т.е. x

Серьезно, что за хуета, блджад.
Аноним 12/04/18 Чтв 23:23:07  436475
>>436469
Ты в глаза долбишься или как?
Ширина = длина-5
Высота = длина+5
y=x(x-5)*(x+5)
y=244
Дальше сам.
Аноним 13/04/18 Птн 14:40:08  436487
Посоветуйте хороший онлайн-курс общей физики, вроде того, что читают химикам и инженерам на первом курсе. Всю Курсеру перерыл - не нашёл такого, на "Открытом образовании" тоже нет.
Аноним 13/04/18 Птн 15:48:00  436488
>>436456
На картинке же видно, что в "detector"
Аноним 13/04/18 Птн 18:37:29  436490
Поясните ньюфагу: что такое сети-антисети треды?
Аноним 13/04/18 Птн 18:39:22  436491
>>436490
О проекте SETI. Унылые срачики о чужих деньгах.
Аноним 13/04/18 Птн 18:44:39  436494
>>436491
Ну так это ж тред добровольных вычислений, он до сих пор есть.
Аноним 13/04/18 Птн 21:20:55  436498
>>436453
Ну вообще говоря с вирусами тоже есть варианты. Самое узкое место - это репликация вирусного генома; если у нас ДНК-содержащий вирус, то репликация ДНК не будет идти при помощи ферментов клетки-хозяина, чтобы они включились, нужно овердофига условий, которые соблюдаются только приделении. Тот же герпес носит свою ДНК-завиимую ДНК-полимеразу с собой она отличается от человеческой и её можно ингибировать ацикловиром. Если вирус РНК-содержащий, то он вынужден таскать с собой РНК-зависимую ДНК-полимеразу, которой учеловера даже близко нет, и, например, у ВИЧа её ингибируют AZT. Но все эти ферменты довольно сильно разнятся от вируса к вирусу, к каким-тоудалось быстро и просто найти достаточно эффективные ингибиторы, к другим нет.
Аноним 13/04/18 Птн 21:35:22  436499
>>436490
Был тут некоторое время назад упоротый, который засрал раздел кучей тредов, в которых яростно доказывал, что его точка зрения (Земля - уникальна во вселенной, внеземная жизнь невозможна, глобальное потепление это мировой заговор, истребление мегафауны - миф и т.д.) является не только единственно верной, но и единственно возможной научной версией, а любые сомнения в его наборе постулатов это неорелигиозный псевдонаучный догматический фанатизм.
Пытался поддерживать три своих тематических треда, и активно срал в соседних. Плюс даже за пределами двачика засветился.
Аноним 13/04/18 Птн 21:36:53  436500
>>436499
>но и единственно возможной научной версией, а любые сомнения в его наборе постулатов это неорелигиозный псевдонаучный догматический фанатизм.
Мда, саентач не меняется, только как-то это всё уже приелось.
Аноним 13/04/18 Птн 21:39:22  436501
>>436499
Но он был прав, хуя вам так припекло, я не понимаю.
Аноним 13/04/18 Птн 21:42:11  436502
>>436501
Астробиологи по всему миру не могут прийти к консенсусу по этому вопросу.
Но рандомный хуй с сосача конечно правее их всех.
Можешь конечно покукарекать про argumentum ad verecundiam, только вот по отношению к научному консенсусу это кукареканье не работает
Аноним 13/04/18 Птн 21:44:07  436503
Господа, из /b/ послали нахуй сколько (минимально для мелочи) оборотов должно быть у той-самой центрифуги (2.2к/мин хватит?)?

мимоодноклеточное
Аноним 13/04/18 Птн 21:57:13  436504
>>436499
понятно, ему в зюг, но причём здесь сети? он форсил розетту?
Аноним 13/04/18 Птн 22:00:04  436505
>>436501
>>436499
>Земля - уникальна во вселенной
хуй знает, в любом случае, напланетяне до нас не долетят и мы до них тоже, ибо очень далеко
>внеземная жизнь невозможна
то же самое
>глобальное потепление это мировой заговор
а тут он прав. то есть, оно есть, но не по вине людей, а это естественный процесс
>истребление мегафауны - миф
а вот здесь он не прав
Аноним 13/04/18 Птн 22:02:14  436506
>>436505
>а тут он прав. то есть, оно есть, но не по вине людей, а это естественный процесс
И у тебя конечно же есть просто железобетонные пруфы этого замечательного вскукарека идущего вопреки имеющегося консенсуса, продемонстрировав которые ты враз сможешь убедить климатологов всей планеты изменить собственное мнение?
Аноним 13/04/18 Птн 22:50:04  436507
>>436442
Лол, чем отличаются бактерии прокариоты, доядерные от остальных археи, грибы, растения, животные - эукариоты, ядерные?
У ядерных ДНК находится в виде хромосом, то есть ДНК окружено вспомогательными белками, запрятано в оболочку, это и есть хромосома. Все хромосомы в 2-х экземплярах, то есть, дублируют друг друга, на случай, если какая-то из них случайно мутирует. Ну и наконец, хромосомы находятся в ядре, окружены специальной оболочкой от агрессивной цитоплазмы.
У бактерий же ничего этого нет. Молекула ДНК в единственном экземпляре плавает по цитоплазме не защищённая ничем и атакуется всяким говном, по этому очень часто мутирует.
Аноним 13/04/18 Птн 23:04:54  436511
>>436507
Вот уж лучше б молчал, ей богу. В каждом предложении по несколько ошибок.
У архей нет ядра и они не эукариоты.
У эукариот ДНК собирается в хромосомы только в перед делением и после репликации, до того ДНК находится в виде хроматина - размазанной каши из нитей ДНК и белков, регулирующих считывание.
Хромосома - это не то, что в оболочке, это плотно упакованная единичная нить ДНК, сформированная в третичную структуру.
Не все хромосомы в двух экземплярах (как минимум мужская Y-хромосома у большинства высших многоклеточных - единична; как максимум, существуют гаплоидные, диплоидные, триплоидные, тетраплоидные, пентаплоидные клетки, дальше уже просто полиплоидные).
Оболочка ядра не столько от цитоплазмы защищает, сколько от вообще всего, вирусов в том числе.
У бактерий ДНК далеко не в единственном экземпляре - каждое кольцо ДНК у них содержится в больших количествах и поэтому при делении они вообще не заморачиваются с тем, чтобы доставлять фрагменты ДНК в разные концы клетки - и так сколько-нибудь да попадёт в обе клетки.
И у бактерий есть прекрасные механизмы защиты и репарации ДНК, позволяющие нужным генам быть либо высоковариабельными, либо наоборот консервативными.
Аноним 13/04/18 Птн 23:56:54  436516
>>435409 (OP)
Как запилить очередную имиджборду?
Нужна ли там вообще модерация?
А что если её повесить её в TOR'e?
Аноним 14/04/18 Суб 00:25:43  436518
>>436516
/web/ /pr/ /crypt/
Аноним 14/04/18 Суб 01:00:30  436521
Почему микробиология, биохимия настолько ебически сложная по сравнению с обычной? Обычной, значит, только слабоумные и училки сейчас занимаются?
Аноним 14/04/18 Суб 01:06:42  436522
>>436521
А что ты считаешь обычной? Микробиология - это раздел биологии, биохимия - раздел химии. Это значит, что биология включает микробиологию и все её сложности, а химия включает биохимию.
Любая наука в любом направлении ебически сложная, потому что ею занимаются минимум десятки весьма неглупых людей.
Разве что.. сейчас... я смогу... т...т-т-т... теология, блять... Но "наукой" она является только в атрофировавшихся из-за ожирения мозгах наших чиновников.
Аноним 14/04/18 Суб 08:28:04  436526
chem1.png (35Кб, 510x395)
chem2.png (171Кб, 640x480)
>>436521
> биохимия настолько ебически сложная
Первый пик - обычная молекула в химии
Второй пик - обычная молекула в биохимии
Вопросы?
Аноним 14/04/18 Суб 09:58:49  436527
>>436521
Что ты представляешь себе под "обычной"?
мимо-химик
Аноним 14/04/18 Суб 11:35:58  436533
>>436521
Лол, химия бывает неорганическая, органическая, аналитическая, физическая, коллоидная. Это в любых химических ВУЗах изучают. Ну и ещё как бонус биохимия, но это уже специализация.
Аноним 14/04/18 Суб 11:37:30  436534
>>436516
Было бы круто. Был раньше 2ch-ru.net у них было зеркало в ТОРе, но админа Ская это заебала. Работа неблагодарная. Деньги из своего кармана, на борде тусуются дегенераты и развлекаются за твой счёт.
Аноним 14/04/18 Суб 11:38:59  436535
>>436511
>Вот уж лучше б молчал, ей богу
Нет уж, хорошо, что высказался, потому что благодаря Тебе я узнал что-то новое :3
Аноним 14/04/18 Суб 12:02:42  436536
>>436534
Имхо, другие доски не нужны, и другие разделы кроме наукача тоже не нужны.
Аноним 14/04/18 Суб 12:08:08  436537
image.png (54Кб, 217x552)
>>436535
Можешь здесь ещё глянуть:
>>435780
>>435784
>>435785
>>435851
Аноним 14/04/18 Суб 16:48:50  436547
>>436536
Там был ламповый /бб/ где тусовались наркоманы и вели научные дискуссии про рецепторы и нейромедиаторы :3
Аноним 14/04/18 Суб 20:57:24  436562
>>436547
Недавно видел одного странного типа в общ.транспорте, стоял и безостановочно крутился из стороны в сторону в каком-то своём ритме, тоже наверное рецепторы изучает.
Аноним 14/04/18 Суб 21:34:12  436564
Представьте себе полый шар, внутренняя поверхность которого - идеальное зеркало. Можно пропускать в него лучи света так, что они оказываются в ловушке. По сути таким образом можно сколь угодно долго хранить энергию. Как много энергии сможет уместить такой шар? Как долго можно запускать в него фотоны?
Аноним 14/04/18 Суб 21:45:42  436566
>>436564
https://www.reddit.com/r/askscience/comments/47j38a/is_it_possible_to_trap_light_in_a_hollow_ball/
https://www.reddit.com/r/askscience/comments/4anry6/does_light_lose_energy_when_reflected/
https://www.reddit.com/r/askscience/comments/h1huy/could_you_trap_light_inside_a_spherical_perfectly/
Аноним 14/04/18 Суб 21:46:21  436567
>>436564
идеальных зеркал не бывает, свет потеряется, шар перегреется. как ты будешь свет превращать в электричество? КПД низкий будет.
Аноним 14/04/18 Суб 22:20:01  436568
>>436567
Я знаю, что их не бывает. Просто хотел представить такую ситуацию. Если было бы можно регулировать отражающую способность такого зеркала, то можно было бы влиять на теплоотдачу и нагревать какой нибудь теплоноситель. Тут только вопрос в том, как много энергии такой шар бы уместил.
Аноним 14/04/18 Суб 22:40:31  436569
>>436568
Будет какой-то предел "плотности светового потока внутри", когда высвечиваться из отверстия для засвечивания будет больше, чем будет возможно через него/в него засветить.
И сильно раньше этого будет высвечиваться обратно значительная часть того что засвечивает внутрь.
КМК
Хотя.. Может возможно сделать что-то, что будет давать зайти, но не давать выходить вообще? Что-то типа лабиринта из зеркал намутить..
Аноним 14/04/18 Суб 22:49:27  436570
>>436569
>>436564
Если практически, то тупо всё будет превращаться в тепло, т.к. идеальных зеркал не существует.
Если с одной стороны всегда отражает, а с другой всегда пропускает - это демон Максвелла, так что теоретически невозможно. Если бы было возможно - тогда пожалуйста, можно сколько угодн онарушать энтропию и делать всё что угодно. Можно даже создать ЧД, когда накопится достаточно световой энергии.
Если есть дырочка, в которую пропускает (и таки гипотетическое идеальное зеркало, а не что-то реальное) - то свет, проходящий в дырочку, уравновесится со светом, выходящим из дырочки.
Аноним 14/04/18 Суб 23:03:21  436572
>>436570
> Если практически, то тупо всё будет превращаться в тепло, т.к. идеальных зеркал не существует.
> Можно даже создать ЧД, когда накопится достаточно световой энергии.
О! Вот ЧД и будем использовать в качестве аккумулятора света. Правда неясно как оттуда потом энергию достать
Аноним 14/04/18 Суб 23:42:21  436575
>>436570
> Можно даже создать ЧД, когда накопится достаточно световой энергии.
Ок, допустим это не идеальное но одностороннее зеркало( в одну сторону пропускает, с другой отражает), накопившийся свет будет нагревать сферу и она сама будет светиться но в уже другом спектре. будет ли такая конструкция демоном Максвелла?

Или вот люминофор что нарушает правило Стокса(переизлученный пектр ниже поглощенного), он может поглощать тепловое излучение(в т.ч. и то что внутри него) и излучать в виде видимого света.

Или если люминофор обычный, он может переизлучать тепло в виде радиоволн?

- мимодругойанон
Аноним 14/04/18 Суб 23:44:27  436576
>>436572
Доставать и не надо, ЧД это как солнце только наоборот: кидай туда любое излучение или материю, получай гравитацию или охлаждение без затрат энергии -> получай полезную энергию.
Аноним 14/04/18 Суб 23:48:55  436577
>>436572
Хм, ну, если твоя сфера была порядка 1м радиуса, то всего-лишь надо подождать... не знаю... несколько сотен квадрилионов лет? В общем, рано или поздно она сама испарится и преимущественно в виде элекромагнитного излучения.
>>436575
Разумеется. Уже только потому, что пропускает только с одной стороны. И пусть конструкция диодов не вводит тебя в заблуждение - недостающая энергия для туннелирования электронов только в одном направлении берётся с третьего контакта. А такое зеркало - это типичный демон максвелла, который пропускает нечто только в одну сторону.
Но если ты что-то нагреваешь светом и оно переизлучает на более низком спектре - тогда норм, такое везде и всегда происходит. Люминофоры, излучающие в более низком спектре - не исключение, нет ничего особенного в том, чтобы переизлучить свет меньшей энергии. Намного сложнее собрать энергию фотонов и излучить в спектре с большей энергией. Не уверен, быть может, такое и реализуемо, но я о таких люминофорах не слышал.
Аноним 15/04/18 Вск 00:23:07  436583
>>436577
>И пусть конструкция диодов не вводит тебя в заблуждение - недостающая энергия для туннелирования электронов только в одном направлении берётся с третьего контакта.
Какого еще третьего контакта? Ты с транзисторами не перепутал?

>А такое зеркало - это типичный демон максвелла, который пропускает нечто только в одну сторону.
А если только в определенном спектре(дихроичное зеркало или как-о так)? Скажем, в тепловом? Тогда при определенной температуре тепловая энергия будет преобразовываться в излучение другой частоты, а тело охлаждаться.

>Но если ты что-то нагреваешь светом и оно переизлучает на более низком спектре
А если не нагреваю а нагревает среда? Тогда тело охлаждается? Ведь более низкий спектр = более низкая температура.

>Намного сложнее собрать энергию фотонов и излучить в спектре с большей энергией.
Там суть в том что прилетает 2 фотона малой энергии, а излучается один большой. При чем вторым фотоном может быть "внутренний" фотон теплового излучения.
В итоге тело переизлучает в более высоком спектре, но при этом еще и охлаждается.
Аноним 15/04/18 Вск 00:33:54  436586
Посмотрел как сейчас выглядят первые модели квантовых компьютеров. Эта бандура ещё больше ЭВМ. Там нужно множество слоёв защиты и дикое охлаждение. Видать в будущем будет только один способ сделать их открытыми для любого желающего. Этим гробом будут обладать разные компании. А ты будешь пользоваться его вычислительными мощностями при помощи облачных технологий. Винду специальную под это напишут. Оплачиваешь подписку и пользуешься. Если соединение потеряется по какой-то причины, твой комп переходит на вычислительные мощности старого железа. Будет такое работать пацаны?
Аноним 15/04/18 Вск 00:43:47  436587
>>436586
Так уже говорили о бессмысленности персональных компьютеров. Погоди, вот откроют высокотемпературные сверхпроводники, и все сдвинется с места.
Аноним 15/04/18 Вск 00:47:25  436588
>>436587
Я честно говоря не думаю, что наши законы физики позволят создать малоэнергозатратные системы охлаждения. Квантовый проц должен быть полностью изолирован от сторонних воздействий и максимально возможно охлаждён. Я не верю, что эта штука когда-то уменьшится до размеров наших пк. Она даже до размеров серверного шкафа не дойдёт. Это всегда будет гроб.
Аноним 15/04/18 Вск 02:22:42  436591
image.png (239Кб, 634x279)
>>436588
Аноним 15/04/18 Вск 02:24:54  436592
>>436588
Нээ, здесь сложно сказать. Уж на что я скептик, но в принципе нельзя сказать, что невозможны квантовые высокотемпературные эффекты, если уж даже фотосинтез работает с использованием квантовых эффектов. К тому же, уже есть технология атомарного холодильника, буквально - один атом, который переносит тепло с одного места в другое. В этой сфере он, впрочем, едва ли применим (да и вообще кроме лаботарории эта технология сильно не факт что куда-то выберется).
Однако гипотетически вряд ли есть что-то невозможное в создании системы, которая бы усиливала квантовые эффекты до масштабов органических молекул и дальше позволяла бы сцепить такие молекулы и тем самым организовать квантовые вычислители. Да, если это возможно, то потребует ещё уйму научных прорывов, и тем не менее нельзя просто так отрицать эту возможность.
Аноним 15/04/18 Вск 02:28:55  436593
>>436588
К тому же не забывай, что у нас нет нормальной теории, объясняющей сверпроводимость. БКШ - это чуть лучше, чем подгоночный костыль, который, к тому же, практически не работает для высокотемпературных сверхпроводников. Если создать достаточно точную теорию сверхпроводимости - возможно, лимит температуры будет сильно выше комнатной. А в таком случае и вовсе никаких проблем с созданием квантовых компьютеров не будет.
Аноним 15/04/18 Вск 02:34:45  436594
Другой момент, что квантовые вычисления в прикладных задачах нахуй не нужны сейчас и в ближайшие несколько лет - точно. Мы ничего толком не умеем ими вычислять. Может быть если придумают алгоритмы сжатия или ускорения каких-нибудь ходовых векторных рассчётов - тогда да, ещё хоть какой-то прок есть. А пока что квантовые технологии - это по большей части хайп, чем реальная практическая польза. Да и наиболее вероятная теоретическая польза от них тоже по большей части лежит вне задач обычных юзерских компов - это в первую очередь моделирование физических процессов.
Аноним 15/04/18 Вск 09:11:58  436600
>>436575
>но одностороннее зеркало
Не хочу тебя разочаровывать, но односторонних зеркал не существует. То, что выглядит как односторонние заркала, на деле является просто полупрозрачным отражающим материалом, а кажущийся эффект возникает из за разницы в освещении с разных сторон.
Аноним 15/04/18 Вск 10:32:37  436601
>>436570
>демон Максвелла, так что теоретически невозможно
Возможно и уже не раз реализовано экспериментально. Даже целый вид двигателей изобрели Maxwell's demon engine, которые тапло вырабатывают.

Обновляй знания, наука не стоит на месте.
Аноним 15/04/18 Вск 12:03:37  436606
LADEE.jpg (406Кб, 945x680)
LADEE2.jpg (66Кб, 617x416)
>>436588
>не думаю, что наши законы физики позволят создать малоэнергозатратные системы охлаждения
Можно сделать пачку таких гробов и вынести на обратную сторону Луны, - пару десятку градусов можно обычными системами охлаждения накинуть если что, должно хватить, а сигналы передавать на Землю лазерами через систему спутниковых зеркал, ну будет небольшая задержка, но в потоке не критично.
Аноним 15/04/18 Вск 12:13:33  436608
>>436606
>вынести на обратную сторону Луны,
лалка, там не холодно, там вакуум
Аноним 15/04/18 Вск 12:22:38  436612
Почему говорят что для достижения световой скорости телами/частицами с ненулевой массой нужно бесконечное количество энергии?
Всегда принимал это как должное и даже не задумывался, а недавно вот всплыло в голове что ведь формула е=мс^2, а c совсем не бесконечная, а вплоне себе конечная и не то что бы очень большая лол по крайней мере результат, возведёния её в квадрат и умножения получившегося на массу самой лёгкой известной имеющей массу частицы не будет же больше теоретической потенциальной энергии запасённой в веществе обозримой части вселенной?

и почему е=мс^2, но е=(mv^2)/2?
Аноним 15/04/18 Вск 12:27:14  436614
>>436608
Это кстати ничего, но затупил да, что это к нам она одной стороной, к Солнцу-то всеми.
Аноним 15/04/18 Вск 12:30:08  436615
>>436612
e=mc^2 - это уравнение энергии от массы покоя. Каким боком ты тут привязал скорость - я без понятия, но у тебя своя альтернативная логика.
А бесконечное количество, потому что чем больше разгоняешь частицу, тем медленнее движется её локальное время и тем меньше она взаимодействует с тем, что её разгоняет, поэтому нужно всё больше усилий прикладывать, чтобы разогнать её ещё немного.
Аноним 15/04/18 Вск 12:35:41  436616
>>436612
E=mc^2 это для покоя.
Для движения получается E=mc2/sqrt(1-v2/c2)
Аноним 15/04/18 Вск 12:41:06  436617
>>436615
>e=mc^2 - это уравнение энергии от массы покоя. Каким боком ты тут привязал скорость - я без понятия
в эту формулу подставляют СКОРОСТЬ света же
>>436616
спс
Аноним 15/04/18 Вск 14:37:32  436621
>>436606
>Можно сделать пачку таких гробов и вынести на обратную сторону Луны, - пару десятку градусов можно обычными системами охлаждения накинуть если что, должно хватить, а сигналы передавать на Землю лазерами через систему спутниковых зеркал, ну будет небольшая задержка, но в потоке не критично.
Ага, то-то в Spitzer бак с жидким гелием добавляли чтобы оборудование охлаждать. А как гелий кончился - так он сразу до 10-20 K нагрелся.
Аноним 15/04/18 Вск 16:34:22  436625
>>436621
Ох ладно, пойду уроки делать.
Аноним 15/04/18 Вск 16:37:40  436626
>>436606
Слишком геморно. И квантовый комп слишком нестабильный. За ним нужен глаз да глаз. Множество слоёв защиты экранирующих чип от внешних воздействий. В квантовой физике даже наблюдатель портит частоту эксперимента, ссаные фотоны могут всё испортить. Без защиты размером с комнату всё полетит даже от того, что ты пёрнешь в соседней комнате. Мелкие квантовые компы это уже какая-то фантастика. Там нужно защитное поле которое будет держать его всегда горизонтально и не пропускать внутрь никакие излучения.
Аноним 15/04/18 Вск 21:45:55  436633
maxresdefault.jpg (165Кб, 1280x720)
Еще с зимы заметил такую вещь - когда солнце садится, температура падает, а потом в 7-9 вечера снова повышается, после чего снова падает. Действительно имеет место такое явление и как оно объясняется?
Аноним 15/04/18 Вск 22:24:17  436634
Чем наставивание дешёвого бухла на дубовых опилках реально хуже полноценной бочки? Ускорение реакций очень неравномерное?
Аноним 15/04/18 Вск 22:36:40  436636
>>436634
Понтов меньше. Каждый Вася может в деревнях самогон гнать хоть на всей таблице Менделеева, а вот 9000-летний виски из другого конца планеты уникален.

Аноним 15/04/18 Вск 23:00:23  436637
Можно ли пердежом и поносом передвигаться в космосе?
Аноним 15/04/18 Вск 23:06:05  436638
Можно ли генетически модифицировать человека так, чтобы вместо спермы у него был бы напал, поджигаемый еще по пути наружу Ну хоть без поджигания
Аноним 15/04/18 Вск 23:07:18  436639
Какой будет оргазм у человека, которому подрочили со сверхсветовой скоростью?
Аноним 15/04/18 Вск 23:08:43  436640
Можно ли создать сцп 682 ирл с помощью генной инженерии?
Аноним 15/04/18 Вск 23:40:08  436643
>>436637 да
>>436638 нет
>>436640 нет
Аноним 16/04/18 Пнд 06:11:29  436647
Может ли звезда быть с температурой в 0 К?
Аноним 16/04/18 Пнд 06:52:56  436649
>>436647
Ничто не может быть с температурой в 0К
Аноним 16/04/18 Пнд 08:07:30  436650
>>436639
Термоядерный.
Аноним 16/04/18 Пнд 10:29:22  436653
Вопрос про остывание нейтронных звёзд.
Везде в инете пишут только про механизм этого, то что в начале этот процесс происходит за счёт нейтрино из центра, после-с поверхности световым излучением. Но нигде не нашёл описания того что происходит с самой нейтронной звездой при этом. Особенно интересно остывание прям вообще, или около того.
Вот белые карлики могут остыть практически полностью и превратиться просто в шарик холодного железа/углерода/кремния.
А нейтронные звёзды что?
Аноним 16/04/18 Пнд 11:10:07  436656
>>436653
Кстати двачну вопрос, разве излучение АЧТ не требует наличия движущихся зарядов? Что там в этой вашей нейтронной дыре излучает?
Аноним 16/04/18 Пнд 11:11:58  436657
>>436649
Почему? Это же просто состояние покоя, у какого-нибудь атома вполне себе температура 0.
Аноним 16/04/18 Пнд 11:15:34  436658
>>436657
Атом не может находиться в покое с импульсом 0 из-за неопределенности Гейзенберга. Кванты - это тебе не хуйня из под ногтей.
Аноним 16/04/18 Пнд 11:22:17  436659
>>436657
Температура это статистическая вещь, для атома никто не будет говорить о температуре
>>436658
Температура 0, это когда тело находится в низшем квантовом состоянии. Для атома это реально, но см. выше.
Аноним 16/04/18 Пнд 11:26:09  436660
>>436659
У температуры дохуя разных определений.
Да и в случае статистического определения, единственный семпл - это тоже статистика. И у единственного атома импульс нулю не равен.
Аноним 16/04/18 Пнд 11:38:10  436661
>>436660
>И у единственного атома импульс нулю не равен.
В определении температуры из квантовой статистики, там поебать на импульсы, главное какие уровни заселены.
Аноним 16/04/18 Пнд 11:42:11  436662
>>436661
Т.е. у ионизированного атомного ядра или даже нейтрона нет температуры?
Аноним 16/04/18 Пнд 12:18:34  436664
>>436662
Или если ты про уровни самого нейтрона, то у него спектр непрерывный.
Аноним 16/04/18 Пнд 12:47:09  436666
>>436653
>>436656
У неё кора из обычных элементов, по большей части - из железа. Так что она спокойно остывает, при этом её спектр - типично тепловой. Так что когда через многие миллиарды миллиардов миллиардов и т.д. лет она остынет до чуть более близких нам температур, она будет выглядеть оранжевой, затем красной, затем бурой, затем чёрной.
Аноним 16/04/18 Пнд 12:54:22  436667
>>436666
Такую простоту я и сам мог придумать, но кажется мне что нихуя всё так не будет, учитывая её содержимое, и то что она на грани превращения в чд.
Аноним 16/04/18 Пнд 13:04:40  436668
>>436667
Чувак, я если не знаю - не отвечаю.
Учитывая её содержимое разница с белыми карликами у НЗ в том, что НЗ меньше и остывает на очень много порядков дольше.
И они не на грани. Могли бы стать ЧД - стала бы ей. Потеря тепловой энергии только отдаляет это.
Аноним 16/04/18 Пнд 13:14:03  436669
>>436668
Если взять ложечку нейтронного вещества, оно постепенно распадётся (ну тип нейтрон в свободном состоянии нестабилен)?
Аноним 16/04/18 Пнд 13:14:58  436670
>>436669
Период полураспада 15 мин.
Аноним 16/04/18 Пнд 13:18:49  436671
>>436670
Почему тогда нейтронная звезда каждые 15 минут не теряет половину нейтронов?
Аноним 16/04/18 Пнд 13:20:50  436672
>>436671
Потому что электрон и протон на которые распадается нейтрон, никуда съебать не могут и сливается обратно в нейтрон.
Аноним 16/04/18 Пнд 16:50:21  436682
>>436105
ты путаешь ТОК и ПОЛЕ
ток - движение заряженных частиц
ЭМ поле - вещь сложная, тк есть куча разных моделей, первая(школьная/теория поля) - поле это просто математическая хуйня, в каждой точке пространства написано какое-то число(вектор), написаны они в соответсвии с законами Максвелла (постулируетя распределение токов и зарядов, вычисляется конфигурация полей), вторая(квантовая теория поля) - ёбань из ёбаней, но примерно она говорит, что из вакуума рождаются виртуальные частицы и взаимодействуют с реальными
Аноним 16/04/18 Пнд 16:54:13  436683
>>436650
Сначала пизданёт сверхновая,
потом пизданёт гиперновая,
а затем вся малафья схлопнется в сверхмассивную супердыру
, и ещё раз пизданёт ещё одним Большим взрывом.
Аноним 16/04/18 Пнд 17:34:21  436686
>>436683
А, кстати, да... Поле хиггса же метастабильно и ему только и нужен хороший пинок, чтобы туннелировать в состояние с меньшей энергией. Сверхсветовая дрочка подразумевает нарушение всех законов физики и бесконечную энергию, и следовательно поле Хиггса может хоть куда уебать. И когда оно порвётся в одном месте - разрыв начнёт расширяться по всей вселенной со скоростью света.
Аноним 16/04/18 Пнд 23:52:56  436705
2018-04-16-2152[...].png (196Кб, 658x699)
>>436686
У меня от тебя ПУЗЫРЬ СМЕРТИ
Аноним 17/04/18 Втр 00:12:27  436706
>>436667
Какое-то время будет остывать, но несильно, потому что какое-никакое вещество на неё падает постоянно, неебически ускоряясь и подогревая нз.
Аноним 17/04/18 Втр 00:17:32  436707
>>436706
Так почему остывает-то? Что излучает АЧТ если электронов нет?
Аноним 17/04/18 Втр 01:28:19  436711
>>436707
Не тупи. Кора, блядь, из обычных элементов с обычными блять электронами. При этом устаканивается баланс с ядром из нейтронов или что там глубже нейтронов есть - всё это замедляется, упорядочивается, всё свободное пространство и уровни выравниваются - это и есть остывание. И жертвой всех процессов в любом случае становится кора, которая и выводит тепло в виде излучения. Плюс в ядре легко может быть избыток электронов и тогда НЗ - это магнетар и он дополнительно остывает за счёт ЭМ взаимодействия.
Аноним 17/04/18 Втр 01:37:47  436713
>>436711
Т.е. по сути излучает тонкая хуйня вокруг НЗ а всё остальное тепло передается конвекцией? ох дохуя оно остывать будет
Аноним 17/04/18 Втр 01:45:40  436714
>>436713
Не совсем конвекцией какой мы её знаем, но да. И да, она охуенно долго остывает. И не только потому что там квантовомеханические препоны на пути вывода тепла, но в первую очередь потому что при всей её массе и энергии она охренительно маленькая и охренительно же горячая.
Аноним 17/04/18 Втр 04:36:55  436715
А в газировках ЦО2 растворён почти в максимальной концентрации или можно намного газированнее-шипучее сделать? И что мешает?
Аноним 17/04/18 Втр 07:32:11  436716
Почему e^(i*2pi)=1?
Аноним 17/04/18 Втр 07:56:50  436718
>>436671
>Почему тогда нейтронная звезда каждые 15 минут не теряет половину нейтронов?
Потому что 15 минут, это в вакууме, без стабилизации другими частицами.
Аноним 17/04/18 Втр 07:57:12  436719
>>436716
Потому что соотношение Эйлера.
Аноним 17/04/18 Втр 08:59:00  436720
>>436711
>из обычных элементов с обычными блять электронами.
А НА ВИКИПЕДИИ написано что нихуя там нет обычных элементов в коре
Аноним 17/04/18 Втр 11:06:21  436724
>>436720
Не, там правда есть тонкий слой вокруг нейтронного вещества из остатков говна которое бомбануло. Оно там еще лопается иногда, случаются ЗВЕЗДОТРЯСЕНИЯ и периодичность у пульсаров из-за этого сбивается.
check Ширер "Звезды: рождение, жизнь и смерть"
Аноним 17/04/18 Втр 11:08:22  436725
>>436724
>Шкловскиый
Быстрофикс.
17/04/18 Втр 11:18:49  436726
Что такое коммунизм по Марксу, просто вдруг понял что всё не может быть бесплатным и доступным даже в теории.
Аноним 17/04/18 Втр 11:29:10  436727
>>436726
>Что такое коммунизм по Марксу
Вся теория Маркса - псевдонаука основанная на ложных постулатах и манялогике Гегеля.
нет правда, Ленин марксизм называл наукой
Подробности тут: http://baguzin.ru/wp/wp-content/uploads/2014/08/%D0%9A%D0%B0%D1%80%D0%BB-%D0%9F%D0%BE%D0%BF%D0%BF%D0%B5%D1%80.-%D0%A7%D1%82%D0%BE-%D1%82%D0%B0%D0%BA%D0%BE%D0%B5-%D0%B4%D0%B8%D0%B0%D0%BB%D0%B5%D0%BA%D1%82%D0%B8%D0%BA%D0%B0.pdf
Аноним 17/04/18 Втр 11:37:34  436728
image.png (29Кб, 1356x82)
>>436720
Два хуя этому пидарасу.
Аноним 17/04/18 Втр 11:42:41  436729
>>436726
Там суть не столько в доступности сколько в равном распределении. Коммунизм - это идея о том чтобы высшей целью поставить реализацию потребностей каждого(насколько это возможно) в равной степени, независимо от вклада. Так же это отмена рынка, обобществление ресурсов и планирование.
Экономике это все не противоречит, тут скорее социология и политика мешают.
Аноним 17/04/18 Втр 11:47:18  436730
>>436729
Это не только экономике противоречит, но и здравому смыслу. Потому что каждый васян будет в такой модели вкладывать нихуя, надеясь получить как все. И если даже кто-то ещё попытается в такой модели работать, то очень быстро он посмотрит на то, что он весь такой старается, а получает как все, которые делают нихуя, и стараться перестанет - нет никакой мотивации. Но если все производят нихуя, то получается и потреблять всем будет тоже нихуя.
Аноним 17/04/18 Втр 11:54:20  436731
>>436729
У меня только одна идея эксплуатация робота и ИИ человеком. Когда робот работает на физических работах, а человек на умственной и финансовой и на человека трятят много чтобы каждый смог стать учёным.
Аноним 17/04/18 Втр 11:58:21  436732
>>436731
А по факту это всегда и везде (смотри экономики якобы коммунистических стран) эксплуатация человека человеком. Когда въябывают все, а получают единицы.
Аноним 17/04/18 Втр 13:20:46  436735
>>436728
атомное ядро=обычный элемент?
Аноним 17/04/18 Втр 13:23:30  436736
>>436735
Да. Это же не вырожденный нейтронный газ.
Аноним 17/04/18 Втр 16:36:32  436747
>>436730
>Но если все производят нихуя, то получается и потреблять всем будет тоже нихуя.
Вот на осознании этого факта и строится мотивация. Коммунизм предполагает что люди смогут осознать выгоду в в далекой перспективе и им не нужно будет играть в рынок и мотивироваться деньгами, мотивация будет идти напрямую от общего производства.

Аноним 17/04/18 Втр 16:38:55  436748
>>436732
Это потому что все что называется коммунизмом по факту большевизм, а он корнями идет в радикальный социализм, который схож с национал-социализмом: все ресурсы не на благо народу а на укрепление государства.
Аноним 17/04/18 Втр 16:46:07  436750
Почему параллельный пучОк света расходится, а лазер - нет?
Аноним 17/04/18 Втр 16:54:13  436751
>>436750
Лазер тоже расходится.
Аноним 17/04/18 Втр 17:04:18  436752
>>436747
Теорема Нэша, ссыт на лицо коммунизму.
Аноним 17/04/18 Втр 17:27:56  436753
>>436752
Там и Поппер ссал на лицо коммунизму.
Все кому не лень ссут в лицо коммунизма, а коммунизм утирается и продолжает кукарекать "ваша критика - не критика, нельзя критиковать диалектическую теорию примитивной формальной логикой!"
Аноним 17/04/18 Втр 19:28:59  436758
>>436639
>Какой будет оргазм у человека, которому подрочили со сверхсветовой скоростью?
Оргазм пойдёт в обратную сторону. Т.е. вместо того, чтобы замалафить, хуй станет всасывать в себя окружающую материю и превратится в сингулярность. И произойдёт это так быстро, что подопытный ничего не почувствует, потому что мозг подопытного всосётся в хуй быстрее, чем движутся сигналы по нервам.
Аноним 17/04/18 Втр 19:32:09  436759
>>436638
Нельзя. Потому что запрещено.
Аноним 17/04/18 Втр 19:34:32  436760
>>436637
Нельзя. А то будет загрязнение электрических цепей орбитальной станции и проёб техники стоимостью в миллиарды долларов.
Аноним 17/04/18 Втр 19:36:23  436761
>>436633
Да, такое случается. Обычно к вечеру начинают сильнее топить центральное отопление. Особенно зимой.
Аноним 17/04/18 Втр 22:27:19  436765
>>436751
Почему тогда пучек света от ламп не используют для резки металла и в хирургии?
Аноним 18/04/18 Срд 00:47:12  436770
>>436765
Ну в резке и хирургии всё равно используются системы фокусировки лазера, как раз чтобы расходящийся пучок собрать. А используют лазеры потому что проще получить пучек направленного излучения большой мощности. Куда проще чем пытаться собрать ту же мощность излучения из излучающей во все стороны лампы.
Аноним 18/04/18 Срд 00:57:10  436771
>>436765
Потому, чем собственно лазер отличается от лампы - его излучение когерентно. Т.е. фазы световых волн синхронизированы и сам источник излучения испускает фотоны только при прохождении фронта волны, усиливая его. И это даёт намного большую мощность.
Аноним 18/04/18 Срд 01:11:52  436772
>>436771
>И это даёт намного большую мощность.
Не совсем так. Если ты возьмешь лампочку накаливания и хуйнешь в её 100 W то КПД у неё будет близко к 100% (если ты конечно по всем длинам волн проинтегрируешь, включая ИК). У лазера же теоретический максимальный КПД около 30%, т.е. закачав в лазор 100 W в идеальных условиях ты на выходе получишь только 30 W. А у реальных лазеров и того меньше. Разница с лампочкой как раз в том что эти 30 W будут сконцетрированы с малым угловым рассеянием и в единстенной длине волны.
Аноним 18/04/18 Срд 02:48:59  436773
Что-то вы не договариваете про расходимость пучка.
Аноним 18/04/18 Срд 06:08:34  436775
>>435409 (OP)
Я нихуя не понимаю, вид это объективно существующая штука или социальный конструкт? В школе говорили, что разделение на виды определяется способностью давать плодовитое потомство, но последние лет 10 каждый год десятки новостей о том, что мол наши предки и неандертальцы или там денисовцы скрещивались, при этом что неандертальцы что денисовцы считаются отдельными видами по отношению к нашим предкам жившим в тот период с ними. Вот еще наткнулся на такой феномен как кольцевые виды
https://ru.wikipedia.org/wiki/Кольцевой_вид
Тут вообще чистый континум и любое бинарное разделение будет произвольным. Так что же такое вид?
Аноним 18/04/18 Срд 06:33:14  436776
>>436775
Я вычитал в каком-то бложике много лет назад, что на самом деле все люди это один вид хомо эректус. А сапиенсы, неандертальцы и денисовцы - это поздние подвиды эректуса. Я интуитивно согласен с этим мнением.
Аноним 18/04/18 Срд 08:21:19  436779
>>436775
Я тут уже несколько раз пытался объяснить.
"Окончательная ясность" в вопросе о том, какие веточки в хорошо изученной генеалогии каких-то популяций являются видами, какие разновидностями, подвидами, расами, вариететами или просто популяциями - окончательной ясности в подобных вопросах не просто нет. Ее быть не может и никогда не будет. Причем по мере накопления данных и уточнения деталей ясности должно становиться все меньше. Просто потому, что эволюция идет постепенно, дивергенция идет постепенно, несовместимые аллели накапливаются постепенно, репродуктивная изоляция развивается постепенно. "Разновидности" становятся "видами" постепенно.
Еще дедушка Дарвин писал об этом.
Так что все эти разговоры "вид, не вид, подвид, раса" я считаю совершенно бессмысленным занятием. Особенно когда у нас есть не только данные по современным популяциям (один временной срез), а довольно детально реконструирована история популяций в прошлом.
Можно ориентироваться только на зоологические традиции. И ни в коем случае не абсолютизировать получаемые выводы. Например, в зоологии принято популяции, между которыми есть частичная постзиготическая несовместимость, и которые скрещиваются друг с другом лишь эпизодически, считать скорее видами.
Аноним 18/04/18 Срд 10:58:07  436781
Какое взаимодействие участвует, когда нейтрон врезается в ядро и передаёт ему кинетическую энергию для раскола? И слабое взаимодействие - оно только превращает частицы или может стандартно притягивать-отталкивать?
Аноним 18/04/18 Срд 11:01:16  436784
>>436714
А если окунуть её в океан с антигравитационной присадкой, чтоб не притягивался? Насколько громкий и долгий ПШ-Ш-ШШ-Ш будет?
Аноним 18/04/18 Срд 12:31:40  436786
>>436784
Океан нахуй испарится весь а НЗ заметно не охладится.
Аноним 18/04/18 Срд 14:35:28  436788
>>436781
>Какое взаимодействие участвует, когда нейтрон врезается в ядро и передаёт ему кинетическую энергию для раскола?
Сильное.
> слабое взаимодействие - оно только превращает частицы или может стандартно притягивать-отталкивать?
Оно много что может делать, но из-за того что его переносчики (Z и W бозоны) имеют массу - его дальность ограничена временем свободного существования виртуальных Z и W бозонов. Поэтому у слабого взаимодействия сечения такие милипиздрические.
Аноним 18/04/18 Срд 15:14:44  436789
>>436784
Ну ты мог бы и сам докумекать - масса НЗ сравнима с солнечной массой. Вот и прикинь кто победит - тоненькая водная плёночка на крошечной планете или Солнце.
Если ты планируешь остужать НЗ водой, то ты её будешь только греть и увеличивать её массу. Если ты просто планируешь кипятить воду излучением НЗ, то скорость её охлаждения вообще никак не изменится - ей абсолютно изофаллично излучать в космос или в материю.
Аноним 18/04/18 Срд 17:58:23  436792
>>436786
>>436784
Он же взорвётся нахуй эпически, какое испарение? При такой температуре разложение воды будет твёрдо 100%
Аноним 18/04/18 Срд 18:05:42  436793
>>435409 (OP)
Не уверен что это тупой вопрос, но не знаю где спросить еще:

как именно ДНК формируют материал в нужную им структуру?

На ютубе много видео с визуализацией, но там просто летают частицы, попадая туда куда им надо. Какой силой ДНК заставляет их перемещаться?
Аноним 18/04/18 Срд 18:08:19  436794
>>436793
Не понял что тебе не понятно... какой материал? ДНК сама по себе ничего не формирует кроме спирали.
Аноним 18/04/18 Срд 18:17:00  436795
>>436793
ДНК ничего не перемещает, ДНК - это носитель информации. Для производства белка из этой информации у тебя должна присутствовать хуева туча клеточных механизмов. Сначала часть ДНК с нужным кодирующим геном копируется в и-РНК, после чего уже на эту и-РНК садится рибосома и к ней уже одна за другой присоединяются т-РНК с соответствующими аминокислотами которые впоследствии соединяются в нить белковой молекулы.

https://youtu.be/WsofH466lqk
https://youtu.be/5bLEDd-PSTQ

Без всех этих клеточных структур из ДНК нихуя само не насинтезируется.
Аноним 18/04/18 Срд 18:33:09  436796
>>436772
У лампочки накаливания КПД в районе 5%. Почти всё уходит на накаливание. Кэп.
Аноним 18/04/18 Срд 18:34:31  436797
>>436796
Это в видимом свете там 5%.
Остальное в ИК.
Аноним 19/04/18 Чтв 14:50:22  436826
>>436795
Я далек от химии и биологии, мне стало интересно понять, какая сила заставляет работать все эти сложные механизмы в клетке; если дойти до самого простого из них, то что является первоосновой для его движения?
Например, в первом виде на 1:13 говорят "TBP binds into DNA". Мне непонятно, что заставляет этот механизм работать? И что заставляет потом аттачиться ТFIIA и TFIIB?
Вроде разобрался куда копать, если ты не понял о чем я или лень расписывать, не беспокойся..
Аноним 19/04/18 Чтв 16:57:20  436829
>>436826
А, ты про это? Так это просто диффузия в основном.
Аноним 19/04/18 Чтв 17:43:08  436830
>>436829
Одна из нерешенных проблем химии.
«Лучше, чем идеальные» энзимы: Почему скорость реакций с некоторыми энзимами выше, чем скорость диффузии?
Аноним 19/04/18 Чтв 23:04:23  436837
Я правильно понимаю, что вселенная и всё в ней произошло из одного атома, как жизнь пошла из одной клетки?
Аноним 19/04/18 Чтв 23:15:57  436838
>>436837
Нет
Аноним 19/04/18 Чтв 23:17:29  436839
>>436838
Тогда как? И почему в нашей днк есть гены всех растений и животных?
Аноним 19/04/18 Чтв 23:30:57  436840
>>436839
Тоже нет. Только некоторые гены общие есть (с большим количеством различий - они просто похожи и обычно, впрочем не всегда, выполняют схожую функцию) - потому что растения и животные произошли от общего предка.
Только это никоим боком не связано с одним атомом.
Аноним 19/04/18 Чтв 23:35:42  436842
>>436840
Растения появились раньше, чем животные вылезли на сушу.
Аноним 19/04/18 Чтв 23:47:28  436843
>>436842
И? Насекомые вылезли на сушу раньше, моллюски скорее всего тоже. И что? Каким боком по-твоему это влияет на эволюционное древо? Или что, думаешь, в воде секса нет?
Аноним 19/04/18 Чтв 23:56:26  436844
>>436843
А какая связь между сексом и генетической изменчивостью и древом эволюции? У бактерий секса нет ну ок, есть горизонтальный перенос генов у некоторых, но не суть однако это им не мешает в древо выстраиваться.
Аноним 19/04/18 Чтв 23:56:46  436845
>>436843
Растения появились до того, как на земле зародилась животность. После появления кислорода начали появляться насекомые.
Аноним 19/04/18 Чтв 23:58:48  436847
image.png (1321Кб, 2420x915)
>>436842
Аноним 20/04/18 Птн 00:00:19  436848
>>436844
Ага, а ещё насекомые и моллюски - тоже животные, но я сгоряча. Потому тупость из разряда "растения появились раньше животных, потому что первыми вылезли из воды" - это даже для этого треда тупость (потому что вместо того, чтобы спрашивать, школьник лезет утвержать что-либо, что ему взбредёт).
>>436845
Кислород выработали ещё до появления растений и животных, и выработали его цианобактерии.
Аноним 20/04/18 Птн 00:02:57  436850
>>436848
Его было не достаточно.
Аноним 20/04/18 Птн 00:06:21  436851
image.png (39Кб, 300x549)
>>436850
https://en.wikipedia.org/wiki/Great_Oxygenation_Event
Содержание кислорода вышло на приблизительно современный уровень за несколько сот миллионов лет до появления первых эукариот и тем более первых растений.
Аноним 20/04/18 Птн 00:23:55  436854
>>436851
Ладно. Но они живут в океане. Пока не появились растения, чтобы было что есть, животные не выползли из воды на сушу.
Аноним 20/04/18 Птн 00:28:19  436856
>>436854
Ясно, ты либо тролль, либо альтернативно одарённый. В любом случае, это тред для тех, кто хочет что-то узнать, поэтому либо спрашивай, либо в /b/
Аноним 20/04/18 Птн 00:31:37  436857
>>436856
По-твоему животные вылезли и наплодили растения из своей днк? Если бактерии создают кислород, зачем нужны растения? Можно размножать бактерии.
Аноним 20/04/18 Птн 01:00:13  436859
>>436857
Один из видов предположительно архей когда-то приобрёл симбиотических бактерий, ставших впоследствии митохондриями. Затем из этой ветви появилась ветвь уже практически современных эукариот, имеющих ядро. Эти эукариоты большей частью были гетеротрофами, т.е. питались другими организмами или органическими остатками, но не относящиеся ни к животным, ни к растениям, а к простейшим. Затем одна из линий эукариот приобрела ещё одних симбиотических цианобактерий, впоследствии ставших хлоропластами. Однако и эта ветвь ещё относится к простейшим, поскольку обладала смешанными чертами. И уже со временем эти линии разделились. Так что сосни раз, потому что предки растений обладали в большей мере животными чертами (и ранее царство простейших относили к животным) и в любом случае не появились первыми. И сосни два, потому что первым царством, освоившим сушу, были грибы. И в принципе животные вполне могли питаться ими.
> зачем нужны растения
Зачем нужен ты, если бактерии тоже могут создавать говно? Затем, что растения и животные оказались достаточно успешными, чтобы выжить и процветать. Затем, что многоклеточные организмы изначально развились в многоклеточных, потому что кооперация и усложнение даёт преимущества в выживании и распространении генов. Затем, что эволюции и жизни в целом насрать на обоснования существования чего-либо, да и на эффективность тоже - им не насрать на конкуренцию, и из-за этого приходится повышать эффективность.
Аноним 20/04/18 Птн 01:27:33  436861
>>436857
>Если бактерии создают кислород, зачем нужны растения?
Детская неизбирательная телеология.
Аноним 20/04/18 Птн 02:51:09  436865
К слову о происхождении видов... ИМХО, LUСA - последний общий предок всего живого, - вероятно, никогда не существовал. Это, конечно, чистой воды спекуляция, но давайте рассудим - с момента выхода в свет "Происхождения видов" Дарвина филогенетическое дерево видов представлялось всем как... собственно, дерево - один корень, от которого ветвятся виды вплоть до нынешнего состояния. И примерно тогда же появилась концепция LUCA. Однако с тех пор у нас появилась генетика и убедительные доказательства того, что это дерево на самом деле скорее граф с огромным количеством горизонтальных переносов и даже с такими явлениями, как митохондрии и хлоропласты, которые в прошлом были отдельными живыми организмами, а сейчас стали всего лишь органеллами. Но если рассматривать их эволюционное древо, то оно почти повторяет эволюционное древо всех эукариот. Почти. И всё-таки не повторяет. Плюс к тому, у нас есть гибридизация. Ну и, наконец, просто вирусы, капсиды с ДНК и прочие вещи, которые нет-нет, да и обеспечивают горизонтальный перенос. В итоге древо жизни - очень условный конструкт, в значительной мере размазанный и нечёткий.
Так что, кмк, и LUKA - с существенной вероятностью мог быть не каким-то отдельным организмом или, по крайней мере, не чем-то таким, что однозначно и бесспорно являлось бы предком, и даже самый первый в полном смысле живой организм тоже мог быть не единственным, а просто чуть более удачливым "почти в полном смысле" живым комплексом молекул среди миллиардов других почти таких же.
Аноним 20/04/18 Птн 03:19:14  436866
>>436859
У митохондрий и хлоропластов есть своя ДНК?
Аноним 20/04/18 Птн 04:02:45  436867
>>436866
https://ru.wikipedia.org/wiki/Митохондриальная_ДНК
https://ru.wikipedia.org/wiki/Пластом
Аноним 20/04/18 Птн 04:12:15  436868
>>436865
А есть инфа о том когда появился горизонтальный перенос и насколько сложные клеточные механизмы нужны для его осуществления? Просто если есть свидетельсва что горизонтальный перенос требовал слодной многоступенчатой эволюциии это в принципе будет свитетельством что у организмов ближе к LUCA никакого горизонтального переноса быть не могло и вполне может означать что LUCA был и он был один.
Аноним 20/04/18 Птн 11:23:41  436873
>>436868
Есть свидетельства, что он появился до самой ДНК - ещё в эпоху РНК-мира. Потому что это в общем-то проще, чем НЕ передавать генетическую информацию. И сложнее НЕ читать привнесённый фрагмент, если на нём есть маркеры чтения.
Аноним 20/04/18 Птн 13:57:54  436876
>>436867
То есть можно пересаживать пластиды и получать новые организмы, отлично. А что если пересадить хлоропласты животным? Получатся бульбозавры?
Аноним 20/04/18 Птн 14:06:43  436877
>>436876
Такие бульбозавры уже существуют:
Брюхоногий моллюск Elysia chlorotica
Саламандра Ambystoma maculatum
Вроде был кто-то ещё, но не помню как найти.
Аноним 20/04/18 Птн 14:10:57  436878
>>436877
А, хотя нет, саламандра не юзает хлоропласты напрямую, у неё для этого есть симбиотическая бактерия.
Но вот моллюск - именно использует хлоропласты и даже обзавёлся необходимыми для этого генами.
Аноним 20/04/18 Птн 16:25:46  436879
>>436829
>>436830
Работа рибосомы сегодня досконально изучена или нет?
Вот, например, есть видео
https://gizmodo.com/unprecedented-video-shows-how-dna-is-organized-in-real-1823211811
там же мало что видно, лол.
Аноним 20/04/18 Птн 16:38:49  436880
>>436879
У нас мало что досконально изучено, в основном, единичные молекулы и их базовые принципы, связи с другими комплексами, но всё это едва ли даже 10% всех процессов, происходящих в клетках.
Там же есть художественное представление происходящего.
Аноним 20/04/18 Птн 18:44:30  436885
>>436880
То есть читая все эти статьи и и смотря красивые видео про рибосому, надо иметь в виду, что все может оказаться совершенно наоборот (я не против науки, я просто хочу понять, как относиться ко всей этой инфе)?
Аноним 20/04/18 Птн 19:25:59  436888
>>436885
Не, наоборот - вряд ли, очень вряд ли. Скорее, всё может оказаться намного сложнее, с большим количеством нюансов. А смотря видосики - что всё совершенно точно сложнее и хаотичнее.
Аноним 21/04/18 Суб 02:17:58  436910
https://2ch.hk/sci/res/436907.html
https://2ch.hk/sci/res/436907.html
https://2ch.hk/sci/res/436907.html
Аноним 21/04/18 Суб 10:47:43  436919
14125213400740.jpg (67Кб, 604x454)
>>436907 (OP)
sdey 23/06/18 Суб 22:20:57  441456
как объяснить эксперимент с запутанными фотонами? когда их разнесли на большое расстояние за несколько сотен километров и определили спин одного из фотонов, другой фотон мгновенно, без задержки, получил спин, что противоречит основе физики: скорость не может превышать скорость света в вакууме.


Топ тредов
Избранное